You are on page 1of 179

Item: 1 of 25 ~.

I • M k <:] t> al ~· ~
QIO: 2946 .l. ar Previous Next lab 'lifllues Notes Calculator

•1 •
A 42-year-old woman presents to the emergency department because of bilateral leg weakness and lower
•2
back pa in making wa lking difficu lt, and paresthesia involving the toes . On physica l exam ination she has
•3 diminished an kle-jerk and knee-jer k reflexes bi laterally, mild facia l wea kness, and abnorma l sweating, but
•4 has norma l sensation despite the pa resthesia. Her medica l histo ry is notable for a diarrhea l illness 3 weeks before
adm ission . Cereb rospina l fluid analysis shows increased protein and a no rma l cel l count.
•5
•6
Wh ich of the fo llowing is the function of the cells targeted by antibodies in th is cond it ion'
•7
•8 :

•9
• 10 A. Conduction of nerve impu lses from central nervous system neu rons to pe ripheral muscle
· 11 B. Conduction of nerve impu lses from the cereb ral cortex to spina l motor neu rons
• 12
C. Contraction in response to changes in membrane potential
• 13
• 14 D . Mye lination of centra l nervous system axons
• 15 E. Mye lination of pe riphera l nervous system axons
• 16
• 17

• 18
• 19
• 20
• 21

6
lock
s
Suspend
0
End Block
Item: 1 of 25 ~. I • M k <:] t> al ~· ~
QIO: 2946 .l. ar Previous Next lab 'lifllues Notes Calculator

1 •

•2 The correct answer is E. 71 °/o chose this.


•3 Gu illain-Barre syndrome is an autoimmune disease mediated by T lymphocytes that target Schwann cells of the
•4 peripheral nervous system (PNS). Schwann cells myel inate neurons of the PNS, but not those of the central
nervous system (CNS); ol igodendrocytes are responsible for CNS neuron myelination. The classic presentation
•5
of Gui lla in-Barre syndrome is an acute ascend ing paralysis. Most cases occur 2-4 wee ks after a respi rato ry or
•6 gastrointestina l infection. Campylobacter jejuni is a common cause of gastroenteritis, the most common
•7 pathogen associated with Gu illain-Barre synd rome. Cytomegalovi rus, Epstein-Barr virus, Mycoplasma
•8 pneumoniae, and recent immunizations are also risk factors. Gui llain-Barre is currently thought to occu r
because of immune responses against non -self-antigens that are misdirected toward the host (molecula r
•9
mimicry). About 25% of patients require respiratory support as the paralysis ascends over hours to days, but
• 10 symptoms usua lly subside ove r the ensu ing weeks . Current treatment options include intravenous
· 11 immunog lobul in or plasmaphe resis, both of wh ich diminish autoantibody levels. Corticoste roids are not effective.
Mycoplasma pneumoniae Campylobacter jejuni Plasmapheresis Epstein-Barr virus Autoimmune disease Cytomegalovirus Molecular mimicry
• 12
Immunoglobulin therapy Schwann cell Central nervous system Peripheral nervous system Gastroenteritis Neuron lymphocyte Antibody Myelin Oligodendrocyte
• 13
Corticosteroid Campylobacter Autoimmunity Pathogen Paralysis Autoantibody Nervous system T cell Intravenous therapy Mycoplasma Virus Gastrointestinal tract
• 14
Human gastrointestinal tract Infection
• 15
• 16 A is not correct. 11% chose this .
• 17 Lower moto r neurons are not di rectly targeted in Gui lla in-Ba rre syndrome . However, the loss of myelin leads to
• 18
lower motor neu ron dysfunction and degeneration that are responsible for the disease symptoms .
Myelin lower motor neuron Neuron Motor neuron
• 19
• 20 B is not correct. 5°/o chose this .
• 21
Because uppe r motor neurons (UMNs) are part of the CNS, they are not ta rgeted by autoantibodies in Guillain -
• R:u·,...Q e\/nrl r""t"'\n"'\C. Mnr""C.t"\\/C.t"' I I M 1\1 IQe i nne h =a \ I C. :J~ rh : u'"':H""'h Qri et-ir eQt nF fi nrl i nne rh =at- rl i fFQr fr"'I'\I"'Y'\ thl"\ec. ,..., f ln\II.IQr"'

6
lock
s
Suspend
0
End Block
Item: 1 of 25 ~. I • M k <:] t> al ~· ~
QIO: 2946 .l. ar Previous Next lab 'lifllues Notes Calculator

1 •
. . . .
Lower motor neurons are not directly targeted in Gui lla in-Barre syndrome . However, the loss of myelin leads to
•2
lower motor neuron dysfunction and degeneration that are responsible for the disease symptoms.
•3 Myelin lower motor neuron Neuron Motor neuron

•4
B is not correct. 5°/o chose this.
•5
Because upper motor neu rons (UMNs) are part of the CNS, they are not targeted by autoantibodies in Guilla in-
•6 Barre synd rome. Moreove r, UMN lesions have a characte ristic set of find ings that differ from those of lower
•7 motor neu ron (LMN) lesions. With UMN lesions, one expects to see hyperreflexia, not hyporeflexia and, in
•8
addition, one wou ld loo k for the presence of spasticity and Babinski sign. However, in some situations, patients
may present with both UMN and LMN findings, as is the case with amyotrophic lateral sclerosis .
•9 Plantar reflex Amyotrophic lateral sclerosis Hyporeflexia Hyperreflexia Spasticity lower motor neuron Neuron Motor neuron Central nervous system
• 10 Upper motor neuron
· 11
C is not correct. 3°/o chose this .
• 12
Muscle is not directly ta rgeted in Guillain-Ba r re syndrome, although it does atrophy in response to denervation .
• 13 Myasthenia gravis is an autoimmune disorder in which autoantibodies target acetylchol ine recepto rs at the
• 14 neuromuscu lar junction . Myasthenia gravis most often presents with muscle wea kness intensified by muscle
• 15 use .
Myasthenia gravis Neuromuscular junction Autoimmune disease Acetylcholine Autoantibody Autoimmunity Atrophy Muscle weakness Receptor (biochemistry)
• 16
Muscle
• 17

• 18 D is not correct. 10% chose this .


• 19 Ol igodendrocytes myelinate CNS neurons. Autoantibodies target oligodend rocytes in mu ltip le sclerosis, which
would present as neurological deficits separated in space and time. Guillain-Ba r re syndrome is a PNS system
• 20
disease that targets Schwann cel ls, not ol igodendrocytes .
• 21 Multiple sclerosis Schwann cell Oligodendrocyte Autoantibody Central nervous system Neuron Neurology

6
lock
s
Suspend
0
End Block
Item: 1 of 25 ~. I • M k <:] t> al ~· ~
QIO: 2946 .l. ar Previous Next lab 'lifllues Notes Calculator

1 • motor neuron (LMN) lesions. With UMN lesions, one expects to see hyperreflexia, not hyporeflexia and, in
•2 addition, one wou ld look for the presence of spasticity and Babinski sign. However, in some situations, patients
•3
may present with both UMN and LMN findings, as is the case with amyotrophic lateral sclerosis.
Plantar reflex Amyotrophic lateral sclerosis Hyporeflexia Hyperreflexia Spasticity lower motor neuron Neuron Motor neuron Central nervous system
•4
Upper motor neuron
•5
•6 C is not correct. 3°/o chose this .
•7
Muscle is not directly targeted in Guillain-Barre syndrome, although it does atrophy in response to denervation .
Myasthenia gravis is an autoimmune disorder in which autoantibodies target acetylchol ine recepto rs at the
•8
neuromuscu lar junction . Myasthenia gravis most often presents with muscle weakness intensified by muscle
•9 use.
Myasthenia gravis Neuromuscular junction Autoimmune disease Acetylcholine Autoantibody Autoimmunity Atrophy Muscle weakness Receptor (biochemistry)
• 10
Muscle
· 11
• 12 D is not correct. 10% chose this .
• 13 Ol igodendrocytes myelinate CNS neurons. Autoantibodies target oligodend rocytes in mu ltip le sclerosis, which
• 14 would present as neurological deficits separated in space and time. Guillain-Barre syndrome is a PNS system
disease that targets Schwann cel ls, not ol igodendrocytes .
• 15
Multiple sclerosis Schwann cell Oligodendrocyte Autoantibody Central nervous system Neuron Neurology
• 16
• 17

• 18 Bottom Line:
• 19 Acute ascend ing para lysis after an infection is the classic presentation of Guil lain-Barre syndrome . It is caused
• 20
by autoimmune antibodies aga inst Schwann cel ls, which are responsible for PNS mye lination .
Schwann cell Paralysis Antibody Myelin Autoimmune disease Autoimmunity Infection
• 21

6
lock
s
Suspend
0
End Block
Item: 1 of 25 ~ 1 • M k -<:J 1>- Jil ~· !:';-~
QIO: 2946 ..L ar Pre v ious Next Lab fli!ltues Notes Calcula t o r
&
- - -
&
1
.2
FA17 p 493.2
•3
Acute inflammatory lost common subtype of Guillain-Barre Associated with infections (eg, Campylobacler
·4 demyelinating syndrome. Autoimmune condition that jejuni, viral) - autoimmune attack of
•5 polyradiculopathy destroys Schwa11n cells - inAammation and peripheral myelin due to molecular mimicry,
•6 demyelination of peripheral nerves and motor inoculations, and stress, but no definitive link
.7 fibers. Results inS) mmetric ascending muscle to pathogens.
weakness/paralysis and depressed tendon Respiratory support is critical until recovery.
·8
reAexes beginning in Jo,,er e'\tremitics. Facial Additional treatment: plasmapheresis, IV
.9 paralysis in 50% of cases. t\ lay sec autonomic immunoglobulins. 1 o role for steroids.
• 10 dysregulation (eg, cardiac irregularities,
• 11 hrpertension, hypotension) or sensory
• 12 abnormalities. Almost all patients survive; the
majority recover completely after weeks to
• 13
months.
• 14
t CSF protein with norma l cell count
• 15 (albumi nocytologic dissociation). t protein
• 16 may cause papi lledema .
• 17
. 18 FA17 p 464.2

• 19 Schwann cells Each Sch,,·ann cell myelinates only I P rs axon. J\ lay be injured in Guillain-Barre syndrome.
• 20 Also promote axonal regeneration. Derived
• 21 from neural crest.

a
Lock
s
Suspend
8
End Bl ock
Item: 1 of 25 ~ 1 • M k -<:J 1>- Jil ~· !:';-~
QIO: 2946 ..L ar Pre v ious Next Lab fli!ltues Notes Calcula t o r

1
&
-- ... "' "'
&

(albuminocytologic dissociation). t protein


•2 may cause papilledema.
•3

·4 FA17 p 464.2
•5
Schwann cells Each Schwann cell myelinates only I PN'S axon. ?\ lay be injured in Guillain-Barre syndrome.
•6 Also promote axonal regeneration. Derived
•7 from neural crest.
·8
.9
• 10
• 11
• 12 FA17 p 464.3
• 13
Oligodendrocytes lyclinates axons of neurons in C S. Each Derived from neuroectoderm.
• 14 oligodendrocyte can myelinatc many axons "Fried egg" appearance histologically.
?Node of Ranv_ie_r~-
• 15 (- 30). Predominant type of glial cell in white Injured in multiple sclerosis, progressive
• 16 matter. multifocal leukoencephalopathy (PML),
leukodystrophies.
• 17
• 18 I-
Ollgodendrocyte
• 19
• 20
• 21

a
Lock
s
Suspend
8
End Bl ock
Item: 2 of 25 ~ 1 • M k -<:J 1>- Jil ~· !:';-~
QIO: 2655 ..L ar Pre v ious Next Lab fli!ltues Notes Calcula t o r
& &
1
A 45-year-old ma le presents wit h a 1-week history of altered cognitive function and headaches. He was
.2
diagnosed with human immunodeficiency virus (HIV) infection 10 years ago. His partner reports that the
•3 patient has been compliant with his treatment and he has not taken any new med ications recently.
·4
•5
•6
.7
·8
.9
• 10
• 11

• 12
• 13
• 14
• 15
• 16
• 17 At the center of the slide, cells are shown that are activated in response to centra l nervous system damage. What
. 18 is the embryologic orig in of this type of cell?
• 19 :
• 20 A. Endoderm
• 21 B. Mesoderm

a
Lock
s
Suspend
8
End Bl ock
•2
•3

·4
•5
•6
•7

·8
.9
• 10
• 11
At the center of the slide, cells are shown that are activated in response to central nervous system damage. What
• 12
is the embryologic origin of this type of cell?
• 13
:
• 14
A. Endoderm
• 15
• 16
B. Mesoderm

• 17 C. Neural crest
• 18 D. Neuroectoderm
• 19
E. Surface ectoderm
• 20
• 21

a
Lock
s
Suspend
8
End Block
Item: 2 of 25 ~ 1 • M k -<:J 1>- Jil ~· !:';-~
QIO: 2655 ..L ar Prev ious Next Lab fli!ltues Notes Calculat o r

& &
1
2 The correct answer is B. 43°/o chose this.
•3 The cel ls w ith elongated, hyperchromatic nuclei at the center of the image
·4 (see blue circles in the image to the r ight) are microg lia . Microg lia are the
resident phagocytes of the central nervous system. Th is patient has an HIV-
•5
associated neurocognitive disorder, which is associated with neuronal
•6 damage. Microglia migrate to areas of tissue damage to help clear away dead
•7 and dying cells. They also release cytokines that stimulate the immune
system to respond to the area of injury. Like other macrophages, microglia
·8
differentiate from circulating blood monocytes and are thus of mesodermal
.9 origin. Other components of the central nervous system derived from the
• 10 mesoderm include the dura mater and the connective tissue of the periphera l
• 11 nervous system (endo-, peri -, and epineuria) .
Dura mater Microglia Mesoderm Central nervous system Peripheral nervous system Macrophage Cell nucleus
• 12
Immune system Phagocyte Cytokine Monocyte Nucleus (neuroanatomy) Germ layer Connective tissue
• 13
Nervous system Dysplasia HIV-associated neurocognitive disorder
• 14
• 15 A is not correct. 5°/o chose this .
• 16 The endoderm gives rise to the epithelia l lining of the gastroint estinal tract, urogen it al system, and respiratory
tract. The majority of the gastrointestina l system, includ ing the pancreas and liver is endoderma l in orig in .
• 17
Additional ly, the thyroid gland and pharyngeal pouches are derived from endoderm .
• 18 Endoderm Human gastrointestinal tract Gastrointestinal tract Thyroid Pancreas Genitourinary system Respiratory tract Uver Epithelium Germ layer
• 19 Squamous epithelial cell Pharynx Pharyngeal pouch (embryology) Gland
• 20
C is not correct. 20°/o chose this.
• 21
• NPIIr~l rrPc:t rPIIc: nivP ric:P tn C:PVPr~l tic:<:IIPC: nf thP nPntnll<: c:vc:tPm inrll1rlinn nPrinhPr~l n~nnli~ ~ffPrPnt

a
Lock
s
Suspend
8
End Bl ock
Item: 2 of 25 ~. I • M k <:] t> al ~· ~
QIO: 2655 .l. ar Previous Next lab 'lifllues Notes Calculator
1 1
1 •
Squamous epithelial cell Pharynx Pharyngeal pouch (embryology) Gland
2
•3 C is not correct. 20% chose this.
•4 Neural crest cells give r ise to several t issues of the nervous system, including periphera l gangl ia, afferent
sensory nerves (dorsal root) , and Schwann cel ls.
•5
Neural crest Schwann cell Ganglion Nervous system Afferent nerve fiber Sensory neuron Anatomical terms of location Dorsum (biology)
•6
D is not correct. 27% chose this.
•7
The neuroectoderm gives rise to the posterior pituitary, central nervous system neurons, oligodendrocytes,
•8
astrocytes, ependyma l cel ls, and pineal gland. Microg lia, however, arise from monocytes and are thus of
•9 mesodermal origin .
• 10 Pineal gland Microglia Central nervous system Neuroectoderm Posterior pituitary Ependyma Oligodendrocyte Astrocyte Neuron Germ layer Nervous system

· 11 Pituitary gland Monocyte Mesoderm

• 12 E is not correct. 5°/o chose this.


• 13 The surface ectoderm is an embryolog ic structure that gives r ise to the lens of the eye, epidermis, and anterior
• 14 pitu itary .
Ectoderm Anterior pituitary Epidermis Surface ectoderm Embryology Epidermis (zoology) lens (anatomy) Pituitary gland Anatomical terms of location
• 15
• 16
• 17 Bottom Line:
• 18 Microg lia are mesoderm-derived phagocytes of the centra l nervous system. They t ransform into ameboid cel ls
• 19 and orchestrate the clean-up effort in response to tissue damage .
Microglia Central nervous system Phagocyte Nervous system Amoeba
• 20
• 21

6
lock
s
Suspend
0
End Block
Item: 2 of 25 ~. I • M k <:] t> al ~· ~
QIO: 2655 .l. ar Previous Next lab 'lifllues Notes Calculator
• Pituitary gland Monocyte Mesoderm
1

2 E is not correct. 5°/o chose this.


•3 The surface ectoderm is an embryolog ic structure that gives rise to the lens of the eye, epidermis, and anterior
.4 pitu itary.
Ectoderm Anterior pituitary Epidermis Surface ectoderm Embryology Epidermis (zoology) lens (anatomy) Pituitary gland Anatomical terms of location
•5
•6
.7 Bottom Line:
•8 Microg lia are mesoderm-derived phagocytes of the centra l nervous system. They transform into ameboid cel ls
•9 and orchestrate the clean-up effort in response to tissue damage .
Microglia Central nervous system Phagocyte Nervous system Amoeba
• 10

· 11
• 12
liU;fiiJI•J for year: 2017 " ]
• 13 FI RST AID FAC T S .

• 14
• 15 FA11 p463.4

• 16 Microglia Phagocytic scavenger cells of CNS I IIV-infected microglia fuse to form


• 17 (mesodermal, mononuclear origin). Activated multinucleated giant cells in Ci S.
in response to tissue damage. lot readily
• 18
discernible by 1 issl stain.
• 19
• 20
• 21

6
lock
s
Suspend
0
End Block
Item: 3 of 25 ~ 1 • M k -<:J 1>- Jil ~· !:';-~
QIO: 2657 ..L ar Pre v ious Next Labfli!llues Notes Calcula t o r

IAA]
& &
1
A 25-year-old woman present s t o the clinic because of a 2-day history of visual dist ur bance . Physica l
2
examination reveals left-sided monocu lar vision loss, bilateral internuclear ophthalmoplegia, and a slight
•3 intention t remor . Lumbar puncture shows an increased level of lgG in the cerebrospinal f luid.
·4
•5 Which of the following cell types is directly and significantly affected in this patient's d isease?
•6
:
•7 A. Microg lia
·8
B. Myocytes
.9
C. Neu rons
• 10
• 11 D. Oligodendrocytes
• 12 E. Schwann
• 13
• 14
• 15
• 16
• 17
• 18
• 19
• 20
• 21

a
Lock
s
Suspend
8
End Bl ock
Item: 3 of 25 ~. I • M k <:] t> al ~· ~
QIO: 2657 .l. ar Previous Next lab 'lifllues Notes Calculator

1 •

2 The correct answer is D. 78°/o chose this.


3 The patient is presenting with symptoms and find ings consistent with multiple sclerosis (MS), a demyelinating
•4
disease whose manifestations include optic neurit is (sudden visual loss), medial long itud inal fascicu lus
syndrome (internuclear ophthalmopleg ia), hemiparesis, hemisenso ry symptoms, and/o r bowel and bladde r
•5
incontinence. Patients with MS frequently have slightly increased protein levels with ol igoclona l I gG bands in the
•6 cerebrospinal fluid. The classic t riad of MS consists of (1) nystagmus, (2) intention tremor, and (3) scanning
•7 speech (speech in which wo rd syllables are separated by pauses) . MS affects oligodendrocytes, cells that
function to myel inate axons of the central nervous system. Patients with MS classical ly have plaques (areas
•8
of oligodend rocyte loss and reactive gl iosis) scatte red th roughout their CNS white matter .
•9 Oligodendrocyte Optic neuritis Intention tremor Multiple sclerosis Hemiparesis Gliosis Medial longitudinal fasciculus Nystagmus Cerebrospinal fluid

• 10 Demyelinating disease Ophthalmoparesis Central nervous system White matter Axon Tremor Nervous system Visual impairment Immunoglobulin G Protein

· 11 Urinary bladder Urinary incontinence Fecal incontinence Internuclear ophthalmoplegia Gastrointestinal tract Myelin

• 12
A is not correct. 5°/o chose this .
• 13
Microg lia, the resident phagocytes of the central ne rvous system, are ind irectly affected by multiple sclerosis. I n
• 14 addition to their immunologic role, they are believed to have a neuroprotective and repai r ro le in response to
• 15 oligodend rocyte destruction. Howeve r, they are not the primary cel l population affected in mu ltip le sclerosis .
Oligodendrocyte Multiple sclerosis Microglia Central nervous system Neuroprotection Phagocyte Nervous system Immune system Immunology
• 16
• 17 B is not correct. 2°/o chose this .
• 18 Diseases that affect myocytes fa ll within the family of myopathies, including muscu lar dystrophy and
• 19 po lymyositis. Myocytes are not affected in mu ltip le scle rosis .
Multiple sclerosis Polymyositis Myopathy Myocyte
• 20
• 21 C is not correct. 6°/o chose this .

6
lock
s
Suspend
0
End Block
Item: 3 of 25 ~. I • M k <:] t> al ~· ~
QIO: 2657 .l. ar Previous Next lab 'lifllues Notes Calculator

1 Urinary bladder Urinary incontinence Fecal incontinence Internuclear ophthalmoplegia Gastrointestinal tract Myelin

2
A is not correct. 5°/o chose this.
3
Microgl ia, the resident phagocytes of th e central nervous system, are indirectly affect ed by multiple sclerosis . I n
.4 addition t o their immunologic role, they are believed to have a neu roprotective and repa ir role in response t o
•5 oligodendrocyt e dest ruction . However, t hey are not the primary cell popu lation affect ed in mult iple sclerosis .
Oligodendrocyte Multiple sclerosis Microglia Central nervous system Neuroprotection Phagocyte Nervous system Immune system Immunology
•6
.7 B is not correct. 2°/o chose this.
•8 Diseases that affect myocyt es fal l within the fami ly of myopathies, includ ing muscu lar dystrophy and
•9 polymyosit is. Myocytes are not affected in mult iple sclerosis .
Multiple sclerosis Polymyositis Myopathy Myocyte
• 10

· 11 C is not correct. 6°/o chose this.


• 12
Although t he cells that myelinate neurons are affect ed in mult iple sclerosis, neu rona l axons appea r to be
relatively preserved .
• 13 Multiple sclerosis Neuron Axon
• 14
E is not correct. 9°/o c hose this .
• 15
Schwann cel ls myel inat e peripheral nervous system (PNS) axons . Unli ke ol igodend rogl ia, Schwann cells
• 16
myelinate only one PNS axon each and serve to promote axona l regeneration . MS is a demyelinating disease of
• 17 the central nervous system and thus does not affect Schwann cel ls. Peripheral myelination can be affected by
• 18 Gu illain-Barre synd rome (GBS), wh ich often man ifests as a rapid ly ascending weakness after a vi ral illness .
Oligodendrocyte Axon Demyelinating disease Peripheral nervous system Schwann cell Myelin Nervous system Neuroregeneration Virus
• 19
• 20
• 21 Bottom Line:

6
lock
s
Suspend
0
End Block
Item: 3 of 25 ~. I • M k <:] t> al ~· ~
QIO: 2657 .l. ar Previous Next lab 'lifllues Notes Calculator
-·· • • • • •• • .. - • ,. • • _,. - • •• • 1' - • ·- • •• •

1
oligodend rocyte destruction. Howeve r, they are not the primary cel l population affected in mu ltip le sclerosis.
2 Oligodendrocyte Multiple sclerosis Microglia Central nervous system Neuroprotection Phagocyte Nervous system Immune system Immunology

3
B is not correct. 2°/o chose this.
•4
Diseases that affect myocytes fa ll within the family of myopathies, including muscu lar dystrophy and
•5 po lymyositis. Myocytes are not affected in mu ltip le scle rosis.
•6 Multiple sclerosis Polymyositis Myopathy Myocyte

•7
C is not correct. 6°/o chose this .
•8 Although the cel ls that myel inate neurons are affected in mu ltip le sclerosis, neuronal axons appear to be
•9 re lative ly preserved .
Multiple sclerosis Neuron Axon
• 10

· 11 E is not correct. 9°/o chose this.


• 12 Schwann cel ls myelinate peripheral nervous system (PNS) axons . Un like ol igodendrogl ia, Schwann cells
• 13
myel inate on ly one PNS axon each and se rve to promote axonal regeneration. MS is a demyel inating disease of
the central nervous system and thus does not affect Schwann cel ls. Periphe ral myel ination can be affected by
• 14
Gu illain-Barre syndrome (GBS), which often manifests as a rapidly ascend ing wea kness after a vira l illness .
• 15 Oligodendrocyte Axon Demyelinating disease Peripheral nervous system Schwann cell Myelin Nervous system Neuroregeneration Virus

• 16
• 17
Bottom Line:
• 18
MS is a demyel inating disorder of the CNS characte rized by ol igodendrocyte loss and reactive g liosis. I ts
• 19
neuro log ic symptoms include scanning speech, intention tremor, and nystagmus .
• 20 Oligodendrocyte Intention tremor Gliosis Nystagmus Tremor Central nervous system Demyelinating disease Neurology Myelin

• 21

6
lock
s
Suspend
0
End Block
Item: 3 of 25 ~ 1 • M k -<:J 1>- Jil ~· !:';-~
QIO: 2657 ..L ar Pre v ious Next Labfli!llues Notes Calcula t o r

1
&
' . &

2
FA17 p 464.3
3
Oligodendrocytes Iyelinates axons of neurons in C 1 S. Each Dcri,·ed from neuroectoderm.
·4 oligodendroc}te can myelinate many axons "Fried egg" appearance histologically.
_.c:Node of RanVJe
_. _r _ _
•5 (- 30). Predominant type of glial cell in white Injured in multiple sclerosis, progressive
•6 Axon matter. multifocalleukoencephalopathy ( P.\ IL),
•7 leukodystrophies.

·8
J_ _
Obgodendrocyte
.9
• 10
• 11
• 12 FA17 p 493.1
• 13 Multiple sclerosis Autoimmune inAammat ion and demrclination ofCNS (brain and spinal cord). Patients can
• 14 present with optic neuritis (sudden loss of vision resulting iu Nlarcus Cunn pupils}, 1 0,
• 15 hemiparesis, hemisensory symptoms, bladder/bowel dysfunction. Symptoms may exacerbate with
increased body temperature (eg, hot bath, exercise). Relapsing and remitting is most common
• 16
clinical course. lVlost often affects women in their 20s and 30s; mo re common in Caucasians
• 17
living farther from equator. 'eck Aexion may precipitate sensation of electric shock running
• 18 down spine (Lhermitte phenomenon).
• 19 Charcot triad ofMS is a Sll\ :
• 20 • Scanning speech
Intention tremor (also Incontinence and Internuclear ophthalmoplegia)
• 21
• • v .. C' .... ,.. ......... ,.r

a
Lock
s
Suspend
8
End Bl ock
Item: 3 of 25 ~ 1 • M k -<:J 1>- Jil ~· !:';-~
QIO: 2657 ..L ar Pre v ious Next Labfli!llues Notes Calcula t o r
& &
1
FA17 p 4 93.1
2
Multiple sclerosis Autoimmune inAammation and dem)elinalion of CNS (brain and spinal cord). Patients can
3 present with optic neuritis (sudden loss of vision resulting in Ylarcus Gunn pupils), !NO,
·4 hemiparesis. hemisensory symptoms, bladder/bowel drsfunction. Srmptoms may exacerbate with
•5 increased body temperature (cg, hot bath, exercise). Relapsing and remitting is most common
•6 clinical course. ~ lost often affects women in their 20s and 30s; more common in Caucasians
)i,'ing farther from equator. ~ eck flexion may precipitate sensation of electric shock running
.7
down spine (Lhermitte phenomenon).
·8 Charcot triad of~ IS is a SI ~ :
.9 • Scanning speech
• 10 Intention tremor (also Incontinence and Internuclear ophthalmoplegia)
• 11 '\fystagmus
• 12 FINDINGS t IgG level and myelin basic protein in C F". Oligoclonal bands are diagnostic. MRI is gold
• 13
standard. Periventricular plaques rJ (areas of oligodendrocyte loss and reactive gl iosis) with
preservation of axons. Multiple white matter lesions disseminated in space and time.
• 14
• 15
• 16
• 17
. 18
• 19
TREATMENT Slow progression" ith disease-modifying therapies (eg, ~ -i nterferon, glatiramer, natalizumab). Treat
• 20 acute flares with I steroids. S) mptomatic treatment for neurogen ic bladder (catheterization,
• 21 muscarinic antagonists), spasticit) (baclofen, CASAR receptor agonists), pain (TCAs,

a
Lock
s
Suspend
8
End Bl ock
Item: 4 of 25 ~ 1 • M k -<:J 1>- Jil ~· !:';-~
QIO: 4423 ..L ar Pre v ious Next Lab fli!ltues Not es Calcula t o r

2
A 25-year-old woman presents to her primary care physician with frequent headaches that worsen when she IAA]
bends over. She says that she often burns the back of her arms wh ile cooking but never notices until her
3 husband says something . She has looked up her symptoms on line, and is very worried that she has MS. She
.4 is referred to a neurologist, where an MRI revea ls the congenital anatomical malformation seen below.
•5
•6
.7
·8
.9
• 10
• 11

• 12
• 13
• 14
• 15
• 16
Sensory deficits are due to impingement upon which part of the spinal cord?

• 17 :

. 18
A . Anterior horn cel ls
• 19 B. Dorsa l columns
• 20 C. Dorsa l root gangl ia
• 21
• n C::ni nnroroholl::a ... to- :::or+

a
Lock
s
Suspend
8
End Bl ock
Item: 4 of 25 ~ 1 • M k -<:J 1>- Jil ~· !:';-~
QIO: 4423 ..L ar Pre v ious Next Lab fli!ltues Not es Calcula t o r

1
&
........ .. . · - ......... - . . . . . ... &

2
3
•4
•5
•6
•7

·8
.9
• 10
• 11
• 12
Sensory deficits are due to impingement upon wh ich part of the spina l cord?
• 13
:
• 14
A. Anterior horn cel ls
• 15
• 16
B. Dorsa l columns

• 17 C. Dorsa l root gang lia


• 18 D. Spinocerebellar tract
• 19
E. Spinothalamic tract
• 20
• 21

a
Lock
s
Suspend
8
End Bl ock
Item: 4 of 25 ~. I • M k <:] t> al ~· ~
QIO: 4423 .l. ar Previous Next lab 'lifllues Notes Calculator

1 •

2 The correct answer is E. 61 °/o chose this.


3 The condition being descr ibed is an Arnold-Ch iari malformation (displacement of the hindbrain, specifically the
cerebel lar tonsils and medu lla, through the foramen magnum) . This is often associated with syringomyelia,
4
w hich common ly affects the C8-Tl level. Syringomyel ia can impinge on the crossing fibers of the spinothalamic
•5 tract, resulting in bilatera l loss of pain and temperature sensation in the upper extremit ies.
•6 Syringomyelia Spinothalamic tract Arnold-Chiari malformation Foramen magnum Cerebellar tonsil Medulla oblongata Cerebellum Hindbrain

•7 A is not correct. 6°/o chose this.


•8 Anterior horn cel ls may also be damaged in syr ingomyel ia and resu lt in atrophy of the intrinsic hand muscles .
•9 However, this is a motor deficit and not a sensory deficit .
Syringomyelia Anterior grey column Atrophy Hand Anatomical terms of location
• 10

· 11 B is not correct. 17% chose this.


• 12 The dorsal co lumns are involved in proprioception and may be damaged in tabes dorsa lis associated with
• 13
tertiary syphilis .
Tabes dorsalis Proprioception Syphilis Posterior column Dorsum (biology) Anatomical terms of location
• 14
• 15 C is not correct. 8°/o chose this .
• 16
Dorsa l root gangl ia conta in the cell bodies of sensory neu rons. Loss of conduction to the dorsal root ganglia is
the cause of some of the key symptoms of Charcot-Marie-Tooth neuropathy. Inherited defects in the myel in
• 17
sheath impair the conduction of signa ls that wou ld norma lly be transmitted to the dorsa l root gangl ion, resulting
• 18 in a loss of limb sensation .
Dorsal root ganglion Myelin Soma (biology) Ganglion Neuron Sensory neuron Peripheral neuropathy Anatomical terms of location Genetic disorder
• 19
• 20 Dorsum (biology)

• 21

D is not correct. 8°/o chose this .
6
lock
s
Suspend
0
End Block
Item: 4 of 25 ~. I • M k <:] t> al ~· ~
QIO: 4423 .l. ar Previous Next lab 'lifllues Notes Calculator

1 •
B is not correct. 17% chose this.
2 The dorsal columns are involved in proprioception and may be damaged in tabes dorsa lis associated with
3 tertiary syphil is.
4 Tabes dorsalis Proprioception Syphilis Posterior column Dorsum (biology) Anatomical terms of location

•5 C is not correct. 8°/o chose this.


•6 Dorsa l root gangl ia conta in the cell bodies of sensory neu rons. Loss of conduction to the dorsal root ganglia is
•7 the cause of some of the key symptoms of Charcot-Marie-Tooth neuropathy. Inherited defects in the myelin
sheath impair the conduction of signa ls that would normally be transmitted to the dorsal root ganglion, resulting
•8
in a loss of limb sensation .
•9 Dorsal root ganglion Myelin Soma (biology) Ganglion Neuron Sensory neuron Peripheral neuropathy Anatomical terms of location Genetic disorder
• 10 Dorsum (biology)
· 11
D is not correct. 8°/o chose this .
• 12
The spinocerebel lar tracts are composed of axons that originate in the spinal cord and ter minate in the
• 13 ipsilateral cerebel lum. These structures are essential for mainta ining the sense of propr ioception.
• 14 Spinocerebellar ataxia is caused by a variety of mutations, but all result in a progressive loss of coordination .
Spinocerebellar ataxia Proprioception Ataxia Cerebellum Spinocerebellar tract Spinal cord Anatomical terms of location Axon Ipsilateral Mutation
• 15
• 16
• 17 Bottom Line:
• 18
The fibers of the spinothalamic tract that cross at the anterior white commissure can be damaged by the
• 19 enlargement of the central canal seen in syringomyelia .
Anterior white commissure Syringomyelia Spinothalamic tract Central canal Commissure
• 20
• 21

6
lock
s
Suspend
0
End Block
Item: 4 of 25 ~ 1 • M k -<:J 1>- Jil ~· !:';-~
QIO: 4423 ..L ar Pre v ious Next Lab fli!ltues Not es Calcula t o r
& &
1

2 FA17 p 462.2
3
Syringomyelia Cystic cavity (syrinx) within central canal Syrinx =tube, as in syringe.
4 of spinal cord (yellow arrow in ). Fibers ]\ lost common at C8-Tl.
•5 crossing in anterior white commissure
•6 (spinothalamic tract) are I) pica II} damaged
.7 first Results in a "cape-l ike," bilateral loss
of pain and temperature sensation in upper
·8
extremities (fine touch sensation is preserved).
.9 ssociated with Chiari mal format ions (red
• 10 arrow shows low-lying cerebellar tonsils in fJ).
• 11 trauma, and tumors.
• 12
• 13
• 14
• 15
• 16 FA17 p 500.1
• 17 Spinal cord lesions
AREA AFFECTED DISEASE CHARACTERISTICS
. 18
• 19
Poliomyelitis and Werdnig-Hoffmann Congenital degeneration of anterior horns of spinal
disease cord. L I ' lesions only. "Floppy baby" with marked
• 20
• 21
• •• hypotonia and tongue fasciculations. Infantile type
has median age of death of 7 months. utosomal

a
Lock
s
Suspend
8
End Bl ock
Item: 4 of 25 ~. I • M k <:] t> al ~· ~
QIO: 4423 .l. ar Previous Next lab 'lifllues Notes Calculator

1 • •
FA17 p 500.1
2
Spinal cord lesions
3 AREA AFFECTED DISEASE CHARACTERISTICS
4 Poliomyelitis and Werdnig-Hoffmann Congenital degeneration of anterior horns of spinal
•5 disease cord. LM I lesions on ly. "Floppy baby" with marked
hypotonia and tongue fasciculations. Infantile type
•6
•7 •• has median age of death of 7 months. Autosomal
recessive inheritance.
•8 Poliomyelitis ..... asymmetric weakness.
•9 \.Verdnig-Hoffmann disease ..... symmetric weakness.
• 10 Amyotrophic lateral sclerosis Combined UM l and L I deficits with no sensory
or bowel/bladder defi cits (due to loss of cortical and
· 11
• 12
• 13
•• • • spinaI cord motor neurons, respectively).
Can be caused by defect in superoxide dismutase l.
Commonly presents with asymmetric limb weakness
• 14 (hands/feet), fase iculations, eventual atrophy. Fatal.
• 15 Commonly known as Lou Gehrig disease.
Treatment: riluzole .
• 16
Posterior Sjlinal arteries Complete occlusion of anterior Spares dorsal columns and Lissauer tract; mid-
• 17 ~
spinal artery thoracic ASA territory is watershed area, as artery
• 18 of Adamkiewicz supplies ASA below- T 8. Presents
• 19 with UM I deficit below the lesion (corticospinal
• 20 tract), LM N deficit at the level of the lesion (anterior
horn), and loss of pain and temperature sensation
• 21 •
• below the lesion (sninothalamic lractt
6
lock
s
Suspend
0
End Block
Item: 4 of 25 ~ 1 • M k -<:J 1>- Jil ~· !:';-~
QIO: 4423 ..L ar Pre v ious Next Lab fli!ltues Not es Calcula t o r

1
& . &

2
Complete occlusion of anterior Spares dorsal columns and Lissauer tract; mid-
spinal artery thoracic AS territory is watershed area, as artery
3
of Adamkiewicz supplies ASA below- T8. Presents
4 with U l 1 deficit below the lesion (corticospinal
•5 tract), L~ ll\ deficit at the le,·el of the lesion (anterior
•6 horn), and loss of pain and temperature sensation
below the lesion (spinothalamic tract).
.7
·8 Tabes dorsalis Caused b} 3° S} phil is. Results from degeneration
.9 (demyelination) of dorsal columns and roots
_. progressive sensory ataxia (impaired
• 10
proprioception -.. poor coordination).
• 11 Associated'' ith Charcot joints, shooting pain, rg) II
• 12 Robertson pupils.
Exam will demonstrate absence of DTRs and
• 13
<±> Romberg sign.
• 14
Syringomyelia Syrinx expands and damages anterior white
• 15
commissure of spinothalamic tract (2nd-order
• 16
• 17
. 18
• neurons) -.. bilatera l loss of pain and temperature
sensation in cape-like distribution; seen with Chiari
l malformation; can expand and affect other tracts.
• 19
, Vitamin B12 deficiency Subacute combined degeneration (SCD)-
demyelination of Spinocerebellar tracts, lateral

'
• 20
Corticospinal tracts, and Dorsal columns. taxie
• 21 gait, paresthesia, impaired position/vibration sense.

a
Lock
s
Suspend
8
End Bl ock
Item: 4 of 25 ~ 1 • M k -<:J 1>- Jil ~· !:';-~
QIO: 4423 ..L ar Pre v ious Next Lab fli!ltues Not es Calcula t o r

1 • (demyelination) of dorsal columns and roots •

2
__. progressive sensory ataxia (impaired
proprioception -... poor coordination).
3
Associated" ith Charcot joints, shooting pain, Arg) II
4 Robertson pupils.
•5 Exam "ill demonstrate absence of DTRs and
•6 EEl Romberg sign .
•7 Syringomyelia yrinx expands and damages anterior white
commissure of spinothalamic tract (2nd-order
·8
neurons) __. bilateral loss of pain and temperature
.9
• 10
• sensation in cape-like distribution; seen '' ith Chiari
I malformation; can expand and affect other tn1cl~.
• 11
• 12
, Vitamin 812 deficiency Subacute combined degeneration (SCD)-
dcmyclination of Spinocerebellar tracts, lateral
• 13
• 14
• 15
' Cauda equina syndrome
Corticospinal tracts, and Dorsal columns. Ataxic
gait, paresthesia, impaired position/vibration sense.

Unilatera l symptoms including radicular pain, absent·


• 16 knee and ankle reflex, loss of bladder and nn:1l
• 17 sphincter control. Can cause saddle anesthesia .
Treatment: emergent surgery and steroids.
• 18
Due to compression of spinal roots from L2 and
• 19 below, often caused by intra,·ertebral disk herniation
• 20 or tumors.
• 21

a
Lock
s
Suspend
8
End Bl ock
Item: 5 of 25 ~ 1 • M k -<:J 1>- Jil ~· !:';-~
QIO: 2947 ..L ar Pre v ious Next Lab fli!ltues Not es Calcula t o r

IAA]
& &
1
A medical student in histology class is studying the peripheral nervous system . Wh ile viewing a specimen
2
under high magnification, she not ices that the neurons have interruptions in the myelin sheath along the
3 length of their axons at regu lar intervals.
4
•5 Which of the following describes the primary function of these gaps?
•6
:
•7 A. Sites of insulation
·8 B. Sites of inward current
.9
C. Sites of low membrane capacitance
• 10
• 11 D. Sites of neurotransmitter release
• 12 E. Sites of phagocytosis
• 13
• 14
• 15
• 16
• 17
• 18
• 19
• 20
• 21

a
Lock
s
Suspend
8
End Bl ock
Item: 5 of 25 ~. I • M k <:] t> al ~· ~
QIO: 2947 .l. ar Previous Next lab 'lifllues Notes Calculator

1 •

2
The correct answer is B. 62°/o chose this.
3
Nodes of Ranvier are gaps in the myelin sheath that occur along mye linated axons. I n between the nodes, ionic
4
current is conducted via the axonal cytop lasm as resistance to current flow th rough cytoplasm is lower than
5 across the cell membrane. At each node, however, myelin is absent and the resistance is lower across the
•6 membrane. He re the cu rrent activates voltage-gated sod ium channels in the membrane, al lowing ions to flow
•7
into the cytoplasm and boost the propagating depolarizing signa l. I ncreased capacitance and decreased
resistance at the nodes of Ranvier alter the action potential and resu lt in this "jumping " of the action potential
•8 from one node to another, known as sa ltatory conduction.
•9 Action potential Saltatory conduction Node of Ranvier Myelin Cell membrane Cytoplasm Axon Sodium channel Sodium Depolarization Capacitance

• 10 Biological membrane Ionic bonding

· 11
A is not correct. 9°/o chose this .
• 12 The mye lin sheath, formed by Schwann cells in the pe riphera l nervous system and ol igodend rocytes in the
• 13 central nervous system, insulates the axon and increases action potentia l conduction velocity. The nodes of
• 14 Ranvier are gaps in the mye lin sheath, and therefore sites of little insu lation .
Action potential Node of Ranvier Axon Central nervous system Myelin Schwann cell Peripheral nervous system Oligodendrocyte Nervous system
• 15
Nerve conduction velocity Electrical resistivity and conductivity Insulator (electricity)
• 16
• 17 C is not correct. 23% chose this .
• 18 The nodes of Ranvier are sites of high membrane capacitance relative to the myelinated axon segments. The
higher membrane capacitance facilitates rapid action potential propagation .
• 19
Action potential Node of Ranvier Axon Capacitance Myelin Biological membrane Cell membrane Membrane
• 20
• 21
D is not correct. 6°/o chose this .
• f\ 1 - · ---.a..----- = ~.a..--- --- --•-----1 ,c _ _ _ .a..L-- - - - -· · - - - .... : - -----L----- _.... ""''-- - .. ·-- .... ___ : __ • .,..._ ___ -'---- : -- •- .... ._ __

6
lock
s
Suspend
0
End Block
Item: 5 of 25 ~. I • M k <:] t> al ~· ~
QIO: 2947 .l. ar Previous Next lab 'lifllues Notes Calculator

1 • from one node to another, known as sa ltatory conduction .


2 Action potential Saltatory conduction Node of Ranvier Myelin Cell membrane Cytoplasm Axon Sodium channel Sodium Depolarization Capacitance

3 Biological membrane Ionic bonding

4 A is not correct. 9°/o chose this.


5 The mye lin sheath, formed by Schwann cells in the pe riphera l nervous system and ol igodend rocytes in the
•6 central nervous system, insulates the axon and increases action potentia l conduction velocity. The nodes of
•7
Ranvier are gaps in the mye lin sheath, and therefore sites of little insu lation .
Action potential Node of Ranvier Axon Central nervous system Myelin Schwann cell Peripheral nervous system Oligodendrocyte Nervous system
•8
Nerve conduction velocity Electrical resistivity and conductivity Insulator (electricity)
•9
• 10 C is not correct. 23% chose this .
· 11
The nodes of Ranvier are sites of high membrane capacitance relative to the myelinated axon segments. The
higher membrane capacitance facilitates rapid action potential propagation .
• 12 Action potential Node of Ranvier Axon Capacitance Myelin Biological membrane Cell membrane Membrane
• 13
D is not correct. 6°/o chose this .
• 14
Neu rotransmitters are released f rom the presynaptic membrane at the axon termina l. These chemica ls then
• 15
diffuse across the synaptic cleft to activate receptors in the dendrites of postsynaptic neu rons, transmitting
• 16 messages th rough nerve networks .
• 17 Axon terminal Axon Synaptic cleft Chemical synapse Neurotransmitter Synapse Dendrite Active zone Neuron Cell membrane Biological membrane

• 18 E is not correct. 0°/o chose this .


• 19 Microg lia l cel ls carry out phagocytosis in the central ne rvous system . Elsewhe re this function is the task of
• 20 tissue macrophages .
Phagocytosis Central nervous system Macrophage Nervous system Microglia
• 21

6
lock
s
Suspend
0
End Block
Item: 5 of 25 ~. I • M k <:] t> al ~· ~
QIO: 2947

1
.l.
-_:.:---·-·
ar Previous Next

-:.:- -- -· --·- -·- ·-·


lab 'lifllues Notes Calculator

Action potential Node of Ranvier Axon Central nervous system Myelin Schwann cell Peripheral nervous system Oligodendrocyte Nervous system
2
Nerve conduction velocity Electrical resistivity and conductivity Insulator (electricity)
3
4 C is not correct. 23% chose this.
5
The nodes of Ranvier are sites of high membrane capacitance re lative to the myelinated axon segments. The
higher membrane capacitance facilitates rapid action potential propagation .
•6
Action potential Node of Ranvier Axon Capacitance Myelin Biological membrane Cell membrane Membrane
•7
D is not correct. 6°/o chose this .
•8
Neu rotransmitters are released f rom the presynaptic membrane at the axon termina l. These chemica ls then
•9
diffuse across the synaptic cleft to activate receptors in the dendrites of postsynaptic neu rons, transmitting
• 10 messages th rough nerve networks .
· 11 Axon terminal Axon Synaptic cleft Chemical synapse Neurotransmitter Synapse Dendrite Active zone Neuron Cell membrane Biological membrane

• 12 E is not correct. 0°/o chose this .


• 13 Microg lia l cel ls carry out phagocytosis in the central ne rvous system . Elsewhe re this function is the task of
• 14 tissue macrophages .
Phagocytosis Central nervous system Macrophage Nervous system Microglia
• 15
• 16
• 17 Bottom Line:
• 18 Nodes of Ranvier are the gaps in myelination along the axon . They are sites of inward current propagation via
• 19 voltage-gated sodium channel activation .
Voltage-gated sodium channel Node of Ranvier Axon Sodium channel Myelin Sodium
• 20
• 21

6
lock
s
Suspend
0
End Block
Item: 5 of 25 ~ 1 • M k -<:J 1>- Jil ~· !:';-~
QIO: 2947 ..L ar Pre v ious Next Lab fli!ltues Not es Calcula t o r
& &
1

2 lal;tii!11•J for year:


FIRST AID FAC TS
2017 •
3
4
FA17 p 464.3
5
Oligodendrocytes ~ lrel i nates axons of neurons in CNS. Each Ocri,·cd from neuroectode rm.
•6 "Fried egg" appearance histologically.
Node of Ranvier oligodendrocyte can myelinate many axons
.7 (- 30). Predominant typ e of glial cell in white Injured in multiple scle rosis, progressive
·8 matte r. multifocalle ukoencephalopathy ( P.\ IL),
.9 leukodystrophies.

• 10 l-
Obgodendrocyte
• 11

• 12
• 13
• 14 FA17 p464.2

• 15 Schwann cells Each Schwann cell myclinates only l P S axon. May be in jured in Guilla in-Barre syndrome.
• 16 Also promote axonal regene ra tion. Deri ved
Nucleus { Schwann cell
• 17 1 from neural crest.

. 18
• 19
Myelin sheath} Node of Ranvier
• 20 It]

• 21

a
Lock
s
Suspend
8
End Bl ock
Item: 6 of 25 ~ 1 • M k -<:J 1>- Jil ~· !:';-~
QIO: 4426 ..L ar Pre v ious Next Lab fli!ltues Notes Calcula t o r

IAA]
& &
1
A loss of function mutation affecting a particula r tumor suppressor gene causes the bilatera l lesions shown 1n
2
t he image .
3
4
5
•6
•7

·8
.9
• 10
• 11
• 12
• 13
• 14
• 15
• 16
• 17
• 18
• 19
• 20
• 21

a
Lock
s
Suspend
8
End Bl ock
• 11
• 12
On wh ich chromosome is this tumor suppressor gene found?
• 13
:
• 14
A. Sq
• 15
• 16
B. ll q
• 17 c. 13q
• 18 D. 17p
• 19
E. 22q
• 20
• 21

a
Lock
s
Suspend
8
End Block
Item: 6 of 25 ~. I • M k <:] t> al ~· ~
QIO: 4426 .l. ar Previous Next lab 'lifllues Notes Calculator

1 •

2
The correct answer is E. 49°/o chose this.
3 Th is image shows bilateral acoustic neuromas (also known as vest ibu lar schwannomas) . Bilatera l lesions of this
4 sort can on ly be caused by neurofibromat osis t ype 2 . These acoust ic neuromas man ifest with hearing loss,
5 headache, t innitus, or vertigo. The tumor suppressor gene t hat is lost in t his disorder is on chromosome 22q .
Tinnitus Neurofibromatosis Tumor suppressor gene Vertigo Gene Vestibular schwannoma Headache Hearing loss Neoplasm Vestibular system Schwannoma
6
Chromosome Chromosome 22 (human)
•7
•8 A is not correct. 9°/o chose this .
•9 The loss of the APC t umor suppressor gene on chromosome Sq causes some cases of colorect al cancer .
Tumor suppressor gene Colorectal cancer Gene Neoplasm Cancer Adenomatous polyposis coli Chromosome Chromosome s (human)
• 10

· 11 B is not correct. 12% chose this.


• 12 The loss of the WT1 tumor suppressor gene on chromosome l lq can resu lt in Wi lms tumor .
WTl Tumor suppressor gene Gene Wilms' tumor Neoplasm Chromosome
• 13
• 14 C is not correct. 12% chose this .
• 15 The loss of the Rb tumor suppressor gene on chromosome 13q can result in retinoblast oma and ost eosarcoma .
Osteosarcoma Tumor suppressor gene Retinoblastoma Gene Retinoblastoma protein Neoplasm Chromosome
• 16
• 17 D is not correct. 18% c hose this .
• 18 The loss of the p53 tumor suppressor gene on chromosome 17p is a mutation that can be found in many
cancers as well as Li-Fraumen i syndrome .
• 19
li-Fraumeni syndrome Tumor suppressor gene Gene PS3 Chromosome Mutation Neoplasm Chromosome 17 (human)
• 20
• 21

6
lock
s
Suspend
0
End Block
Item: 6 of 25 ~. I • M k <:] t> al ~· ~
QIO: 4426 .l. ar Previous Next lab 'lifllues Notes Calculator

1 Chromosome Chromosome 22 (human)

2 A is not correct. 9°/o chose this.


3 The loss of the APC tumor suppressor gene on chromosome Sq causes some cases of colorectal cancer.
4 Tumor suppressor gene Colorectal cancer Gene Neoplasm Cancer Adenomatous polyposis coli Chromosome Chromosome s (human)
5 B is not correct. 12% chose this.
6 The loss of the WT1 tumor suppressor gene on chromosome l lq can result in Wi lms tumor.
.7 WTl Tumor suppressor gene Gene Wilms' tumor Neoplasm Chromosome

•8 C is not correct. 12% chose this.


•9 The loss of the Rb tumor suppressor gene on chromosome 13q can result in retinoblastoma and osteosarcoma .
• 10 Osteosarcoma Tumor suppressor gene Retinoblastoma Gene Retinoblastoma protein Neoplasm Chromosome

· 11 D is not correct. 18% chose this.


• 12 The loss of the p53 tumor suppressor gene on chromosome 17p is a mutation that can be found in many
• 13 cancers as well as Li-Fraumen i syndrome .
li-Fraumeni syndrome Tumor suppressor gene Gene PS3 Chromosome Mutation Neoplasm Chromosome 17 (human)
• 14
• 15
• 16 Bottom Line:
• 17
Bi lateral acoustic neuromas are associated with neurofibromatosis type 2, wh ich resu lts from loss of function of
• 18 the NF2 gene on chromosome 22q .
Neurofibromatosis type II Neurofibromatosis Gene Vestibular schwannoma Chromosome 22 (human) Merlin (protein) Chromosome
• 19
• 20
• 21
... i!!ii.IJ.S'.ii.i _ ,. I

6
lock
s
Suspend
0
End Block
Item: 6 of 25 ~ 1 • M ar k -<:J I> ~ £!1}>'
• !!":-~
QIO: 4426 ..L Previous Next Lab lues Notes Cal culator

& &
1
FA17 p464.2
2

3 Schwann cells Each Sch" ·ann cellmyelinates onlr I P 1$ axon. lay be injured in Guillain-Barre syndrome.
Iso promote axonal rcgcncrat ion. Derived
4
from neural crest.
5
6
.7 Myelm sheath } Node of RarMer
~
·8
.9
FA17p496.1
• 10
Adult primary brain tumors
• 11
TUMOR DESCRIPTION HISTOLOGY
• 12
Glioblastoma Common, highly malignant 1° brain tumor with Astrocyte origin, GFAP ®. "Pseudopalisading"
• 13 multiforme (grade IV - 1-year median survival. Fou nd in cerebral rn
pleomorphic tumor cells border central
• 14 astrocytoma) hemispheres fJ. Can cross corpus callosum areas of necrosis and hemorrhage.
• 15 ("butterfly glioma").
• 16 Oligodendroglioma Relatively rare, slow growi ng. Most often in Oligodendrocyte origin. "Fried egg" cells-
• 17 frontal lobes B!. "Chicken-wire" capi llary round nuclei with clear cytoplasm [!]. Often
pattern . calcified.
. 18
• 19 Meningioma Common, typically benign 1° brain tumor. Arachnoid cell origin. Spindle cells
t1ost often occurs near surfaces of brain and concentrically arranged in a whorled
• 20
in parasagittal region. Extra-axial (e,ternal pattern; psammoma bodies 0 (laminated
• 21
• to brain oarenclwma) and mav have a dural ca lei fi ca tions).
a
Lock
s
Suspend
8
End Block
Item: 6 of 25 ~ 1 • M ar k -<:J I> ~ £!1}>'
• !!":-~
QIO: 4426 ..L Previous Next Lab lues Notes Cal culator

& &
1
FA17 p 4 96.1
2
Adult primary brain tumors
3
TUMOR OESCRIPTION HISTOlOGY
4
Glioblastoma Common, highly malignant 1° brain tumor with Astrocyte origin, CFAP ®. UPseudopalisading"
5 multiforme (grade IV - 1-year median survival. Found in cerebral rn
pleomorphic tumor cells border central
6 astrocytoma) hemispheres rJ. Can cross corpus callosum areas of necrosis and hemorrhage.
.7 ("butterAy glioma").
·8 Oligodendroglioma Relatively rare, slow gro\\ ing. ~ l o~t often in Oligodendrocyte origin. "Fried egg" cells-
.9 frontal lobes [!t. '·Chicken-wire" capillary round nuclei with clear cytoplasm [!). Often
• 10
pattern. calcified.
• 11 Meningioma Common, typically benign 1° brain tumor. Arachnoid cell origin. Spindle cells
• 12
Most often occurs near surfaces of brain and concentrically arranged in a whorled
in parasagittal region. Extra-axial (e,ternal pattern; psammoma bodies D (laminated
• 13
to brain parenchyma) and may have a dural calcifications).
• 14 attachment {"ta il" 0). OFten asymptomatic;
• 15 may present with seizures or foe<ll neurologic
• 16 signs. Resection and/or radiosurgery.
• 17 Hemangioblastoma lost often cerebellar m.
Associated with \On Blood vessel origin. Closely arranged, thin-
. 18 Hippei-Lindau syndrome when found with walled capil laries with minimal intervening
• 19
retinal angiomas. Can produce erythropoietin parenchyma C).
• 20
.... zopolycythemia .
Pituitary adenoma Adenoma may be nonfunctioning or llyperplasia of only one type of endocrine cells
• 21
• J.H"'Mt.Pr-f.u,,..+inninh \ fn~f ,...,.u-.·u,,nnlu fntu'u·J ;n n;t.,a-3n_, / i,. l-111"fntrnnh nnn"lrlntrn•"''.,

a
Lock Suspend
s 8
End Block
Item: 6 of 25 ~ 1 • M k -<:J 1>- Jil ~· !:';-~
QIO: 4426 ..L ar Pre v ious Next Lab fli!ltues Notes Calcula t o r

1
& po) ) &

2 Pituitary adenoma Adenoma may be non functioning or Hyperplasia of only one type of endocrine cells
hyperfunctioning. ::vtost commonly found in pituitary (ie, lactotroph, gonadotroph,
3
from lactotrophs (prolactinoma) somatotroph, corticotroph).
4
..... hyperprolactinemia; less commonly
5 adenoma of somatotrophs (G il) ..... acromegaly/
6 gigantism; corticotrophs (ACTH) ..... Cushing's
0 7 disease. Rarely, adenoma of th) rotrophs (T II )
o8
and gonadotroph (FSI I, Lll). Bitemporal
hemianopia due to pressure on optic chiasm
.9
( shows normal \'iSual field above, patient's
• 10 perspecti,·c below}. Sequelae include hyper-
• 11 or hypopituitarism, which may be caused by
• 12 pituitary apoplexy.
• 13 Schwan noma Classically at the cerebellopontinc angle 13, Schwann cell origin D, S-100 $ .
0 14 but can be along any peripheral nerve. Often
• 15
loe<llizecl to Cr VII I in internal acoust ic
meatus -7 vestibular schwannoma. Bilateral
0
16
vestibular schwannomas found in lF-2.
0
17 Resection or stereotactic rad iosurgcry.
• 18
• 19
• 20
• 21

a
Lock
s
Suspend
8
End Bl ock
Item: 6 of 25 ~ 1 • M ar k -<:J I> ~ £!1}>'
• !!":-~
QIO: 4426 ..L Previous Next Lab lues Notes Cal culator

& &
1
2
3
4
5
6
•7

·8
.9
• 10
• 11
• 12
• 13
• 14
• 15
• 16
• 17
• 18
• 19
• 20
• 21

a
Lock
s
Suspend
8
End Block
Item: 6 of 25 ~ 1 • M k -<:J 1>- Jil ~· !:';-~
QIO: 4426 ..L ar Pre v ious Next Lab fli!ltues Notes Calcula t o r
& &
1

2 FA17 p218.2
3 Tumor suppressor Loss of fun ction ... t cancer risk; both (h, o) alleles of a tu mor suppressor gene must be lost for
4 genes expression of disease.
5 GENE GENEPROOUCT ASSOCIATED CONDITION

6 APC 'egative regulator of ~-catenin/\V I T patlm ar Colorectal cancer (associated with FAP)
0 7 BRCA1/ BRCA2 01'\A repair protein Breast, o,·arian, and pancreatic cancer
o8 CDKN2A pl6. blocks C 1 ... S phase r-.. Jelanoma, pancreatic cancer
.9 DCC D CC - D eleted in C olon C ancer Colon cancer
• 10 DPC4/SMAD4 DPC - D eleted in Pancreatic C ancer Pancreatic cancer
• 11 MEN1 !\lenin r-.. rE ' I
• 12 NF1 1\eurofibromatosis type 1
eurofibromin (Ras GTPase activating protein)
• 13
NF2 Merlin (schwannomin) protein Neurofibromatosis type 2
0 14
PTEN Tyrosine phosphatase ofPJP3 (eg, protein kinase Breast cancer, prostate cancer, endometrial
• 15 B (AKTJ activation) cancer
0
16 Rb Reti noblastoma, osteosa rcoma
Inhibits E2F; blocks C 1 ... S phase
0
17
TPS3 p53, acti,·ates p21, blocks C 1 ... S phase Most human cancers, Li-Fraumeni syndrome
. 18 (multiple malignancies at early age, aka, SOL.\
• 19 cancer syndrome: Sarcoma, Breast, Leukemia,
• 20 \ drenal gland)
• 21 TSC1 Hamartin protein Tuberous sclerosis

a
Lock
s
Suspend
8
End Bl ock
Item: 6 of 25 ~ 1 • M k -<:J 1>- Jil ~· !:';-~
QIO: 4426 ..L ar Pre v ious Next Lab fli!ltues Notes Calcula t o r

1 egat e g pa ) Co o ec a ca ce (ass )
2 BRCA 1/ BRCA2 0 lA repair protein Breast, ovarian, and pancreatic cancer
3 CDKN2A pl6, blocks C 1 .... S phase ~ l ela noma, pancreatic cancer
4 DCC D CC- Deleted in C olon C ancer Colon cancer
5 DPC4/SMAD4 DPC - O eleted in Pancreatic C ancer Pancreatic cancer
6 MEN1 lenin ~ I E~ I
7
NF1 eurolibromin (Ras C T Pase activating protein)
0

i\eurofibromatosis type 1
o8
NF2 lerlin (schwannomin) protein r'-.eurofibromatosis type 2
.9
PTEN Tyrosine phosphatase of PIP 3 (eg, protein kinase Breast cancer, prostate cancer, endometrial
• 10
B [AKTJ activation) cancer
0
11
Rb Inhibits E2F; blocks C 1 .... S phase Retinoblastoma, osteosarcoma
0
12
TPS3 p53, acti,·atcs p21, blocks C 1 .... S phase Most human cancers, Li-F'raumeni syndrome
0
13 (multiple malignancies at early age, aka, SDL.\
0 14 cancer syndrome: Sarcoma, Breast, Leukemia,
• 15 Ad renal gland)
0
16 TSC1 Hamartin protein Tuberous sclerosis
0
17 TSC2 Tuberin protein Tuberous sclerosis
0
18 VHL Inhibits hypoxia inducible factor 1a \ Oil H ippei-Lindau disease
0 19 WT1 Transcription factor that regula tes urogenital W ilms tumor (nephroblastoma)
• 20 development
0
21

a
Lock
s
Suspend
8
End Bl ock
Item: 7 of 25 ~ 1 • M k -<:J 1>- Jil ~· !:';-~
QIO: 2931 ..L ar Pre v ious Next Labfli!llues Notes Calcula t o r
& &
1
A previously healthy 54-year-old man presents w ith a 3-year history of progressive, symmetric muscle
2
weakness that has confined him to a wheelchair. Over the past week, the patient has experienced slurring of
3 his speech and difficulty swal lowing. He denies sensory deficits, myalgias, fever, or arthralgias, and his
4 mental status is intact. Physical examination reveals muscle atrophy, tongue fasciculations, and extensor plantar
responses.
5
6
Which of the following histologic findings is most typical of this patient's underlying disease?
0 7
o8 :
A . Anterior horn and lateral column degeneration with gliosis
.9
• 10 B. Demyeli nation of dorsal columns and spinocerebellar tracts
• 11 C. Deposits of amyloid 13 protein
• 12
D. Irregularly scattered parenchymal and spinal cord lesions
• 13
0 14
E. Whorled pattern of concentrically arranged spindle cells within the meninges
• 15
0
16
0
17
• 18
• 19
• 20
• 21

a
Lock
s
Suspend
8
End Bl ock
Item: 7 of 25 ~. I • M k <:] t> al ~· ~
QIO: 2931 .l. ar Previous Next Lab 'lifllues Notes Calculator

1 •
A previously healthy 54-year-o ld man presents with a 3-year history of progressive, symmetric muscle
2
weakness that has confined him to a wheelchair. Over the past week, the patient has experienced slurring of
3 his speech and difficu lty swal lowing . He denies sensory deficits, myalgias, fever, or arth ralgias, and his
4 mental status is intact. Physical examination reveals muscle atrophy, tongue fasciculations, and extensor plantar
responses.
5
6
Wh ich of the fo llowing histologic f indings is most typical of this patient's underlying disease'
7
•8 :
• A. Anterior horn and lateral co lumn degeneration with gl iosis
•9
B. Demyel ination of dorsal columns and spinocerebellar tracts
Correct!
• 10
Amyotrophic lateral
· 11 C. Deposits of amyloid ~ protein sclerosis (ALS), also
• 12
D. Irregularly scattered parenchyma l and spinal cord lesions known as Lou Gehrig 's
• 13 disease, usua lly begins in
E. Whor led pattern of concentrica lly arranged spind le cells within the men inges midd le age and progresses
• 14
to death over a per iod of a
• 15 few years . ALS is
• 16 character ized by loss of upper and lower motor neurons, with sparing of sensory neurons. The loss of anterior
• 17 horn cel ls results in progressive weakness that proceeds from neurogenic muscu lar atrophy to paralysis. In this
scenario, the patient deve loped lower motor neuron signs of wea kness, atrophy, and fascicu lation. The patient
• 18
also has a positive Babinski reflex (extensor plantar response), which is indicative of upper motor neuron lesions.
• 19 Upper motor lesions are caused by damage to the lateral corticospinal t ract. A lack of sensory deficits, mya lgias,
• 20 fever, or arth ralgias is also consistent with ALS. Patients usual ly retain mental function over the course of the
disease . Death is usua lly caused by paralysis of respi ratory muscles .
• 21

6
lock
s
Suspend
0
End Block
Item: 7 of 25 ~. I • M k <:] t> al ~· ~
QIO: 2931 .l. ar Previous Next lab 'lifllues Notes Calculator

1 •

2 The correct answer is A. 72°/o chose this.


3 Amyotrophic lateral sclerosis (ALS), also known as Lou Gehrig's disease, usual ly begins in midd le age and
4 progresses to death over a per iod of a few years. ALS is character ized by loss of upper and lower motor
neurons, with sparing of sensory neurons. The loss of anterior horn ce lls results in prog ressive weakness that
5
proceeds from neurogen ic muscula r atrophy to paralysis. In th is scenario, the patient developed lower motor
6 neuron signs of weakness, atrophy, and fasciculation. The patient also has a posit ive Babinski reflex (extensor
7 planta r response), which is ind icative of upper motor neu ron lesions. Upper motor lesions are caused by
•8
damage to the lateral corticospinal t ract. A lack of sensory deficits, mya lgias, fever, or arthralgias is also
consistent with ALS. Patients usual ly retain mental function over the course of the disease. Death is usual ly
•9
caused by paralysis of respiratory muscles .
• 10 Plantar reflex Fasciculation Amyotrophic lateral sclerosis lower motor neuron Pyramidal tracts Upper motor neuron Neuron lateral corticospinal tract

· 11 Motor neuron lou Gehrig Primitive reflexes Nervous system Anterior grey column Atrophy Sensory neuron Paralysis Muscle atrophy Horn (anatomy)

• 12 Anatomical terms of motion

• 13
B is not correct. 13% chose this .
• 14
Demyel ination of dorsal columns and spinocerebellar tracts is found in vitamin B 12 neuropathy and Friedrich 's
• 15 ataxia. The lateral corticospinal t racts can also be aff ected . Because the dorsal co lumn-medial lemn iscus
• 16 pathway relays fine touch and proprioception to the cortex, patients present with impai red position and
vibration sense. The lateral corticospinal tracts carry descending motor tracts from the contralatera l cerebral
• 17
hemisphere .
• 18 Cerebral hemisphere Proprioception Demyelinating disease Friedreich's ataxia Spinocerebellar tract Ataxia Vitamin B12 Posterior column
• 19 Anatomical terms of location Peripheral neuropathy Somatosensory system Contralateral Vitamin Pyramidal tracts Cortex (anatomy)
• 20
C is not correct. 4°/o chose this .
• 21

6
lock
s
Suspend
0
End Block
Item: 7 of 25 ~. I • M k <:] t> al ~· ~
QIO: 2931 .l. ar Previous Next Lab 'lifllues Notes Calculator
-- -
1 •
Cerebral hemisphere Proprioception Demyelinating disease Friedreich' s ataxia Spinocerebellar tract Ataxia Vitamin B12 Posterior column
2 Anatomical terms of location Peripheral neuropathy Somatosensory system Contralateral Vitamin Pyramidal tracts Cortex (anatomy)
3
C is not correct. 4°/o chose this.
4
Alzheimer's disease is characterized by extracellular deposition of amyloid ~ protein, common ly known as sen ile
5
or neuritic plaques. These plaques are most numerous in the cerebral cortex and hippocampus . The other
6 classic histologic finding in Alzheimer's disease is neurofibrillary tang les (hyper phosphorylated T proteins) within
7 the neu rons. Patients develop dementia marked mainly by progressive memory loss.
Alzheimer' s disease Hippocampus Cerebral cortex Dementia Amyloid Neurofibrillary tangle Protein Senile plaques Extracellular Histology Neuron Amnesia
•8
Phosphorylation Hyperphosphorylation
•9
• 10 D is not correct. 8°/o chose this .
· 11 This answer choice is consistent with mu ltip le sclerosis (MS) . Such histopathologic findings typical ly are not seen
in patients with ALS. MS is a multifoca l disease, and the lesions in the brain and spina l cord are separated in
• 12
location and t ime . The clinical find ings thus can be quite var ied . Although some symptoms may overlap with
• 13 those of ALS, MS usually is characterized by exacerbations with long asymptomatic remissions, eventua lly
• 14 leading to physical impairment with mental deterioration .
Multiple sclerosis Spinal cord Histopathology Asymptomatic Amyotrophic lateral sclerosis Human brain Brain
• 15
• 16 E is not correct. 3°/o chose this .
• 17 A whorled pattern of concentr ically arranged spind le cel ls describes the classic microscopic appearance of
• 18 psammoma bod ies in men ing ioma. These neop lasms are slow growing and resectable but may reach a la rge
size before symptoms (eg, headaches, blurry vision) lead to detection .
• 19
Psammoma body Meningioma Neoplasm
• 20
• 21
..
6
lock
s
Suspend
0
End Block
Item: 7 of 25 ~. I • M k <:] t> al ~· ~
QIO: 2931 .l. ar Previous Next Lab 'lifllues Notes Calculator

1 •
or neuritic plaques. These plaques are most numerous in the cerebral cortex and hippocampus . The other
2 classic histologic finding in Alzheimer's disease is neurofibrillary tang les (hyper phosphorylated T proteins) within
3
the neu rons. Patients develop dementia marked mainly by progressive memory loss.
Alzheimer' s disease Hippocampus Cerebral cortex Dementia Amyloid Neurofibrillary tangle Protein Senile plaques Extracellular Histology Neuron Amnesia
4
Phosphorylation Hyperphosphorylation
5
6 D is not correct. 8°/o chose this.
7 This answer choice is consistent with mu ltip le sclerosis (MS) . Such histopathologic findings typical ly are not seen
in patients with ALS. MS is a multifoca l disease, and the lesions in the brain and spina l cord are separated in
•8
location and t ime . The clinical find ings thus can be quite var ied . Although some symptoms may overlap with
•9 those of ALS, MS usually is characterized by exacerbations with long asymptomatic remissions, eventua lly
• 10 leading to physical impairment with mental deterioration .
Multiple sclerosis Spinal cord Histopathology Asymptomatic Amyotrophic lateral sclerosis Human brain Brain
· 11
• 12 E is not correct. 3°/o chose this .
• 13 A whorled pattern of concentr ically arranged spind le cel ls describes the classic microscopic appearance of
• 14
psammoma bod ies in men ing ioma. These neop lasms are slow growing and resectable but may reach a la rge
size before symptoms (eg, headaches, blurry vision) lead to detection .
• 15 Psammoma body Meningioma Neoplasm
• 16
• 17
Bottom Line:
• 18
• 19 ALS is characterized by loss of upper (corticospinal tract) and lower (anterior horn) motor neurons, with
sparing of sensory neurons .
• 20 Pyramidal tracts Motor neuron Anterior grey column Sensory neuron Amyotrophic lateral sclerosis Neuron Horn (anatomy)
• 21

6
lock
s
Suspend
0
End Block
Item: 7 of 25 ~ 1 • M k -<:J 1>- Jil ~· !:';-~
QIO: 2931 ..L ar Pre v ious Next Labfli!llues Notes Calcula t o r

• FIRST AID FAC T S


. -. &
1

2
FA17 p 500.1
3
Spinal cord lesions
4
AREAAFFECT£0 DISEASE CHARACTERISTICS
5
Poliomyelitis and Werdnig-Hoffmann Congenital degeneration of anterior horns of spinal
6 disease cord. L I ' lesions only. ''Floppy baby" with marked
h~ poton ia and tongue fasciculations. Infantile type
7
·8 •• has median age of death of 7 months. Autosomal
recessive inheritance.
.9
Poliomyelitis - asrmmetric weakness.
• 10 \Verdnig-Hoffmann disease - symmetric weakness.
• 11 Amyotrophic lateral sclerosis Combined M N and Li\ It deficits with no sensor'•

•• • •
• 12 or bowel/bladder deficits (due to loss of cortical and
• 13 spina I cord motor neurons, respectively).
Can be caused by defect in superoxide dismutase I.
• 14
Commonly presents with asymmetric limb weakness
• 15 (hands/feet), fasciculations, eventual atrophy. F'atal.
• 16 Commonly known as Lou Gehrig disease.
• 17
Treatment: riluzole .
. 18 Complete occlusion of anterior Spares dorsal columns and Lissauer tract; mid-
spinal artery thoracic S territory is watershed area, as artery
• 19
of Adamkiewicz supplies ASA below- T8. Presents
• 20 with UM ' deficit below the lesion (corticospinal
tract), u.l deficit at the le,·el of the lesion (anterior
• 21
• •• ..
a
Lock
s
Suspend
8
End Bl ock
Item: 7 of 25 ~ 1 • M k -<:J 1>- Jil ~· !:';-~
QIO: 2931 ..L ar Pre v ious Next Labfli!llues Notes Calcula t o r
& &
1
Complete occlusion of anterior Spares dorsal columns and Lissauer tract; mid-
2
spinal artery thoracic AS territory is watershed area, as artery
3 of Adamkiewicz supplies ASA below- T8. Presents
4 with UM 1 deficit below the lesion (corticospinal
5
tract), Lf\ I deficit at the Je,·el of the lesion (anterior
horn), and loss of pain and temperature sensation
6 below the lesion (spinothalamic tract).
7
Tabes dorsalis Caused b} 3° S} phil is. Results from degeneration
·8
(demyelination) of dorsal columns and roots
.9 _. progressive sensory ataxia (impaired
• 10 proprioception -.. poor coordination).
• 11 Associated'' ith Charcot joints, shooting pain, Arg} II
Robertson pupils.
• 12
Exam will demonstrate absence of OTRs and
• 13 (f) Romberg sign.
• 14
Syringomyelia Syrinx expands and damages anterior white
• 15 commissure of spinothalamic tract (2nd-order
• 16
• 17 • neurons) - bilatera l loss of pa in and temperature
sensation in cape-like distribution; seen with Chiari
l malformation; can expand and affect other tracts.
. 18
Vitamin B12 deficiency Subacute combined degeneration (SCD)-
• 19
demyelinalion of Spinocerebellar tracts, lateral
I
• 20
• 21
• ' a
Corticospinal tracts, and Dorsal columns. Ataxic
gait, paresthesia, impaired position/vibration sense.

s 8
Lock Suspend End Bl ock
Item: 7 of 25 ~ 1 • M k -<:J 1>- Jil ~· !:';-~
QIO: 2931 ..L ar Pre v ious Next Labfli!llues Notes Calcula t o r

1 • (demyelination) of dorsal columns and roots •

2
__. progressive sensory ataxia (impaired
proprioception -... poor coordination).
3
Associated" ith Charcot joints, shooting pain, Arg) II
4 Robertson pupils.
5 Exam "ill demonstrate absence of DTRs and
6 EEl Romberg sign.
7 Syringomyelia yrinx expands and damages anterior white
commissure of spinothalamic tract (2nd-order
·8
neurons) __. bilateral loss of pain and temperature
.9
• 10
• sensation in cape-like distribution; seen '' ith Chiari
I malformation; can expand and affect other tn1cl~.
• 11
• 12
, Vitamin 812 deficiency Subacute combined degeneration (SCD)-
demyclination of Spinocerebellar tracts, lateral
• 13
• 14
• 15
' Cauda equina syndrome
Corticospinal tracts, and Dorsal columns. Ataxic
gait, paresthesia, impaired position/vibration sense.

Unilatera l symptoms including radicular pain, absent·


• 16 knee and ankle reflex, loss of bladder and nn:1l
• 17 sphincter control. Can cause saddle anesthesia .
Treatment: emergent surgery and steroids.
• 18
Due to compression of spinal roots from L2 and
• 19 below, often caused by intra,·ertebral disk herniation
• 20 or tumors.
• 21

a
Lock
s
Suspend
8
End Bl ock
Item: 8 of 25 ~ 1 • M k -<:J 1>- Jil ~· !:';-~
QIO: 3 0 85 ..L ar Pre v ious Next Lab fli!ltues Notes Calcula t o r
& &
1
An HIV-positive 43-year-old man presents with his caregiver to the physician. His caregiver reports the
2
patient is experiencing significant cognitive and behavioral decline. The caregiver drew blood and found a
3 CD4+ count of 50/ mm 3 • The careg iver also reports that he has been taking the patient's temperature, and
4 he has remained afebrile. Lumbar puncture is done, and resu lts are normal. A pathology sample of the patient's
disease is shown below.
5
6
7
·8
.9
• 10
• 11
• 12
• 13
• 14
• 15
• 16
• 17
• 18
• 19
• 20
• 21
• Imaae courtsev of Armed Forces Institute of Patholoav

a
Lock
s
Suspend
8
End Bl ock
Image courtsey of Armed Forces Institute of Pathology
• 11
• 12
What type of cel l has likely become infected in this patient's bra in?
• 13
:
• 14
A. Astrocytes
• 15
• 16
B. Kupffer cells

• 17 C. Langerhans cel ls
• 18 D. Microglia
• 19
E. Ol igodendrocytes
• 20
• 21

a
Lock
s
Suspend
8
End Block
Item: 8 of 25 ~. I • M k <:] t> al ~· ~
QIO: 3085 .l. ar Previous Next lab 'lifllues Notes Calculator

1 •

2
The correct answer is D. 56°/o chose this.
3
This patient has HIV encepha litis, wh ich is caused by infection of
4
macrophages and microgl ia in the brain. Microglia are macrophages of the
5 central nervous system that, like other tissue macrophages, ar ise from
6 monocytes in the bone marrow and migrate to their resident tissue site . It is
7
postu lated that HIV is carried into the brain by infected monocytes. Neurons
themselves are not infected by HIV, although they may be damaged by
8 inflammatory responses to viral products. The histologic hallma r k of HIV
•9 encephalopathy is the microgl ial nodu le, la rgely consisting of multinucleated
• 10 giant cells (indicated by the ar row in the vignette image), wh ich are formed
by the fusion of HIV-infected microglial cel ls (shown in this image) .
· 11
Microglia Central nervous system HIV Macrophage Monocyte Nodule (medicine) Encephalitis Bone marrow
• 12
HIV-associated neurocognitive disorder Human brain Nervous system Encephalopathy Histology Giant cell Image Copyright © 2007
• 13 Fendrick eta/; licensee BioMed
Neuron Brain Virus Bone Infection
• 14 Central Ltd.
• 15
A is not correct. 14% chose this .
• 16
Astrocytes, wh ich derive their name from their star-shaped appearance, are not part of the multinucleated giant
• 17
cells seen in HIV encephalopathy. Their long cytoplasmic processes surround neurons and capillaries and enable
• 18 them to fulfill many functions in the central nervous system, including serving as metabol ic buffers or detoxifiers
• 19 and suppliers of nutrients. They also contribute to the barrier function in the blood-brain barrier. Finally, they
• 20
are the principa l cell responsible for repair and sca r formation in the brain, known as gliosis. The mal ignant
transfor mation of astrocytes may result in glioblastoma multiforme, a common brain tumor that would lead to a
• 21 simila r cl inical presentation in an immunocompetent individual. This patient is immunosuppressed, however, as •

6
lock
s
Suspend
0
End Block
Item: 8 of 25 ~. I • M k <:] t> al ~· ~
QIO: 3085 .l. ar Previous Next lab 'lifllues Notes Calculator

1 •

2 A is not correct. 14% chose this.


3 Astrocytes, wh ich derive their name from their star-shaped appearance, are not part of the multinucleated giant
4 cells seen in HIV encephalopathy. Their long cytop lasmic processes surround neurons and capillaries and enable
them to fulfill many functions in the central nervous system, including serving as metabol ic buffers or detoxifiers
5
and suppliers of nutrients. They also contribute to the barrier function in the blood-brain barrier. Fina lly, they
6 are the principa l cell responsible for repair and scar formation in the brain, known as gliosis. The mal ignant
7 transformation of astrocytes may result in glioblastoma multiforme, a common brain tumor that would lead to a
simila r cl inical presentation in an immunocompetent individua l. This patient is immunosuppressed, however, as
8
demonstrated by his reduced CD4 count, and the cause of his cond it ion is thus more likely to be infectious .
•9 Glioblastoma multiforme Gliosis Central nervous system Blood-brain barrier Astrocyte CD4 Brain tumor HIV Capillary Immunosuppression Neuron
• 10 Nervous system Neoplasm Encephalopathy Cytoplasm Immunocompetence Giant cell Cancer Malignancy Human brain HIV-associated neurocognitive disorder
· 11 Metabolism Brain
• 12
B is not correct. 6°/o chose this .
• 13
Kupffer cel ls are the macrophages of the liver, not the CNS. They also arise from monocytes and make their
• 14
residence in the fenestrated endothelial- lined sinusoids surround ing the hepatocytes of the liver. There they
• 15 serve a phagocytic role, clearing the rich blood supply to the liver of foreign bodies .
• 16 Macrophage Monocyte liver Hepatocyte liver sinusoid Phagocytosis Phagocyte Sinusoid (blood vessel)

• 17 C is not correct. 6°/o chose this .


• 18 Langerhans ce lls are the macrophages of the skin, not the CNS . In the skin, they serve the macrophage role of
• 19 being an antigen-presenting cel l in wh ich they take up antigen and present it to lymphoid cells, initiating an
immune response to foreign bodies in the skin .
• 20
Macrophage Antigen-presenting cell Immune system Antigen lymphatic system
• 21

6
lock
s
Suspend
0
End Block
Item: 8 of 25 ~. I • M k <:] t> al ~· ~
QIO: 3085 .l. ar Previous Next lab 'lifllues Notes Calculator

1 •
Langerhans ce lls are the macrophages of the skin, not the CNS . In the skin, they serve the macrophage role of
2 being an antigen-presenting cel l in wh ich they take up antigen and present it to lymphoid cells, initiating an
3
immune response to foreign bodies in the skin.
Macrophage Antigen-presenting cell Immune system Antigen lymphatic system
4
5 E is not correct. 18% chose this.
6
HIV is not known to infect oligodendrocytes. However, JC v ir us is a human polyomavirus that infects
oligodend rocytes after it beco mes reactivated. JC virus is often acqu ired in childhood and remains latent within
7
most adu lts in t he kidneys and lymphoid organs. Howeve r, if the immune system becomes severely
8 co mpromised, such as in late stages of AIDS, JC virus can become reactivated . Reactivation of JC virus is known
•9 cl inica lly as prog ressive multifocal leukoencep halopathy (PML), wh ich leads to lysis of ol igodend rocytes. Since
oligodend rocytes are the myel in-producing ce lls within t he centra l nervous system, their destruction by JC virus
• 10
causes an appearance of small foci of demyelination within t he subcortica l wh ite matter. PML can man ifest
· 11 similarly to HIV encepha litis, however, focal neuro log ic deficits would be more likely than the cognitive decline
• 12 seen in this patient. Seizures, ataxia, aphasia, hemiparesis, or sensory deficits are more common
• 13 manifestations. t
Hemiparesis Progressive multifocal leukoencephalopathy JC virus Central nervous system Demyelinating disease Aphasia HIV White matter Ataxia Encephalitis
• 14
Oligodendrocyte Polyomaviridae HIV/AIDS Immune system leukoencephalopathy Nervous system Epileptic seizure lymphatic system Virus Kidney lysis
• 15
Dementia Neurology
• 16
• 17

• 18 Bottom Line:
• 19 HIV encepha litis is caused by infection of macrophages and microgl ia in the brain .
Microglia Encephalitis HIV Macrophage Brain Human brain Infection
• 20
• 21

6
lock
s
Suspend
0
End Block
Item: 8 of 25 ~. I • M k <:] t> al ~· ~
QIO: 3085 .l. ar Previous Next lab 'lifllues Notes Calculator

2 FA17 p463.4

3 Microglia Phagocytic scavenger cells of Cl'\S HTV-infected microglia fuse to form


4 (mesodermal, mononuclear origin). Activated multinucleated giant cells in C1 S.
5
in response to tissue damage. lot readily
discernible by I issl stain.
6
7

8
•9 FA11p114.1

• 10 Infections in immunodeficiency
PATHOGEN I HELLS I BCELLS I GRANULOCYTES I COMPLEMENT
· 11
• 12
Bacteria Sepsis Encapsulated (Please Staphylococcus, Encapsulated species
SHI!\E my SKiS): Burkholderia ce(Jacia, with early component
• 13
Pseudomonas Pseudomonas deficiencies
• 14 aerugmosa, aeruginosa, Serratia, 'eisseria with late
• 15 Streptococcus acardia complement (C5-
• 16 pneumoniae, C9) deficiencies
• 17
llaemophilus t
Influenzae type B,
• 18
Neisseria
• 19 meningitidis,
• 20 Escherichia coli,
• 21 Salmonella,

6
lock
s
Suspend
0
End Block
Item: 8 of 25 ~ 1 • M k -<:J 1>- Jil ~· !:';-~
QIO: 3 0 85 ..L ar Pre v ious Next Lab fli!ltues Notes Calcula t o r

1
aerugmosa, aeruginosa, Serratia, 1eisseria with late
2
Streptococcus 1ocardia complement (C5-
3 pneumoniae, C9) defi ciencies
4 llaemophilus
5 Influen;:ae l)'])e B.
6
i\'eisseria
meningitidis,
7
Escherichia coli,
8 Salmonella,
.9 1\lebsiei/C/
• 10 pneumoniae,
• 11 Group B
Streptococcus
• 12
Viruses C MV, EB , )C Enterovi ra I 1/A I /A
• 13
virus, VZV, chronic cnccphal it is,
• 14
infection with poliovirus
• 15 respiratory/CI viruses (live vaccine
• 16 contraind icatcd)
• 17 Fungi/parasites Candida (local), PC P, G l giardiasis (no lgA) Candida (systemic), 1 /A
• 18 Cryptococcus Aspergillus, Mucor
• 19 1ote: B-cell deficiencies tend to produce recurrent bacterial infections, whereas T-cell deficiencies produce more fungal and
• 20 vira I in fcctions.
• 21

a
Lock
s
Suspend
8
End Bl ock
Item: 9 of 25 ~ 1 • M k -<:J 1>- Jil ~· !:';-~
QIO: 3271 ..L ar Pre v ious Next Labfli!llues Notes Calcula t o r

IAA]
& &
1
A 43-year-old woman visits her obstetrician for her scheduled check-up at 20 weeks' gestation . Her
2
obstetrician recommends that she have a series of screen ing tests. One of these tests shows an elevated
3 serum a-fetoprotein (AFP) level.
4
5 What is the most likely cause of this patient's abnormal laboratory value?
6 :
7 A . Bilateral renal agenesis
8
B. Failure of the ductus arteriosus to close
.9
C. Inadequate folic acid intake
• 10
• 11 D. Inheritance of a recessive disorder
• 12 E. Nond isjunction occurring during meiosis
• 13
• 14
• 15
• 16
• 17
• 18
• 19
• 20
• 21

a
Lock
s
Suspend
8
End Bl ock
Item: 9 of 25 ~. I • M k <:] t> al ~· ~
QIO: 3271 .l. ar Previous Next Lab 'lifllues Notes Calculator

1 •

2 The correct answer is C. 70°/o chose this.


3 An elevated AFP level in amn iotic fluid and maternal serum may ind icate neu ral tube defects such as
meningocele, and meningomyeloce le. In contrast, spina bifida occulta would not have an elevated AFP level
4
because the du ral lining is maintained . These defects are caused by the failure of the caudal portion of the
5 neura l tube to close. The AFP level is elevated because it is leaked into the amniotic fluid from the neu ral tube
6 defect. Chi ldren with these defects suffer from a va rying degree of symptoms that usually include motor and
7 sensory defects in the lower extremit ies and dysfunction of bowe l and bladder control. Folate deficiency during
the first 4 wee ks of pregnancy has been imp licated in causing neural tube defects. Drugs that increase the risk
8
of neura l tube defect include valproate and carbamazepine.
9 Neural tube defect Carbamazepine Spina bifida Amniotic fluid Neural tube Folic acid Valproate Folate deficiency Serum (blood) Dura mater Urinary bladder

0 10 Pregnancy Blood plasma

· 11
A is not correct. 4°/o chose this.
0
12
Bilateral rena l agenesis (Potter sequence) is caused by disruption in the interaction between the ureteric bud
0
13 and the metanephrogenic tissue. Because the fetus does not produce urine (which is a component of amniotic
0 14 fluid), there is a sma ller volume of amniotic fluid than normal. This is described by the term oligohyd ramnios .
0 15
The sma ller amount of protective fluid resu lts in pulmonary hypoplasia, feta l compression with altered facies,
and positioning defects of hands and feet. Bilatera l rena l agenesis is not compatible with life. Oligohydramnios,
0
16 not increased AFP levels, wou ld be noted in this case .
0 17 Oligohydramnios Renal agenesis Pulmonary hypoplasia Potter sequence Amniotic fluid Fetus Urine Hypoplasia Kidney

18
0

B is not correct. 2°/o chose this.


19
0
The ductus arteriosus is a connection between the pu lmonary artery and the aorta that allows oxygenated blood
0 20 from the placenta to bypass the feta l lungs and enter the systemic circulation . This pathway shou ld be open
21 during gestation and is not an abnorma lity. At birth, as the infant takes a breath, an increase in oxygen content
T& -L.-
0

• --• ·--- - .... __ ..., ____ :- - ....--•--1--..J :-- .... --· ,J.._ : __ : - - 1--. ,...,_ -& ~L..- ------~ = -- ..J, · -• • ·- _....,_...,: __ , ·- ..., ___ : __

6
lock
s
Suspend
0
End Block
Item: 9 of 25 ~. I • M k <:] t> al ~· ~
QIO: 3271 .l. ar Previous Next lab 'lifllues Notes Calculator

1 • B is not correct. 2°/o chose this .


2 The ductus arteriosus is a connection between the pu lmonary artery and the aorta that allows oxygenated blood
3
from the placenta to bypass the feta l lungs and enter the systemic circulation . This pathway shou ld be open
during gestation and is not an abnorma lity. At birth, as the infant takes a breath, an increase in oxygen content
4
causes a decrease in prostagland ins, resulting in closure of the connection . I f the ductus arteriosus remains
5 patent after birth, the baby can be given indomethacin to help stimulate the vessel to close.
6 Indometacin Placenta Pulmonary artery Aorta Ductus arteriosus Circulatory system Prostaglandin Gestation Oxygen lung Blood

7 D is not correct. 4°/o chose this.


8 It is possible that th is fetus inherited a recessive disorder such as cystic fibrosis, phenylketonu ria, or sickle ce ll
9 anemia. However, these diseases are usual ly tested for in patients with a family history, using DNA studies of
fetal cells collected from amniotic fluid, not by measuring AFP levels.
0 10
Phenylketonuria Cystic fibrosis Sickle-cell disease Amniotic fluid Fetus Anemia Dominance (genetics) Fibrosis DNA Recessive
· 11
0
12
E is not correct. 20% chose this.
Nondisjunction during meiosis is the usual cause of trisomy 21, the genetic abnormality in Down syndrome .
0
13
Trisomy 21 is more common in the ch ildren of women >35 years old, so this patient's age puts her at increased
0 14 r isk of having a baby with Down synd rome. However, unl ike neu ral tube defects, Down syndrome causes a
0 15 decrease in AFP leve ls and an increase in ~-human chorionic gonadotropin levels.
Down syndrome Nondisjunction Meiosis Trisomy Neural tube Neural tube defect Gonadotropin
0
16
0 17
0
18 Bottom Line:
19
0
Elevated AFP levels in the amniotic f luid and materna l serum can be a sign of neu ral tube closure defects such
0 20 as meningoce le and meningomyeloce le. Folate deficiency can lead to neural tube defects.
Amniotic fluid Spina bifida Folic acid Neural tube Folate deficiency Neural tube defect Blood plasma
0
21

6
lock
s
Suspend
0
End Block
Item: 9 of 25 ~ 1 • M k -<:J 1>- Jil ~· !:';-~
QIO: 3271 ..L ar Pre v ious Next Labfli!llues Notes Calcula t o r
& &
1
2 FA17 p 461 .1

3 Neural tube defects Neuropores fail to fuse (4th week) - persistent con nection between amniotic cavity and spinal
4 canal. Associated\\ ith maternal dit~betes as \\ell as low folic acid intake before conception and
5 during pregnancy. t a-fetoprotein (AFP) in amniotic Auid and maternal serum (except spina
bifida occulta =normal F'P). t acetylcholinesterase ( ChE) in amniotic Auid is a helpful
6
confirmatory test (fetal AChE in CSF' AO\\S through defect into amniotic Auid).
7
Anencephaly Failure of rostral neuropore to close - no forebrain, open cakarium. Clinical findings:
8
polyhydramnios (no swallowing center in brain).
9
Spina bifida occulta Failure of caudal neuropore to close, but no herniation. Usually seen at lower vertebral levels. Dura
• 10
is intact. Associated with tuft of hair or skin dimple at level of bony defect.
• 11
Meningocele \leninges (but no neural tissue) herniate through bony defect. Associated with spina bifida C)Stiea.
• 12
Meningomyelocele ~ leninges and neural tissue (cg, ct~uda cquina) herniate through bony defect.
• 13
_, , / +1- Tuft of hair / Skin defecVthinning / Skin thin or absent
• 14 ~ +1- Skin dimple
Skin ~
• 15
• 16
{ /;Dura
• 17 y-Leptomeninges
• 18
If~
Sptnal ~
• 19
\_Transverse
• 20 cord process
• 21

a
Lock
s
Suspend
8
End Bl ock
Item: 9 of 25 ~ 1 • M k -<:J 1>- Jil ~· !:';-~
QIO: 3271 ..L ar Pre v ious Next Labfli!llues Notes Calcula t o r
& &
1

2 FA17 p59.1
3 Autosomal trisomies
4 Down syndrome Findings: intellectual disabil it), Aat facies, Incidence 1:700.
5 (trisomy 21 ) prominent epicanthal folds, single palmar Orin king age (21).
6 crease, gap between 1st 2 toes, duodenal \lost common ,·iable chromosomal disorder and
atresia, Hirschsprung disease, congenital heart mo~t common cause of genetic intellectual
7
disease (eg, atrioventricular septal defect), disability.
8
Brushfield spots. ssociated with early-onset First-trimester ultrasound commonly shows
9 Alzheimer disease (chromosome 21 codes for t nuchal translucency and hypoplastic nasal
• 10 amyloid precursor protein) and t risk of ALL bone; I serum PAPP-A, t free P-hCC .
• 11 and A~ IL. Second-trimester quad screen shows
95% of cases due to meiotic nondisju nction I a -fctoprotein, t P-hCC, I estriol,
• 12
(t with advanced materna I age; from I:1500 in t inhibin
• 13
women< 20 to 1:25 in women > 45 yea rs old).
• 14 4% of cases clue to unbalanced Robertson ian
• 15 translocation, most typically between
• 16 chromosomes 14 and 21. 1% of cases due
• 17
to mosaicism (no association with materna l
nondisjunction; postfertilization mitotic error).
. 18
Edwards syndrome Findings: PRT!'\CF: Edward- Prominent Incidence 1:8000 .
• 19
(trisomy 18) occiput, Rocker-bottom feet, Intellectua l Election age (18).
• 20
disability, " ondisjunction, Clenched fists 2nd most common autosomal trisomy resulting
• 21 (with o,·erlapping fin gers), low-set Ears, in Ji,·e birth (most common is Down syndrome).

a
Lock
s
Suspend
8
End Bl ock
Item: 9 of 25 ~ 1 • M k -<:J 1>- Jil ~· !:';-~
QIO: 3271 ..L ar Pre v ious Next Labfli!llues Notes Calcula t o r
&
- . . .
&
1
Edwards syndrome Find ings: PRI!\C E Edwa rd- P rominent Incidence 1:8000.
2
(trisomy 18) occiput, Rocker-bottom feet, Intellectual Election age (18).
3 disability, " ondisjunction, C lenched fists 2nd most common autosomal trisomy resulting
4 (with o,·erlapping fin gers), low-set Ears, in Ji,·e birth (most common is Down syndrome}.
5 micrognathia (small jaw), congenital heart PAPP-A and free ~-hCG are l in first trimester.
disease. D eath usually occurs by age I. Quad screen shows l a-fetoprotein, l ~-hCG,
6
' estriol, ' or normal inhibin A.
7
Patau syndrome Findings: se\·ere intellectual disabilit). rocker- Incidence l:I ;,ooo.
8
(trisomy 13) bottom feet, microphthalmia, microcephaly, P uberty ( 13).
9
cleft liP/Palate, holoP rosencephaly, First-trimester pregnancy screen shows ' free
• 10 Polydact}ly, cutis aPlasia, congen ital heart ~-h C G , ' P. PP-.\.
• 11 disease. D eath usuall) occurs by age I.
• 12 Nondisjunction in meiosis I Nondisjunction in meiosis II
• 13
• 14 t J t J
t-
Meiosis I
• 15
t ~
~
• 16

"""'i'""""):
A c)
• 17
. 18
• 19 c :> c :> c :>
c :> Meiosis II
• 20 c :> c :> ~
• 21
• I I I Nondi.sjunction

a
Lock
s
Suspend
8
End Bl ock
Item: 9 of 25 ~ 1 • M k -<:J 1>- Jil ~· !:';-~
QIO: 3271 ..L ar Pre v ious Next Labfli!llues Notes Calcula t o r

)'\,. A A
& &
1
2
/'\,.
3
4
Ill Ill I
n+1 n+1
I Gametes
II II I Ill n +1
n-1 n- 1 n n n-1
5
Trisomy Monosomy Normal Mooosomy Trisomy
6
7
FA17 p 64..2
8
Vitamin 8 9 (folate)
9
FUNCTION Converted to tctrahydrofolic acid (T i ll'}, a Found in leafy green vegetables. Absorbed in
• 10
coenz}me for !-carbon transfer/methylation jejunum. Folate from fol iage.
• 11 reactions. Small reserve pool stored primarily in the liver.
• 12 Important for the synthesis of nitrogenous bases
• 13 in DNA and R 'A.
• 14 DEFICIENCY lacrocytic, megaloblastic anemia; Deficiency can be caused by several drugs (cg,
• 15 hypersegmented polymorphonuclear cells phenytoin, sulfonam ides, methotrexate).
(PMNs); glossitis; no neurologic S)•mptoms Supplemental maternal folic acid at least I month
• 16
(as opposed to vitamin B12 de~ciency). Labs: prior to conception and during early pregnancy
• 17
f homocysteine, normalmethylmalonic acid to ! risk of neural tube defects.
• 18 le,·els. Most common vitam in deficiency in
• 19 the United States. Seen in alcoholism and
• 20 pregnancy.
• 21

a
Lock
s
Suspend
8
End Bl ock
Item: 10 of 25 ~ 1 • M k -<:J 1>- Jil ~· !:';-~
QIO: 2575 ..L ar Pre v ious Next Lab fli!ltues Notes Calcula t o r

IAA]
& &
1
A patient is seen in the clinic . He has an autosoma l-recessive disorder that resu lted in a deficiency in
2
tyrosinase activity in specific cells. He appears pale, with light blond hair and blue eyes, and notes that he
3 sunburns easily and that his eyes are very sensitive to light.
4
5 The cells most involved in this condition are derived from which of the following germ cell layers?
6 :
7 A. Ectoderm
8
B. Endoderm
9
C. Mesoderm
• 10
• 11 D. Neuroectoderm, neura l crest cells
• 12 E. Neuroectoderm, non -neural crest cells
• 13
• 14
• 15
• 16
• 17
• 18
• 19
• 20
• 21

a
Lock
s
Suspend
8
End Bl ock
Item: 10of25 ~. , . M k <:] t> al ~· ~
QIO: 2575 .l. ar Previous Next lab 'lifllues Notes Calculator

1 •

2 The correct answer is D. 66°/o chose this.


Albinism, the lack of pigmentation in the hair, skin, and eyes, resu lts from a lack of tyrosinase activity in
3
melanocytes. Without tyrosinase, these cells are unable to convert tyrosine to me lanin, putting the aff ected
4 pe rson at increased risk of squamous and basa l cel l carcinoma and ma lignant me lanoma. Melanocytes,
5 odontoblasts, pia and arachnoid mater, Schwann cel ls, cells of the gang lia, parafol licu lar C cel ls of the thyroid,
6 chromaffin cells, the aorticopulmonary septum, pharyngeal arch skeleta l components, and the neu rocranium are
all derived from neu ral crest cells, a specific portion of the neu roectoderm.
7
Parafollicular cell Pharyngeal arch Arachnoid mater Basal-cell carcinoma Melanin Neural crest Melanoma Neuroectoderm Schwann cell Thyroid Albinism
8
Melanocyte Septum Chromaffin cell Tyrosine Pigment Malignancy Carcinoma Pharynx
9
10
A is not correct. 16% chose this.
Although the ectoderm does give rise to the epidermis and the hair (as well as many other structu res), it does
· 11
not produce melanocytes .
• 12 Ectoderm Epidermis Epidermis (botany) Epidermis (zoology) Melanocyte

• 13
B is not correct. 4°/o chose this .
• 14
The endode rm is responsib le fo r the epithel ial lining of many internal organs as well as other structures, but
• 15 plays no role in albinism .
• 16 Endoderm Albinism Epithelium Squamous epithelial cell

• 17
C is not correct. 7°/o chose this .
• 18 The mesoder m produces ma ny structures, incl uding the dermis, but does not produce melanocytes.
• 19 Mesoderm Dermis Melanocyte

• 20 E is not correct. 7°/o chose this .


• 21 The neu roectoderm cel ls outside of the neura l crest are the sou rce of the iris (as well as several other optic and •

6
lock
s
Suspend
0
End Block
Item: 10of25 ~. , . M k <:] t> al ~· ~
QIO: 2575 .l. ar Previous Next lab 'lifllues Notes Calculator

1
Melanocyte Septum Chromaffin cell Tyrosine Pigment Malignancy Carcinoma Pharynx
2
A is not correct. 16% chose this.
3
Although the ectoderm does give rise to the epidermis and the hair (as wel l as many other structu res), it does
4
not produce melanocytes .
5 Ectoderm Epidermis Epidermis (botany) Epidermis (zoology) Melanocyte

6
B is not correct. 4°/o chose this.
7
The endoderm is responsib le for the epithel ial lin ing of many internal organs as well as other structures, but
8 plays no ro le in albinism .
9 Endoderm Albinism Epithelium Squamous epithelial cell

10 C is not correct. 7°/o chose this.


· 11 The mesoder m produces many structures, including the dermis, but does not produce melanocytes.
• 12 Mesoderm Dermis Melanocyte

• 13 E is not correct. 7°/o chose this .


• 14 The neuroectoderm cel ls outside of the neu ra l crest are the source of the iris (as wel l as several other optic and
• 15 non -optic str uctures) but do not give rise to melanocytes .
Neuroectoderm Neural crest Iris (anatomy) Melanocyte
• 16
• 17

• 18 Bottom Line:
• 19 Albinism results from a lack of tyrosinase activity in melanocytes, which are der ived from neural crest cells .
• 20 Tyrosinase Albinism Neural crest Melanocyte

• 21

6
lock
s
Suspend
0
End Block
Item: 10 of 25 ~ 1 • M k -<:J 1>- Jil ~· !:';-~
QIO: 2575 ..L ar Pre v ious Next Lab fli!ltues Notes Calcula t o r
& &
1
FA17 p 448.2
2
Pigmented skin disorders
3
Albinism 'ormal melanocyte number" ith l melanin production fJ due to l tyrosinase activitr or defective
4 tyrosine transport. t risk of skin cancer.
5 Melasma (chloasma) I Iyperpigmentation associated \\it h pregnancy ("mask of pregnancy" 1m or OCP use.
6
Vitiligo Irregular areas of complete depigmentation . Caused by autoimmune destruction of melanocrtes.
7
8
9
10
• 11

• 12
• 13
• 14
• 15
• 16
• 17
FA17p581 .1
. 18
Embryologic derivatives
• 19
Ectoderm E\Lernal/ouler layer
• 20
Surface ectode rm Epidem1is; adenohypophysis (from Rathke Cra niopharyngioma-benign Rathke ponch
• 21
• nnw·h \· IPn~ nf P~P· Pni th pJj,. J Jinino~ nf nr<1l lumnr u.1 ith Phni PdPrnl pn·d::~l~ r,.Jrifir:>t inn~

a
Lock
s
Suspend
8
End Bl ock
Item: 10 of 25 ~ 1 • M k -<:J 1>- Jil ~· !:';-~
QIO: 2575 ..L ar Pre v ious Next Lab fli!ltues Notes Calcula t o r
& &
1

2 FA17p581 .1
3 Embryologic derivatives
4 Ectoderm E'tcrnallouter layer
5 Surface ectoderm Epidem1is; adenohypophysis (from Rathke Craniopharyngioma -benign Rathke pouch
6 pouch); lens of e)e; epithelial lining~ of oral tumor with cholesterol cr)stals, calcifications.
7 cavity, sensory organs of ear, and olfacto~
epithelium; anal canal below the pectinate line;
8
parotid, sweat, mammar) glands.
9
Neural tube Brain (neurohypophysis, Ci S neurons, oligo- Neuroectoderm - think CI\S.
10
dendrocytes, astrOC} tcs, epcnd) mal cells, pineal
• 11
gland), retina, spinal cord .
• 12
Neural crest P S {dorsal root ganglia, cranial nerves, Neural crest- think P Sand non-neural
• 13 autonomic ganglia, Schwann cells), structures nearby.
• 14 melanocytes, chromaffin cells of adrenal
• 15 medulla, para follicu lar (C) cells of thyroid,
pia and arachnoid, bones of the skull,
• 16
odontoblasts, aorticopulmonary septum,
• 17
endocardial cushions, myenteric ( ucrbach)
. 18 plexus.
• 19 1\ Iiddlef'meat" layer.
Mesoderm Muscle, bone, connective tissue, serous
• 20 linings of body cavities (eg, peritoneum), r-..Jesodermal defects= VACTERL:
• 21 spleen (derived from foregut mesentery), \ ertebral defects

a
Lock
s
Suspend
8
End Bl ock
Item: 10 of 25 ~ 1 • M k -<:J 1>- Jil ~· !:';-~
QIO: 2575 ..L ar Pre v ious Next Lab fli!ltues Notes Calcula t o r

1 L 0 0 g g , ), s s
2
melanocytes, chromaffin cells of adrena l
medulla, para follicu lar (C) cells of thyroid,
3
pia and arachnoid, bones of I he skull,
4
odontoblasts, aorticopulmonary septum,
5 endocardial cushions, myenteric ( ucrbach)
6 plexus.
7 Mesoderm i\ Iuscle, bone, connective tissue, serous \ I iddle/meat" lm·er.
8 linings of body cavities (eg, peritoneum), 1\ lesodcrmal defects = VACTERL:
spleen (deri\ ed from foregut mesentery), \ erlebral defects
9
cardio,-ascular structures, lymphatics, blood, Anal atresia
10
wall of gut tube, upper ,·agina, l..idneys, Cardiac defects
• 11 adrenal cortex, dermis, testes, ovaries. Tracheo-Esophageal fi stula
• 12 'otochord induces ectoderm lo form Renal defects
• 13 neuroectoderm (neural plate). Its only Limb defects (bone and muscle)
• 14
postnatal derivative is the nucleus pulposus of
the intervertebral disc.
• 15
Endoderm Gut tube epithelium (including ana l canal "I'•.ntcrna 1" 1ayer.
• 16
above the pecti nate line), most of urethra and
• 17
lower vagina (derived from urogenita l sinus),
• 18 luminal epithelial derivatives (eg, lungs,
• 19 liver, gallbladder, pancreas, eustachian lube,
• 20 thymus, parathyroid, thyroid follicu Jar cells).
• 21

a
Lock
s
Suspend
8
End Bl ock
Item: 11 of 25 ~. I • M k <:] t> al ~· ~
QIO: 1077 .l. ar Previous Next lab 'lifllues Notes Calculator

1 •
A 20-yea r-old pri migravid female comes into the labor and delivery wa rd of the local hospital complaining of
2 contractions. She states she thinks she noticed some "fluid leaking" that wet her underga r ments. She denies
3 any prenatal care and th inks she is "about 37 weeks" gestation. The patient goes into labor and vag inally
4 delivers an infant with a small head, low-set ears, widely spaced eyes, and a thin upper lip. Other abnor mal ities
include limb deformities and a left-to-r ight shunt.
5
6
I n utero exposu re to wh ich of the following substances is most consistent with this patient's presentation'
7

8 :
A. Chlorothiazide
9
10 B. Doxycycl ine
• 11 C. Ethano l
• 12
D. Lithium
• 13
• 14
E. Valproic acid
• 15
• 16
• 17

• 18
• 19
• 20
• 21

6
lock
s
Suspend
0
End Block
Item: 11 of 25 ~ 1 • M k -<:J 1>- Jil ~· !:';-~
QIO: 1 0 77 ..L ar Prev ious Next Lab fli!ltues Notes Calculat o r

& &
1
2 The correct answer is C. 68°/o chose this.
3 Consumption of ethanol during pregnancy (particularly between weeks 3
4 through 8 1 when organogenesis occurs) is responsible for more congenital
5 malformations than any other substance and is the leading cause of
preventable intellectual disability. Fetal alcohol syndrome refers to a
6
cluster of abnormal findings that are associated with a high level of ethanol
7 consumption during pregnancy . Findings include pre- and postnatal
8 developmental retardation/ microcephaly/ facial abnormalities (low-set ears/
widely spaced eyes/ thin upper lip/ like that shown in the image)/ limb
9
dislocation/ and heart and lung f istulas. The mechanism responsible for these
10 deformities may include inhibit ion of cell migrat ion.
11 Fetal alcohol spectrum disorder Microcephaly Ethanol Intellectual disability Organogenesis Congenital disorder

• 12 Postpartum period Lung Pregnancy Up Alcohol Cell migration

• 13
• 14
• 15
• 16 I mage courtesy of Teresa
• 17 Kellerman
• 18
A is not correct. 4°/o chose this .
• 19
Chlorothiazide is a thiazide diuretic used as an antihypertensive. Its use during preg nancy is associated w ith
• 20 fetal thrombocytopenia and jaund ice.
• 21 Chlorothiazode Th1az de Diuretic Jaundice Thrombocytopenia Antihypertensive drug Pregnancy

a
Lock
s
Suspend
8
End Block
Item: 11 of 25 ~. I • M k <:] t> al ~· ~
QIO: 1077 .l. ar Previous Next Lab 'lifllues Notes Calculator
• I I - • . I I -
1
Chlorothiazide is a thiazide di uretic used as an antihypertensive. Its use during pregnancy is associated with
2
fetal thrombocytopenia and jaund ice.
3 Chlorothiazide Thiazide Diuretic Jaundice Thrombocytopenia Antihypertensive drug Pregnancy

4
B is not correct. 4°/o chose this.
5
Doxycycline is a tetracycline antibiotic used frequently for the treatment of Lyme disease and for gen ital
6 infections; its use du r ing pregnancy is associated with permanent tooth staining and ena m el hypoplasia.
7 Tetracycl ines are avoided in children younger than 8 years, unless there is a compel ling reason for
8
administration (eg, Rocky Mountain spotted fever ).
Rocky Mountain spotted fever Lyme disease Doxycycline Tetracycline Antibiotics Tetracycline antibiotics Pregnancy Fever Tooth enamel
9
10 Dis not correct. 11% chose this.
11 Lithium use during pregnancy resu lts in a high incidence of feta l cardiac abnormalities, incl uding Ebstein
anomaly and malfor mations of the great vessels .
• 12
Pregnancy lithium
• 13
E is not correct. 13% chose this .
• 14
Valproic acid is a frequently used antiepileptic drug that can cause feta l neural tube defects, cleft lip, and rena l
• 15
anomal ies .
• 16 Valproate Anticonvulsant Cleft lip and palate Neural tube Neural tube defect Kidney

• 17

• 18
Bottom Line:
• 19
Fetal alcohol syndrome is character ized by intellectual disability, microcephaly, facia l abnor malities ( eg, low-
• 20
set ears, widely spaced eyes, thin upper lip), and limb anomalies .
• 21 Fetal alcohol spectrum disorder Microcephaly Intellectual disability lip Alcoholic beverage low-set ears Dysmelia Disability Alcohol Fetus

6
lock
s
Suspend
0
End Block
Item: 11 of 25 ~ 1 • M k -<:J 1>- Jil ~· !:';-~
QIO: 10 77 ..L ar Pre v ious Next Lab fli!ltues Notes Calcula t o r
& &
1
FA17 p 583.1
2
3 Fetal alcohol Leading cause of intellectual disability in the US. 1e" borns of alcohol-consuming mothers
4
syndrome have t incidence of congenital abnormalities, including pre- and postnatal de,-eJopmental
retardation, microcephaly, facial abnorma lities (eg, smooth philtrum, thin vermillion border
5
[upper lip]. small palpebral fissures), limb dislocation, heart defects. Heart-lung fistulas and
6 holoprosencephaly in most severe form. \lechanism is failure of cell migration.
7
8
9
10
11
• 12 FA17 p 541 .2
• 13 Alcoholism Physiologic tolerance and dependence on alcohol with symptoms of withdrawal when intake is
• 14 interrupted.
• 15 Compl ications: alcoholic cirrhosis, hepatitis, pancreatitis, peripheral neuropathy, testicular atrophy.
• 16 Treatment: disulfiram (to condition the patient to abstai n from alcohol use), acamprosate,
naltrexone, supportive care. Support groups such as Alcoholics Anonymous are helpful in
• 17
sustaining abstinence and supporting patient and fami ly.
• 18
We rnicke-Korsakoff Caused by vitamin B1 deficiency. Triad of con fusion, ophthalmoplegia, ataxia (Wernicke
• 19
syndrome encephalopathy). May progress lo irreversible memory loss, confabulation, personality change
• 20 (Korsakoff syndrome). Symptoms may be precipitated by giving dextrose before administering
• 21 ,·itamin B1 to a patient with thiamine deficiencr- Associated with peri,·entricular hemorrhage/

a
Lock
s
Suspend
8
End Bl ock
Item: 11 of 25 ~ 1 • M k -<:J 1>- Jil ~· !:';-~
QIO: 10 77 ..L ar Pre v ious Next Lab fli!ltues Notes Calcula t o r
& &
1

2 FA17 p 541 .2

3 Alcoholis m Physiologic tolerance and dependence on alcohol with symptoms of withdrawal when intake is
4 interrupted.
5 Complications: alcoholic cirrhosis, hepatitis, pancreatitis, peripheral neuropathy, testicular atrophy.
Treatment: disulfiram (to condition the patient to abstain from alcohol use), acamprosate,
6
naltrexone, supporti,·e care. Support groups such as Alcoholics Anonymous are helpful in
7 sustaining abstinence and supporting patient and famil}.
8
Wernicke-Korsakoff Caused by \"ita min 8 1 deficiency. Triad of confusion, ophthalmoplegia, ataxia (Wernicke
9 syndrome encephalopathy). ~ l ay progress to irreversible memory loss, confabulation, personality change
10 (Korsakoff syndrome). Symptoms may be precipitated by gi,·ing dextrose before administering
11 ,·itamin 8 1 to a patient with thiamine deficiency. Associated with peri,·entricular hemorrhage/
• 12
necrosis of mammillary bodies. Treatment: I vitamin 8 1•

• 13
FA1 7 p 582.1
• 14
Teratogens Most susceptible in 3rd-8th weeks (embryonic period-organogenesis) of pregnancy. Before week
• 15
3, "all-or-none" effects. After week 8, growth and function affected .
• 16
TERATOGEN EFFECTS ON FETUS NOTES
• 17
Medications
. 18
ACE inhibitors Renal damage
• 19
• 20 Alkylating agents Absence of digits, multiple anomalies

• 21 Aminoglycosides Ototoxicity A mean guy hit the baby in the ear.



a
Lock
s
Suspend
8
End Bl ock
Item: 11 of 25 ~- I • M k <:] t> al ~· ~
QIO: 1077 .l. ar Previous Next lab 'lifllues Notes Calculator

1 • •

2 FA17 p 582.1

3 Teratogens Most susceptible in 3rd-8th weeks (embryonic period-organogenesis) of pregnancy. Before week
4 3, "all-or-none" effects. After week 8, growth and function affected.
5 TERATOGEN EFFECTS ON FETUS NOTES

6 Medications
7 ACE inhibitors Renal damage
8 Alkylating agents Absence of digits, multiple anomalies
9 Aminoglycosides Ototoxicity A m ean guy hit the baby in the ear.
10 Antiepileptic drugs Neural tube defects, cardiac defects, cleft High-dose folate supplementation
11 palate, skeletal abno rmalities (eg, phalanx/nail recommended. Most commonly ,-alproate,
• 12 hypoplasia, facial dys morphism) carbamazepine, phenytoin, phenobarbital.
• 13 Diethylstilbestrol Vaginal clear cell adenocarcinoma, congen ital
• 14 Miillcrian anomalies
• 15 Folate antagonists Neural tube defects Jncludes trimethoprim, methotrexate,
antiepileptic drugs .
• 16
• 17 lsotretinoin Multiple severe birth defects Contraception mandatory. lsoTER. \.Tinoin.

• 18 lithium Ebstein anomaly (apical displacement of


tricuspid valve)
• 19
• 20
Methimazole Aplasia cutis congenita

• 21 Tetracyclines Discolored teeth, inhibited bone growth "Teeth racycl ines."


• •

6
lock
s
Suspend
0
End Block
Item: 11 of 25 ~ 1 • M k -<:J 1>- Jil ~· !:';-~
QIO: 10 77 ..L ar Pre v ious Next Lab fli!ltues Notes Calcula t o r
&
-·· - - - - - - - - . . &
1
tricuspid ,·a lve)
2
Methimazole Aplasia cutis congenita
3
Tetracyclines Discolored teeth, inhibited bone gro" t h "Teeth racycl ines."
4
Thalidomide Limb defects (phocomelia, micromelia- Limb defects with "tha-limb-domide."
5
"Aipper"' limbs)
6
Warfarin Bone defom1ities, fetal hemorrhage, abor tion, Do not wage wa rfa re on the baby; keep it hepp)
7 ophthalmologic abnormalities with heparin (does not cross placenta).
8 Substance abuse
9
Alcohol Common cause of birth defects and intellectual
10 disabi lity; fetal alcohol syndrome
11 Cocaine Low birth weight, pre term birth, IUGR, Cocaine - vasoconstriction.
• 12 placental abruption
• 13 Smoking Low birth weight (leading cause in developed 'icotine - vasoconstriction.
• 14 (nicotine, CO) countries), preterm labor, placental problems, CO - impaired 0 2 delivery.
• 15 TUCR, SIDS
• 16 Other
• 17 Iodine (lack or excess) Congenital goiter or hypothyroidism (cretinislll)
. 18 Maternal diabetes Caudal regression syndrome (anal atresia
• 19 to sirenomelia), congenital heart defects,
neural tube defects, macrosomia, neonatal
• 20
hypoglycemia
• 21
• ••-•&.....1-"".'" ......
a
Lock
s
Suspend
8
End Bl ock
Item: 11 of 25 ~ 1 • M k -<:J 1>- Jil ~· !:';-~
QIO: 10 77 ..L ar Pre v ious Next Lab fli!ltues Notes Calcula t o r

1
••. - . · "" ' g, g }. p I I)
ophthalmologic abnormalities with heparin (does not cross placenta).
2
Substance abuse
3
Alcohol Common cause of birth defects and intellectual
4
disability; fetal alcohol syndrome
5
Cocaine Lo'' birth weight, preterm birth, IUGR, Cocaine ..... msoconstriction.
6
placental abruption
7
Smoking Low birth weight (leading cause in developed 1\icotine ..... ,·asoconstriction.
8 (nicotine, CO) countries), preterm labor, placental problems, CO ..... impaired 0 2 deJi,·err
9 IUGR, SIDS
10 Other
11 Congenital goiter or hypothrroidism (cretinism)
Iodine (lack or excess)
• 12
Maternal diabetes Caudal regression syndrome (anal atresia
• 13 to sirenomelia), congenital heart defects,
• 14 neural tube defects, macrosomia, neonatal
• 15 hypoglycemia
• 16 Methylmercury Neurotoxicity Highest in swordfish, shark, tilefish, king
• 17 mackerel.
. 18 Vitamin A excess Extremely high risk for spontaneous abortions
• 19
and birth defects (cleft palate, cardiac)

• 20
X-rays J\1icrocephaly, intellectual disabil it} li nimized by lead shielding.
• 21

a
Lock Suspend
s 8
End Bl ock
Item: 12 of 25 ~ 1 • M k -<:J 1>- Jil ~· !:';-~
QIO: 4425 ..L ar Pre v ious Next Lab fli!ltues Not es Calcula t o r

IAA]
& &
1
A 25-year-old woman with normal int el ligence, short stature, large head w ith front al bossing, and shortened
2
limbs sees her physician for genetic counseling prior to conception. She is told there is a 50% chance of
3 passing on a mutation in the f ibroblast growth factor receptor (FGFR) to her offspring.
4
5 Impaired maturation of which tissue results in this condition?
6 :
7 A. Bone
8
B. Cartilage
9
C. Skeletal muscle
10
11 D. Skin
• 12 E. Smooth muscle
• 13
• 14
• 15
• 16
• 17
• 18
• 19
• 20
• 21

a
Lock
s
Suspend
8
End Bl ock
Item: 12of25 ~. , . M k <:] t> al ~· ~
QIO: 4425 .l. ar Previous Next lab 'lifllues Notes Calculator

2
The correct answer is B. 68°/o chose this.
3 This patient has achond roplasia, wh ich is usua lly caused by a spontaneous mutation in the FGFR gene. I t is an
4 autosomal dominant trait and can subsequently be passed on to futu re generations. Endochondra l ossification,
5 w hich is requi red for long bone growth, is impaired, resulting in short statu re. As the associated mutation
causes a constitutive activation of the FGFR, it leads to abnoma l chond roid production by chondroblasts in the
6
prol iferative zone at the physis. Intramembranous ossification is unaffected.
7 Intramembranous ossification Achondroplasia Endochondral ossification Gene long bone Dominance (genetics) Chondroblast Ossification Bone Autosome Mutation

8
A is not correct. 26% chose this.
9
Osteogenesis imperfecta is a genetic disorde r affecting type 1 collagen that makes up bones. Depending on the
10 type of the disorde r, patients can be seve rely deformed, suffe r numerous fractu res from minor trauma, or have
11 short stature.
Osteogenesis imperfecta Collagen Genetic disorder Ossification Short stature
12
• 13 C is not correct. 3°/o chose this .
• 14 Achond roplasia does not affect ske leta l muscle .
Achondroplasia Muscle
• 15
• 16 Dis not correct. 1°/o chose this .
• 17 Achond roplasia does not affect the skin .
Achondroplasia
• 18
• 19 E is not correct. 2°/o chose this .
• 20 Achond roplasia does not affect smooth muscle .
Achondroplasia Muscle
• 21

6
lock
s
Suspend
0
End Block
Item: 12of25 ~. , . M k <:] t> al ~· ~
QIO: 4425 .l. ar Previous Next lab 'lifllues Notes Calculator

1

2 A is not correct. 26% chose this.


3 Osteogenesis imperfecta is a genetic disorder affecting type 1 collagen that makes up bones. Depending on the
type of the disorder, patients can be severely deformed, suffer numerous fractu res from minor trauma, or have
4
short stature.
5 Osteogenesis imperfecta Collagen Genetic disorder Ossification Short stature

6
C is not correct. 3°/o chose this.
7
Achondroplasia does not affect skeletal muscle.
8 Achondroplasia Muscle
9
Dis not correct. 1°/o chose this.
10
Achondroplasia does not affect the skin .
11 Achondroplasia
12
E is not correct. 2°/o chose this .
• 13
Achondroplasia does not affect smooth muscle .
• 14 Achondroplasia Muscle
• 15
• 16
Bottom Line:
• 17

• 18
Achondroplasia resu lts from impaired carti lage matu ration caused by an autosomal-dominant mutation in the
FGFR gene .
• 19 Achondroplasia Gene Mutation Dominance (genetics) Genetic disorder Cartilage Fibroblast growth factor receptor
• 20
• 21

6
lock
s
Suspend
0
End Block
Item: 12of25 ~. , . M k <:] t> al ~· ~
QIO: 4425 .l. ar Previous Next lab 'lifllues Notes Calculator

1 • •

2
FA11 p435.1
3
Achondroplasia Failure of longitudinal bone growth (endochondral ossification) -+ short limbs. Ylembranous
4
ossification is affected -+ large head relative to limbs. Constitutive acti,·ation of fibroblast growth
5 factor receptor (FGFR3) actually inhibits chondrocyte prol iferation.> 85% of mutations occur
6 sporadically; autosomal dominant with full penetrance (homozygosity is lethal). Most common
7 cause of dwarfism.
8
FA17p56.1
9
10 Autosomal dominant Achondroplasia, autosomal dominant polycystic kidney disease, fam ilial adenomatous polyposis,
diseases familial hypercholesterolemia, hereditary hemorrhagic telangiectasia, hereditary spherocytosis,
11
Huntington disease, Li-F'raumeni syndrome, Marfan syndrome, multiple endocrine neoplasias,
12
neurofibromatosis type I (,·on Recklinghausen disease), neurofibromatosis type 2, tuberous
• 13 sclerosis, von 1-1 ippei-Lindau disease.
• 14
• 15 FA11 p434.2
• 16 Bone formation
• 17 Endochondral Bones of axial skeleton, appendicular skeleton, and base of skulL Cartilaginous model of bone is
• 18 ossification first made by chondrocytes. Osteoclasts and osteoblasts later replace with woven bone and then
• 19 remodel to lamellar bone. In adults, woven bone occurs after fractures and in Paget disease.
Defective in achondroplasia.
• 20
Membranous Bones of calvarium, facial bones, and clavicle. Woven bone formed directly without cartilage. Later
• 21 •
• nccifir::otinn rP•nAf1p)prJ tA l'lmPIJ·Jr hAnP

6
lock
s
Suspend
0
End Block
Item: 12 of 25 ~ 1 • M k -<:J 1>- Jil ~· !:';-~
QIO: 4425 ..L ar Pre v ious Next Lab fli!ltues Not es Calcula t o r
&
• &
1

2
Achondroplasia Failure of longitudinal bone growth (endochondral ossification) -+ short limbs. ~cmbranous
ossification is affected - large head relative to Iimbs. Constih1tive acti,·ation of fibroblast growth
3
factor receptor (FCFR3) actually inhibits chondrocyte proliferation.> 85% of mutations occm
4 sporadically; autosomal dominant with full penetrance (homozygosity is lethal). lost common
5 cause of dwarfism.
6
7 FA17 p 56.1

8 Autosomal dominant Achondroplasia, autosomal dominant polycystic kidne} disease, familial adenomatous polyposis,
9 diseases familial hypercholesterolemia, hereditaq hemorrhagic telangiectasia, hereditar~ spherOC) tosis,
10
Huntington disease, Li-F'raumeni S) ndrome, \ larfan syndrome, multiple endocrine neoplasias,
neurofibromatosis type I (von Recklinghausen disease), neurofibromatosis type 2, tuberous
11
sclerosis, von IIippel-Lindau disease.
12
• 13
FA1 7 p 434.2
• 14 Bone formation
• 15 Endochondral Bones of axial skeleton, appendicular skeleton, and base of skull. Cartilaginous model of bone is
• 16 ossification first made by chondrocytes. Osteoclasts and osteoblasts later replace with woven bone and then
• 17 remodel to lamellar bone. In adults, woven bone occurs after fractures and in Paget disease.
• 18 Defective in achond roplasia .
• 19 Membranous Bones of calvarium, facial bones, and clavicle. Woven bone formed directly without cartilage. Later
ossificat ion remodeled to lamellar bone.
• 20
• 21

a
Lock
s
Suspend
8
End Bl ock
Item: 13 of 25 ~ 1 • M k -<:J 1>- Jil ~· !:';-~
QIO: 3 0 93 ..L ar Pre v ious Next Lab fli!ltues Notes Calcula t o r

IAA]
& &
1
A 45-year-old man with type 2 diabetes mellitus presents to his physician for regular follow-up. The patient
2
den ies any changes in v ision, chest pain, difficu lty w ith ga it, or injuries to the feet. There are no carotid
3 bruits on examination, and funduscopic examination is unremarkable. On neurologic examination, the patient
4 has an abnormal tun ing fork examination, w ith absence of vibratory sensation in the feet bilaterally.
5
6 Which of the following receptors is most likely affected in this patient?
7 :

8 A. Krause end bulbs


9 B. Meissner corpuscle
10
C. Merkel nerve endings
11
D. Pacinian corpuscle
12
• 13 E. Ruffini corpuscle
• 14
• 15
• 16
• 17
• 18
• 19
• 20
• 21

a
Lock
s
Suspend
8
End Bl ock
Item: 13 of 25 ~ 1 • M k -<:J 1>- Jil ~· !:';-~
QIO: 3 0 93 ..L ar Pre v ious Next Lab fli!ltues Notes Calcula t o r
A A
1
2
The correct a n sw e r is D. 60°/o chose this.
3
The most common initial sign of peripheral neuropathy in patients with diabetes is a loss
of vibrational sensation. This f inding is most clearly demonstrated on physica l
4 examination by placing a v ibrating tun ing fork on the big toe. The sensory receptors
5 responsible for transducing the sensation of vibration, pressure, and tension are the
6 large, encapsu lated pacinian corpuscles, which are located in the deeper layers of the
skin, ligaments, and joint capsules. They can be distinguished histologically by their
7
onion-like appearance on cross-section (as shown in the diagram) . It is important to
8 tailor the physical examination in a diabetic patient to include testing for the major
9 complications of diabetes: retinopathy, peripheral neuropathy, and vascular disease .
10
Routine funduscopic examination and ophthalmologic evaluation are necessary to
monitor for the development of diabetic retinopathy. A thorough neurologic examination
11 of the distal extremities including monofilament testing and vibration sense testing are
12 key to evaluate for periphera l neuropathy. Moreover, routine examination of the feet is
13 necessary for both patient and physician since neuropathy puts patients at risk for
developing ulcers and infections. Fina lly, listen ing for carotid bruits can play a critical
• 14
ro le in assessing the status of the vasculature because patients are at increased risk for
• 15 coronary artery disease and stroke .
• 16 Lamellar corpuscle Peripheral neuropathy Diabetic retinopathy Ophthalmoscopy Tuning fork Diabetes mellitus coronary artery disease Histology Ophthalmology

• 17 Circulatory system Stroke Neurological examination Sensory receptor Physical examination Blood vessel Physician Retinopathy Neurology

• 18 Anatomical terms of location Toe

• 19
A is not correct. 2 °/o ch ose this .
• 20
Krause end bulbs are sensory receptors found in the oropharynx and conjunctiva of the eye. These structures
• 21 are involved in sensing cold temperatures .

a
Lock
s
Suspend
8
End Bl ock
Item: 13 of 25 ~ 1 • M k -<:J 1>- Jil ~· !:';-~
QIO: 3 0 93 ..L ar Pre v ious Next Lab fli!ltues Notes Calcula t o r
A A
1
2
The correct a n sw e r is D. 60°/o chose this.
3
The most common initial sign of peripheral neuropathy in patients with diabetes is a loss
of vibrational sensation. This f inding is most clearly demonstrated on physica l
4 examination by placing a v ibrating tun ing fork on the big toe. The sensory receptors
5 responsible for transducing the sensation of vibration, pressure, and tension are the
6 large, encapsu lated pacinian corpuscles, which are located in the deeper layers of the
skin, ligaments, and joint capsules. They can be distinguished histologically by their
7
onion-like appearance on cross-section (as shown in the diagram) . It is important to
8 tailor the physical examination in a diabetic patient to include testing for the major
9 complications of diabetes: retinopathy, peripheral neuropathy, and vascular disease .
10
Routine funduscopic examination and ophthalmologic evaluation are necessary to
monitor for the development of diabetic retinopathy. A thorough neurologic examination
11 of the distal extremities including monofilament testing and vibration sense testing are
12 key to evaluate for periphera l neuropathy. Moreover, routine examination of the feet is
13 necessary for both patient and physician since neuropathy puts patients at risk for
developing ulcers and infections. Fina lly, listen ing for carotid bruits can play a critical
• 14
ro le in assessing the status of the vasculature because patients are at increased risk for
• 15 coronary artery disease and stroke .
• 16 Lamellar corpuscle Peripheral neuropathy Diabetic retinopathy Ophthalmoscopy Tuning fork Diabetes mellitus coronary artery disease Histology Ophthalmology

• 17 Circulatory system Stroke Neurological examination Sensory receptor Physical examination Blood vessel Physician Retinopathy Neurology

• 18 Anatomical terms of location Toe

• 19
A is not correct. 2 °/o ch ose this .
• 20
Krause end bulbs are sensory receptors found in the oropharynx and conjunctiva of the eye. These structures
• 21 are involved in sensing cold temperatures .

a
Lock
s
Suspend
8
End Bl ock
Item: 13of25 ~. , . M k <:] t> al ~· ~
QIO: 3093 .l. ar Previous Next lab 'lifllues Notes Calculator

1 • •
A is not co rrect. 2°/o chose this.
2 Krause end bulbs are sensory receptors found in the oropharynx and conjunctiva of the eye. These struct ures
3 are involved in sensing cold temperatu res.
Conjunctiva Pharynx Bulboid corpuscle Sensory receptor
4
5 B is not correct. 19% chose this.
6 Meissner corpuscles, wh ich are responsible for conveying t he sensat ion of light touch, are sma ll encapsulat ed
7 sensory recept ors found just beneath t he der mis of ha irless skin, most prominently in the fingertips, soles of the
feet, and lips . Meissner corpuscles are involved in the recept ion of light discriminatory touch, not v ibrat ory
8
sensation, as is being tested in t his case.
9 Dermis Cutaneous receptor Tactile corpuscle

10
C is not correct. 10% chose this.
11
Merkel nerve end ings are nonencapsulated and found in al l skin types (both hairy and ha irless) and, along with
12 Meissner corpuscles, are bel ieved to be responsible for discriminatory touch .
13 Tactile corpuscle Merkel nerve ending Nerve

• 14 E is not correct. 9°/o chose this .


• 15 Ruffini cor puscles are spindle-shaped, encapsu lated mechanoreceptors that are found in t he soles of t he feet
• 16 and are responsible for transducing pressu re .
Bulbous corpuscle Mechanoreceptor
• 17

• 18
• 19 Bottom Line:
• 20 Diabet ic neu ropat hy can manifest as a loss of vibratory sense, a sensation that involves pacin ian corpuscles.
• 21 lamellar corpuscle Peripheral neuropathy Diabetic neuropathy Cutaneous receptor
• •

6
lock
s
Suspend
0
End Block
Item: 13of25 ~. , . M k <:] t> al ~· ~
QIO: 3093 .l. ar Previous Next lab 'lifllues Notes Calculator

1
FA17 p464.4
2
Sensory receptors
3 RECEPTORTYPE SENSORYNEURONFIBERTYPE LOCATION SENSES
4 Free nerve endings C-slow, unmyelinated fibers II skin, epidermis, some Pain, temperature
5 AS- fast, myelinated fibers VISCera

6 Meissner corpuscles Large, myelinated fibers; adapt Glabrous (hairless) skin Dynamic, line/light touch,
7 quickly position sense
8 Pacinian corpuscles Large, myelinated fibers; adapt Deep skin layers, ligaments, Vibration, pressure
9 qu ickly joints
10 Merkel discs Large, myelinated fibers; adapt Finger tips, superficial skin Pressure, deep static touch (eg,
11
slowly shapes, edges), position sense
12
Ruffini corpuscles Dendritic endings " ·ith Finger tips, joints Pressure, slippage of objects
capsule; adapt slowly along surface of skin, joint t
13
angle change
• 14
• 15
FA17 p 336.1
• 16
Diabetes mellitus
• 17
ACUTE MANIFESTATIONS Polydipsia, polyuria, polyphagia, weight loss, DKA (type 1}, hyperosmolar coma (type 2).
• 18
Rarely, can be caused by unopposed secretion of GH and epi neph rine. Also seen in patients on
• 19
glucocorticoid therapy (steroid diabetes).
• 20
CHRONICCOMPLICATIONS onenzymatic glycation:
• 21 • Small vessel disease (diffuse thickening of basement membrane) - retinopathy (hemorrhage,

6
lock
s
Suspend
0
End Block
Item: 13 of 25 ~ 1 • M k -<:J 1>- Jil ~· !:';-~
QIO: 3 0 93 ..L ar Pre v ious Next Lab fli!ltues Notes Calcula t o r
A A
1
2 FA11 p336.1
Diabetes mellitus
3
4
ACUTE MANIFESTAllONS Polydipsia, polyuria, pol~ phagia, "eight loss, DKA (I) pe 1), hyperosmolar coma (type 2).
Rarely, can be caused by unopposed secretion of G i l and epinephrine. Also seen in patients on
5
glucocorticoid therapy (steroid diabetes).
6
CHRONIC COMPLICAIIONS 1onenzrmatic glycation:
7
• Small ,·essel disease (diffuse thickening of basement membrane) - retinopathy (hemorrhage.
8 exudates, microaneurysms, 'esscl proliferation), glaucoma, neuropathy, nephropathy (nodular
9 glomerulosclerosis, aka Kimmelstiel-\Vilson nodules - progressive proteinuria [initially
10 microalbuminuria; ACE inhibitors are renoprotecti,·e] and arteriolosclerosis - hypertension;
both lead to chronic renal fai lure).
11
Large ''essel atherosclerosis, C D, peripheral \'ascttlar occlusive disease, gangrene - limb loss,
12
cerebrovascular disease. i\ II most common cause of death .
13 Osmotic damage (sorbitol accumulation in o rg;~ns with aldose reductase and ! or absent sorbitol
• 14 dehydrogenase):
• 15 • 'europathy (motor, sensory [glove and stocking distribution], and autonomic degeneration)
Cataracts
• 16
DIAGNOSIS TEST DIAGNOSTIC CUTOFF NOTES
• 17
llbA 1c ~6.5% Reflects average blood glucose
• 18
over prior 3 months
• 19 Fasting plasma glucose ~ 126 mg/dL Fasting for> 8 hours
• 20 2-hour oral glucose tolerance test ~ 200 mg/d L 2 hours after consumption of 75 g
• 21 of glucose in water

a
Lock
s
Suspend
8
End Bl ock
Item: 13 of 25 ~ 1 • Ma rk -<:J I> ~ £!1}>'
• !!":-~
QIO: 3 0 93 ..L Prev ious Next Lab lues Notes Cal culat o r

A A
1 of glucose in \\'atcr
2
Insulin deficiency or severe insulin insensitivity
3
4
J. tissue glucose t glycogenolySis t glucooeogenes~s t proteolysis t l•polys.s
uptake
5
6
7
Hyperglycenua.
glycosuna
l J. protem. weight loss t plasma
free fatty acids

8
{
9
10
t plasma osmolality Osmotic diuresiS

Vom1ting
t ketogenes•s.
11 ketonemia. ketonuna
12 Loss of water.
t th1rst Na·. and K'
13 Hyperventilation/ Anion gap
Kussmaul respiration metabolic ac•dosis
• 14
Hypovolemia
• 15
• 16
Circulation failure.
• 17 .J. bssue perfusion t serum lactate
• 18
• 19 Coma/death
• 20
• 21

a
Lock
s
Suspend
8
End Bl ock
Item: 14 of 25 ~ 1 • M k -<:J 1>- Jil ~· !:';-~
QIO: 2559 ..L ar Pre v ious Next Lab fli!ltues Notes Calcula t o r
A A
5
A 60-year-old woman presents to her primary ca re physician w ith local ized burning pa in and vesicula r skin
6 lesions localized to a single, stark band across her torso, like that shown in the image. This infected area is
7 only observed on her right side and does not cross t he midline.
----~
8
9
10
11
12
13
• 14
• 15
• 16
• 17 I mage courtesy of Wikimedia Commons
• 18
• 19 If researchers were to develop an intravenous medication to cure the underlying cause of this disease, what
• 20 structure wou ld likely present the greatest obstacle to drug penetration?
• 21 :
• 22 A . Blood-brain barrier
• 23 B. Endoneurium
• 24
C. Epineurium
• 25

a
Lock
s
Suspend
8
End Bl ock
Item: 14 of 25 ~ 1 • M k -<:J 1>- Jil ~· !:';-~
QIO: 2559 ..L ar Pre v ious Next Lab fli!ltues Notes Calcula t o r
I •e• - • - 11 - I · - -111 - I I - I -
5
6
7
8
9
10
11
12
13
• 14
• 15 Image courtesy of Wikimedia Commons

• 16
• 17
If researchers were to develop an intravenous medication to cure the underlying cause of th is disease, what
structure would likely present the greatest obstacle to drug penetration?
• 18
:
• 19
A. Blood-brain barrier
• 20
• 21 B. Endoneurium
• 22 C. Epineurium
• 23
D. Perineurium
• 24
• 25

a
Lock
s
Suspend
8
End Bl ock
Item: 14 of 25 ~. I • M k <:] t> al ~· ~
QIO: 2559 .l. ar Previous Next lab 'lifllues Notes Calculator

5
The correct answer is D. 33°/o chose this.
6 This patient presents with herpes zoster, otherwise known as shing les, which is caused by var icella -zoster virus .
7 The virus infects sensory nerves and often on ly one dermatome is affected . The perineu r ium, which surro unds
8 fascicles of nerve fibers, serves as the per meabi lity barrier in the nervous system.
Dermatome (anatomy) Shingles Perineurium Somite Varicella zoster virus Nervous system Herpes simplex Sensory neuron Nerve fascicle Sensory nerve Virus
9
10 A is not correct. 23% chose this.
11 The blood -brain barrier, formed by a complex of endothelia l cel ls, pericytes, and astrocyte foot processes, also
presents a barrier that keeps intravenous med ication from reaching the nervous system . However, herpes
12
zoster does not invo lve the centra l nervous system-it infects dorsa l nerve roots in the periphery .
13 Astrocyte Shingles Blood-brain barrier Nervous system Intravenous therapy Endothelium Pericyte Herpes simplex Pharmaceutical drug

14
B is not correct. 21% chose this .
• 15
Peripheral nerve fascicles contain both aff erent and efferent nerve f ibers carrying sensory and motor
• 16 infor mation, respective ly . Each nerve fi ber is surrounded by an endoneurium. The endoneurium does not
• 17 provide a per meabi lity barrier for the nerve. It is the site of inflammaotry infi ltrate in Guillain-Ba r re syndrome,
• 18
for example .
Endoneurium Efferent nerve fiber Nerve fiber Nerve fascicle Afferent nerve fiber Muscle fascicle Nerve Peripheral nervous system
• 19
• 20 C is not correct. 23% chose this .
• 21 The epineu rium is a layer of dense connective t issue that surrounds al l the nerve fascicles, the per ineurium, and
the blood vesse ls. It functions to bind the f ascicles together. I t is not a permeability barrier, but structural in
• 22
nature .
• 23 Epineurium Perineurium Muscle fascicle Nerve fascicle Connective tissue Dense connective tissue

• 24
• 25
ft-&.JI.--- I=--·
6
lock
s
Suspend
0
End Block
Item: 14 of 25 ~. I • M k <:] t> al ~· ~
QIO: 2559 .l. ar Previous Next lab 'lifllues Notes Calculator

5 fascicles of nerve fibers, serves as the per meability ba r rier in the nervous system .
Dermatome (anatomy) Shingles Perineurium Somite Varicella zoster virus Nervous system Herpes simplex Sensory neuron Nerve fascicle Sensory nerve Virus
6
7 A is not correct. 23% chose this.
8 The blood -brain barrier, formed by a complex of endothel ial cells, pericytes, and astrocyte foot processes, also
presents a barrier that keeps int ravenous medication from reaching the nervous system . However, herpes
9
zost er does not invo lve the cent ra l nervous system -it infects dorsa l nerve roots in the periphery .
10 Astrocyte Shingles Blood-brain barrier Nervous system Intravenous therapy Endothelium Pericyte Herpes simplex Pharmaceutical drug

11
B is not correct. 21% chose this .
12
Peripheral nerve fascicles cont ain bot h afferent and efferent nerve f ibers ca r rying sensory and mot or
13 infor mation, respective ly . Each nerve fiber is surrounded by an endoneu rium. The endoneu rium does not
14 provide a per meability ba r rier for t he nerve. It is the site of inflammaotry infilt rate in Guillain -Ba r re syndrome,
• 15
for example .
Endoneurium Efferent nerve fiber Nerve fiber Nerve fascicle Afferent nerve fiber Muscle fascicle Nerve Peripheral nervous system
• 16
• 17 C is not correct . 23 % chose this .
• 18
The epineu rium is a layer of dense connective t issue that surrounds all the nerve fascicles, t he per ineu rium, and
the blood vesse ls. It functions to bind the fascicles t oget her. I t is not a permeability barrier, but structura l in
• 19
natu re .
• 20 Epineurium Perineurium Muscle fascicle Nerve fascicle Connective tissue Dense connective tissue

• 21
• 22
Bottom Line:
• 23
The perineurium serves as the per meabi lit y barrier in the nervous syst em .
• 24
Perineurium Nervous system
• 25

6
lock
s
Suspend
0
End Block
Item: 14 of 25 ~. I • M k <:] t> al ~· ~
QIO: 2559 .l. ar Previous Next lab 'lifllues Notes Calculator

• •
5
FA17 p 160.2
6
7 Herpesviruses Enveloped, OS, and li near viruses
8 VIRUS ROUTE OF TRANSMISSION CLINICAL SIGNIFICANCE NOTES

9 Herpes Respiratory Gingivostomatitis, keratoconjunctivitis fJ, lost common cause of sporadic


10
simplex secretions, saliva herpes labial is ll), herpetic whitlow on finger, encephalitis, can present as altered
virus-1 temporal lobe encephalitis, esophagitis, mental status, seizu res, and/or
11
erythema multiforme. aphasia.
12
Herpes Sexual contact, Herpes genitali s ~' neonatal herpes. Latent in sacral ganglia. Viral
13 simplex perinatal meningitis more common with
14 virus-2 HSV-2 than with HSV-L
• 15 Varicella- Respiratory Varicella-zoster (chickenpox [!], shingles 0 ), Latent in dorsal root or trigeminal
• 16 Zoster virus secretions encephalitis, pneumonia. ganglia; C V1 branch
• 17 (HHV-3) Most common complication of shingles is post- involvement can cause herpes
herpetic neuralgia . zoster ophthalmicus.
• 18
• 19 Epstein-Barr Respiratory Mononucleosis- fever, hepatosplenomegaly, Infects B cells through CD2L
virus (HHV-4) secretions, pharyngitis, <mel lymphadenopathy (especially Atypical lymphocytes on peripheral
• 20
saliva· aka posterior cervical nodes 1)). Avoid contact sports blood smear ~ - not infected B
• 21 '
"kissing disease," until resolution due to risk of splenic rupture. cells but reactive cytotoxic T cells.
• 22 (common in Associated with lymphomas (eg, endemic <±) fonospot test-heterophile
• 23 teens, young Burkitt lymphoma), nasopharyngeal antibodies detected by agglutination
• 24
adults) carcinoma (especially Asian adults), of sheep or horse RBCs.
lymphoproliferative disease in transplant Use of amoxicillin in mononucleosis
• 25 • t·" \a .. iP nt-c r-a n l ' '"l11 CP ,..h'"lr"lf"'horlct-i,.... •

6
lock
s
Suspend
0
End Block
Item: 14 of 25 ~ 1 • M k -<:J 1>- Jil ~· !:';-~
QIO: 2559 ..L ar Pre v ious Next Lab fli!ltues Notes Calcula t o r
A A
5
Epstein-Barr Respiratory Mononucleosis-fe,·er, hepatosplenomegaly, Infects B cells through C02 I.
6 virus (HHV-4) secretions, ph<uyngitis, and lymphadenopathy (especially Atypical lymphocytes on peripheral
7 sali,·a· aka posterior cervical nodes ). Avoid contact sports blood smear ~ -not infected B
'
8
"kissing disease," until resolution due to risk of splenic rupture. cells but reactive cytotoxic T cells.
(common in Associated with lymphomas (eg, endemic G) ~ fonospot test-heterophile
9
teens, young Burkitt lymphoma), nasopharyngeal antibodies detected by agglutination
10 adults) carcinoma (especiall) Asian adults), of sheep or horse RBCs.
11 lpnphoproliferati,·e disease in transplant Use of amoxicillin in mononucleosis
12 patients. can cause characteristic
maculopapular rash.
13
14
Cytomegalo- Congenital Mononucleosis (8 l\ lonospot) in Infected cells have characteristic
virus (HHV-5) transfusion, immunocompetent patients; infection in "owl eye" inclusions Cl
• 15
sexual contact, immunocompromised, especiall) pneumonia Latent in mononuclear cells.
• 16 sali,·a, urine, in transplant patients; esophagitis; AIOS
• 17 transplant retinitis ("~igh t omega l ovirus"): hemorrhage,
• 18 cotton-wool exudates, vision loss .
• 19
Congenital C IV
• 20 Human Saliva Roseola infantum (exanthem subitum): high Ro~cola: fever first, Ro~ie (checks)
herpes- fevers for several days that can cause seizures, later.
• 21
viruses 6 followed by diffuse macular rash 0 . HH V-7-less common cause of
• 22 and 7 roseola.
• 23
Human Sexua I contact Kaposi sarcoma (neoplasm of endothelial cells). Can also affect Gl tract and lungs.
• 24 herpesvirus Seen in HTV/. IDS and transplant patients.
• 25 8 Dark/,·iolaceous plaques or nodules D

a
Lock
s
Suspend
8
End Bl ock
Item: 14 of 25 ~ 1 • M k -<:J 1>- Jil ~· !:';-~
QIO: 2559 ..L ar Pre v ious Next Lab fli!ltues Notes Calcula t o r
I !~
5
Human Saliva Roseola infantum (exanthem subitum): high Rmeola: fever first, Rmie (checks)
6
herpes- fevers for several days that can cause seizures, later.
7 viruses 6 followed by diffuse macular rash HHV-7-less common cause of
8 and 7 roseola.
9 Human Sexual contact Kaposi sarcoma (neoplasm of endothelial cells). Can also affect Gl tract and lungs.
10 herpesvirus Seen in HIV/AIDS and transplant patients.
11
8 Dark/violaceous plaques or nodules D
representing vascular proliferations.
12
13
14
• 15
• 16
• 17
• 18
• 19
• 20
• 21
• 22
• 23
• 24
• 25

a
Lock
s
Suspend
8
End Bl ock
Item: 14 of 25 ~. I • M k <:] t> al ~· ~
QIO: 2559 .l. ar Previous Next lab 'lifllues Notes Calculator


5
6 FA17 p447.2

7
Dermatologic macroscopic terms (morphology)
LESION CHARACTERISTICS EXAMPLES
8
Macule Flat lesion with well-circumscribed change in Freckle, labial macule fJ
9
skin color< I em
10 Patch Macule> 1 em Large birthmark (congenital nevus) 111
11 Papule Elevated solid skin lesion < I em Mole (nevus) [!1, acne
12 Plaque Papule> l em Psoriasis [!)
13 Vesicle Small fluid-containing blister< I em Chickenpox (varicella), shingles (zoster) 0
14 Bulla Large fluid -contain ing blister> I em Bullous pemphigoid D
• 15 Pustule Vesicle containing pus Pustular psoriasis [!!
• 16 Wheal Transient smooth papule or plaque Hives (urticaria) CJ
• 17 Scale Flaking off of stratum corneum Eczema, psoriasis, SCC D
• 18 Crust Dry exudate Impetigo D
-....:;;;:;:::.~;;::=:::::::::::===::::::::::::::== ;;::::::;;~;;;::::::::::::==========;
• 19
• 20
• 21
• 22
• 23
• 24
• 25 ~

6
lock
s
Suspend
0
End Block
Item: 14 of 25 ~ 1 • M k -<:J 1>- Jil ~· !:';-~
QIO: 2559 ..L ar Pre v ious Next Lab fli!ltues Notes Calcula t o r

5
A
Patch lacule > l em Large birthmark (congenital nevus) [lJ A

6
Papule Eb·ated solid skin lesion < I em 1ole (nevus) m.acne
Plaque Papule> I em Psoriasis [!]
7
Vesicle Small fluid-containing blister< I em C hickenpox (varicella), shingles (zoster)
8
Bulla Large fluid-containing blister > I em Bullous pemphigoid D
9
Pustule Vesicle containing pus Pustular psoriasis [!I
10
Wheal Transient smooth papule or plaque lli,·es (urticaria) C1
11
Scale Flaking off of stratum corneum Eczema, psoriasis, SCC
12
Crust Dry exudate Impetigo D
'
13
14
• 15
• 16
• 17
• 18
• 19
• 20
• 21
• 22
• 23
• 24
• 25

a
Lock
s
Suspend
8
End Bl ock
Item: 14 of 25 ~. I • M k <:] t> al ~· ~
QIO: 2559 .l. ar Previous Next lab 'lifllues Notes Calculator

• •
5 FA17 p451 .1
6 Skin infections
7 Bacterial infections
8 Impetigo Very superficial skin infection. Usually from S aureus or S pyogenes. Highly contagious. Honey-
9 colored crusting m.
10 rn
Bullous impetigo has bullae and is usually caused by S aureus.
11 Erysipe las Infection involving upper dermis and superficial lymphatics, usually from S pyogenes. Presents with
12 well-defined demarcation between infected and normal skin
13 Cellulit is Acute, painful, spreading infection of deeper dermis and subcutaneous tissues. Usually from
14
S pyogenes or S aureus. Often starts with a break in skin from trauma or another infection [!].
• 15 Abscess Collection of pus from a walled-off infection within deeper layers of skin 0 . Offending organism is
almost a lways S aureus .
• 16
• 17
Necrotizing fasciitis Deeper tissue injury, usually fro m anaerobic bacteria or S pyogenes. Pain may be out of proportion
to exam findings. Results in crepitus from methane and C02 production. "Flesh-eating bacteria."
• 18
Causes bullae and a purple color to the skin D .
• 19
Staphylococcal scalded Exotoxin destroys keratinocyte attachments in stratum granulosum only (vs toxic epidermal
• 20 skin syndrome necrolysis, " ·hich destroys epidermal-dermal junction). Characterized by fever and general ized
• 21 m
erythematous rash with sloughing of the upper layers of the epidermis that heals completely.
• 22 EE> 1 ikolsky sign. Seen in newborns and children, adults with renal insufficiency.
• 23 Viral infections
• 24 He rpes Herpes ,·irus infections (HSVl and HSV2) of skin can occur anywhere from mucosa l surfaces to
• 25 normal skin. These include herpes labial is, herpes genita lis, herpetic whitlow CJ (fi nger). •

6
lock
s
Suspend
0
End Block
Item: 14 of 25 ~ 1 • M k -<:J 1>- Jil ~· !:';-~
QIO: 2559 ..L ar Pre v ious Next Lab fli!ltues Notes Calcula t o r
A A
5 Molluscum Umbilicated papules caused by a poxvirus. While frequently seen in children, it may be sexually
6 contagiosum transmitted in adults.
7 Varicella zoster virus Causes \·aricella (chickenpox) and 70ster (shingles). Varicella presents " ·ith multiple crops of
8 lesions in various stages from vesicles to crusts. Zoster is a reacti,·ation of the ,-irus in dermatomal
distribution (unless it is disseminated).
9
10
Hairy leukoplakia Irregular, white, painless plaques on lateral tongue that cannot be scraped off D. EBV mediated.
Occurs in lliV-positi\e patients, orga n transplant recipients. Contrast with thrush (scrapable) and
11
leukoplakia (precancerous).
12
13
14
• 15
• 16
• 17
• 18
• 19
• 20
• 21
• 22

I
• 23
• 24
• 25

a
Lock
s
Suspend
8
End Bl ock
Item: 15 of 25 ~ 1 • M k -<:J 1>- Jil ~· !:';-~
QIO: 2596 ..L ar Pre v ious Next Lab fli!ltues Notes Calcula t o r

IAA]
A A
5
A 42-year-old African-American woman with a history of sarcoidosis presents to a neurologist with a sudden
6 onset of a unilateral inability to close t he eye and decreased tearing. When she wrinkles her forehead or
7 sm iles, the affected side of her face remains relaxed.
8
9 The muscles involved in this condition are derived from wh ich of the following embryolog ic structures?
10 :
11 A . First branchial (pharyngeal) arch
12 B. Second branchial (pharyngeal) arch
13
C. Third branchial (pharyngeal ) pouch
14
• 15
D. Fourth branchial (pharyngeal) pouch
• 16 E. Thyrog lossa l duct
• 17
• 18
• 19
• 20
• 21
• 22
• 23
• 24
• 25

a
Lock
s
Suspend
8
End Bl ock
Item: 15of25 ~. , . M k <:] t> al ~· ~
QIO: 2596 .l. ar Previous Next Lab 'lifllues Notes Calculator


5
6 The correct answer is B. 63°/o chose this.
7 A Bel l pa lsy is a lesion of crania l nerve VII (facial nerve) and affects the muscles of facia l expression. These
muscles are derived from the second branch ial pharyngeal arch . Pat ients with sarcoidosis, tumors, diabetes,
8
AIDS, and Lyme disease are at increased risk for Bel l palsy, although most cases are idiopath ic. Important signs
9 of Bel l palsy are ptosis and facial droop . The second arch also gives r ise to t he posterior bel ly of the digastric,
10 the stylohyo id, and the st aped ius muscles.
Pharyngeal arch Lyme disease Sarcoidosis Bell' s palsy Facial nerve Cranial nerves Stapedius muscle Facial muscles Ptosis (eyelid) Idiopathy Stylohyoid muscle
11
Digastric muscle lesion Skull HIV/AIDS Pharynx Diabetes mellitus Facial expression Neoplasm
12
13 A is not correct. 17% chose this.
14 The fi rst branchia l arch gives rise t o the muscles of mastication, the mylohyoid, the ant erior belly of the
15 digastr ic, the tensor veli pa latin i, and the tensor tympan i. These muscles are innervated by crania l nerve V and
are not affected by Bel l pa lsy .
• 16
Tensor veli palatini muscle Branchial arch Mastication Muscles of mastication Tensor tympani muscle Cranial nerves Bell' s palsy Trigeminal nerve Pharyngeal arch
• 17
Mylohyoid muscle Digastric muscle
• 18
• 19
C is not correct. 12% chose this .
The th ird branch ial pouch gives r ise to t he inferior parathyroid glands and the thymus. It is impl icated in
• 20
DiGeorge syndrome but has no relation to Bell palsy .
• 21 Pharyngeal pouch (embryology) Thymus DiGeorge syndrome Bell' s palsy Parathyroid gland

• 22
D is not correct. 6°/o chose this .
• 23
The fourth branch ial pouch gives rise t o the superior parathyroid glands. It is impl icated in DiGeorge synd rome
• 24 but is not related to Bel l palsy .
• 25 Pharyngeal pouch (embryology) DiGeorge syndrome Bell' s palsy Parathyroid gland
~

6
lock
s
Suspend
0
End Block
Item: 15of25 ~. , . M k <:] t> al ~· ~
QIO: 2596 .l. ar Previous Next lab 'lifllues Notes Calculator

5
• T he fi rst branchi al arch gives rise t o the muscles of mastication, the mylohyoid, the ant erior belly of the
digastr ic, the tensor veli pa latin i, and the tensor tympan i. These muscles are innervated by crania l nerve V and
6
are not affected by Bell pa lsy.
7 Tensor veli palatini muscle Branchial arch Mastication Muscles of mastication Tensor tympani muscle Cranial nerves Bell' s palsy Trigeminal nerve Pharyngeal arch

8 Mylohyoid muscle Digastric muscle

9
C is not correct. 12% chose this.
10
The th ird branch ial pouch gives r ise to t he inferior parathyroid glands and the thymus. It is impl icated in
11 DiGeorge syndrome but has no relation to Bell pa lsy.
12 Pharyngeal pouch (embryology) Thymus DiGeorge syndrome Bell' s palsy Parathyroid gland

13 D is not correct. 6°/o chose this.


14 The fourth branch ial pouch gives rise t o the superior parathyroid glands. It is impl icated in DiGeorge synd rome
15 but is not related to Be ll pa lsy .
Pharyngeal pouch (embryology) DiGeorge syndrome Bell' s palsy Parathyroid gland
• 16
• 17 E is not correct. 2°/o chose this .
• 18 The thyrog lossa l duct connects the thyroid diverticulum to the foregut in the embryo but is obl iterated du r ing
• 19
development . Its on ly remnant in the adu lt is the foramen cecum .
Thyroglossal duct Cecum Embryo Thyroid Foramen cecum (tongue) Foramen cecum (frontal bone) Diverticulum
• 20
• 21
• 22 Bottom Line:
• 23 The muscles of facia l expression are derived from the second branch ial arch .
Branchial arch Facial muscles Pharyngeal arch Facial expression
• 24
• 25 ~

6
lock
s
Suspend
0
End Block
Item: 15of25 ~. , . M k <:] t> al ~· ~
QIO: 2596 .l. ar Previous Next lab 'lifllues Notes Calculator

5
6 FA17 p 587.1
7 Branchial arch derivatives
8 ARCH CARTILAGE MUSCLES NERVES' ABNORMALITIES/COMMENTS

9 1st arch Maxillary process Muscles of Mastication Cr V2 and V3 Pierre Robin sequence-
10
... Maxilla, zygoMatic (temporalis, Masseter, chew micrognathia,
bone lateral and 1edial glossoptosis, cleft
11
Mandibular process pterygoicls), Mylohyoid, palate, airway
12 -+ Meckel carti lage anterior bell)' of digastric, obstruction
13 ... Mandible, tensor tympani, anterior ltl Treacher Collins
14 Malleus and incus, of tongue syndrome- neural
15
spheno.Yiand ibular crest dysfunction
ligament ... mandibular
• 16
2nd arch Reichert cartilage: Muscles of facia l expression, C V ll (facial hypoplasia, facia l
• 17
Stapes, Styloid Stapedius, Stylohyoid, expression) abnormalities
• 18
process, lesser horn platySma, posterior belly smile
• 19 of hyoid, Stylohyoid of digastric
0 20 ligament
• 21 3rd arch Greater horn of hyoid Stylopharyngeus (think CN IX (stylo·
• 22 of stylopharyngeus pharyngeus)
• 23
innervated by swallow stylishly
glossopharyngeal nerve)
• 24
4th- 6th arches Arytenoids, C ricoid, 4th arch: most pharyngeal 4th arch: C Arches 3 and 4 form
• 25
r. I ' ... .. • 1 V I
I
, ., r.

6
lock
s
Suspend
0
End Block
Item: 15 of 25 ~ 1 • M k -<:J 1>- Jil ~· !:';-~
QIO: 2596 ..L ar Pre v ious Next Lab fli!ltues Notes Calcula t o r
A A
5
3rd arch Greater horn of hyoid Stylopharyngeus (think C IX (~tyl o­
6 of stylophan ngeus pharyngeus)
7 innervated bv \wa llo" ~tyl ishly
'
8 glossophar) ngeal nene)
9 4th- 6th arches \ rytenoids, C ricoid, 4th arch: most pharyngeal 4th arch: C rches 3 and 4 form
10 Corniculate, constrictors; erieoth\ roid, X (superior posterior Yl of tongue;
'
C uneiform, T hyroid b<1tor veli palatini laryngeal branch) arch 5 makes no
11
(used to sing and 6th arch: a 11 intrinsic ~imply ~wallow major developmental
12 \ CCCT) muscles of lar) nx e\cepl 6th arch: Ct contributions
13 cricot h, roid X (recurrent
'
14 laryngeal branch)
15 ~pea l..

• 16
3
These are the only C 1s with both motor and sensory components (except 2, which is sensory only).
• 17 \Vhen at the restaurant of the golden arches, children tend to firs t chc'' (I), I hen ~mi l e (2), then swallow st) Iish ly (3) or
• 18 ~ imply !>\\allow (4), and then speak (6).

• 19
• 20 FA17 p 502.2

• 21
Facial nerve lesions

• 22 Upper motor neuron Destruction of motor cortex or connection


lesion beb,·een motor cortex and facial nucleus in
• 23
pons - contralateral paralysis of lower muscles
• 24
of facial expression. Forehead is spared due to
• 25
• its bilateral U ~IN innervation. -
a
Lock
s
Suspend
8
End Bl ock
Item: 15 of 25 ~ 1 • M k -<:J 1>- Jil ~· !:';-~
QIO: 2596 ..L ar Pre v ious Next Lab fli!ltues Notes Calcula t o r
A A
5 When at the restaurant of the golden arches, children tend to first chC\\ (1), then ~mil e (2), then swallow st}lishly (3) or
6 ~ impl y wallo\\ (4), and then speak (6).

7
8 FA17 p 502.2
9 Facial nerve lesions
10 Upper motor neuron Destruction of motor cortex or connection

-
11
lesion between motor cortex and facial nucleus in r.....

12
pons - contralateral paralysis of IO\\er muscles "'""*"
of facial expression. Forehead is spared due to
13 its bilateral Ul\ IN innervation.
14
Lower motor neuron Destruction offacial nucleus or C II
15 lesion anywhere along its course - ipsilateral
• 16 paralysis of upper and lower muscles of
• 17 facial expression r.J, hypcracusis, loss of taste
sensation to anterior tongue.
• 18
When idiopathic (most common), fa cial nerve
• 19 CNVII
palsy is called Bell palsy. tvlay also be caused (LJIN lesion •
• 20 by Lyme disease, herpes simplex, herpes r..,.. ......
polly!
• 21 zoster (Ramsay Hunt syndrome), sa rcoidosis,
• 22 tumors (eg, parotid gland), diabetes mel litus.
• 23
Treatment is corticosteroids, ac)clO\ ir. Most
patients gradually recover function .
• 24
• 25

a
Lock
s
Suspend
8
End Bl ock
Item: 16 of 25 ~ 1 • M k -<:J 1>- Jil ~· !:';-~
QIO: 3174 ..L ar Pre v ious Next Labfli!llues Not es Calcula t o r

IAA]
A A
5
A 16-year-old girl is pregnant with her first ch ild and does not receive any prenatal care. When the baby is
6 born, a small sacral tuft of hair is noted, and ultrasound demonstrates an underlying defect in the dural sac.
7 Th is presentation may have been prevented by maternal supplementation w ith a particular nutrient.
8
9 Which of the following would be seen in an adult who is deficient in the same nutrient?
10 :
11 A. Dermatitis
12 B. Hypersegmented neutrophils
13
C. Night blindness
14
D. Numbness and ting ling in the extremities
15
• 16 E. Poor wound healing
• 17
• 18
• 19
• 20
• 21
• 22
• 23
• 24
• 25

a
Lock
s
Suspend
8
End Bl ock
Item: 16of25 ~. , . M k <:] t> al ~· ~
QIO: 3174 .l. ar Previous Next Lab 'lifllues Notes Calculator


5
6 The correct answer is B. 69°/o chose this.
7 I n embryogenesis the neura l folds beg in to draw together over the top of the
8 neura l groove to form the neu ral tube. Closure proceeds rostra lly and caudally
0
e
9 from near the midpoint of the ea rly emb ryo; therefore the rostral and cauda l •
ends are the most frequent locations fo r fai lure of fusion, resu lting in spina 0 ~00
10 bifida (diagnosed by the tuft of hai r and underlying du ral defect) . Although no 0 ..;~:\ n. •
Q \'1'1 v Og
g o0
11

12
single cause has been determined, prenatal supplementation with fo lic acid
can reduce neural tube defects by up to 80% . Fol ic acid deficiency in the adu lt
is rare, but may be seen in malnou r ished patients such as those with QQ
a
f ....
~
0

0
OD
0
0
0
•~ ~~ e ee
13
excessive alcoho l intake. Folate deficiency leads to megaloblastic anemia, and d
14 the blood smear will show hypersegmented neutroph ils, as in the image here. =----=--"'alL.~---------.;::_-_._,
15 Folate deficiency Spina bifida Megaloblastic anemia Folic acid Neural groove Embryogenesis Anemia Embryo Image copynght © 2008
Acharya eta/; licensee BioMed
16 Human embryogenesis Neural tube defect Neutrophil Neural tube Neural fold Dura mater
Central Ltd.
• 17
• 18 A is not correct. 5°/o chose this .
• 19 De rmatitis, Diarr hea, and Dementia (the three D's) are the major symptoms of pe llag ra, or vitamin B3 (niacin)
• 20
deficiency. Classica lly, people who subsisted primarily on corn deve loped pe llagra due to poor niacin intake .
Pellagra Niacin Vitamin 812 B vitamins Diarrhea Vitamin Maize
• 21
• 22 C is not correct. 5°/o chose this .
Night blindness is a man ifestation of vitamin A deficiency. Vitamin A is a fat-so luble vitamin and can be seen in
• 23
some malabsorbtive synd romes. Other important functions of vitamin A include protection of epithel ial tissue
• 24 and gene t ranscription .
• 25 Gene Vitamin A Nyctalopia Epithelium Visual impairment Vitamin A deficiency Vitamin Transcription (genetics) lipophilicity
~

6
lock
s
Suspend
0
End Block
Item: 16of25 ~. , . M k <:] t> al ~· ~
QIO: 3174 .l. ar Previous Next Lab 'lifllues Notes Calculator

5
Night blindness is a man ifestation of vitamin A deficiency. Vitamin A is a fat-soluble vitamin and can be seen in
6
some malabsorbtive synd romes. Other important functions of vitamin A include protection of epithelial tissue
7 and gene t ranscription .
8 Gene Vitamin A Nyctalopia Epithelium Visual impairment Vitamin A deficiency Vitamin Transcription (genetics) lipophilicity

9 D is not correct. 15% chose this.


10 Both folic acid and vitamin 8 12 deficiencies lead to a mega loblastic anemia with hypersegmented neutrophils. An
11 important distinction is that only vitam in 8 12 deficiencies result in neu ro log ic abnormalities. Methylmalonic
12 acidemia (an elevation of methylmalonic acid) is another way to clinica lly distinguish between 8 12 and folate
deficiency; 8 12 deficiency is often accompan ied by elevated serum methylmalon ic acid levels.
13 Methylmalonic acidemia Megaloblastic anemia Folic acid Methylmalonic acid Folate deficiency Vitamin B12 Anemia Vitamin B vitamins Neutrophil
14
Hypersegmented neutrophil Blood plasma Acidosis
15
E is not correct. 6°/o chose this.
16
Poor wound healing may be a result of many conditions; one hypovitaminosis resu lting in poor wound hea ling is
• 17
hypovitaminosis C, known as scurvy. Vitamin C is requi red for the proper formation of collagen .
• 18 Scurvy Collagen Vitamin c Wound healing Vitamin

• 19
• 20
Bottom Line:
• 21
The administration of fol ic acid to pregnant women decreases the incidence of neu ral tu be defects in the fetus .
• 22
Folic acid deficiency in the adult can cause megaloblastic anemia and hypersegmented polymorphonuclear
• 23 leukocytes.
• 24 Folate deficiency Megaloblastic anemia Folic acid Anemia White blood cell Neural tube defect Fetus Neural tube Granulocyte

• 25 ~

6
lock
s
Suspend
0
End Block
Item: 16 of 25 ~ 1 • M k -<:J 1>- Jil ~· !:';-~
QIO: 3174 ..L ar Pre v ious Next Labfli!llues Not es Calcula t o r
A A
5
6 FA17p 461 .1

7 Neural tube defects Neuropores fail to fuse (4th week) - pcrsislcnt connection bet\\'een amniotic cavity and spinal
8 canal. Associated\\ ith maternal dittbctes as \\ell as low folic acid intake before conception and
9
during pregnancy. t a-fetoprotein (AFP) in amniotic Auid and maternal serum (except spina
bifida occulta =normal F'P). t acetylcholinesterase ( ChE) in amniotic Auid is a helpful
10
confirmatory test (fetal AChE in CSF' flO\\S through defect into amniotic Auid).
11
Anencephaly Failure of rostral neuropore to close - no forebrain, open cakarium. Clinical findings:
12 polyhydramnios (no swallowing center in brain).
13
Spina bifida occulta Failure of caudal neuropore to close, but no herniation. Usually seen at lower vertebral levels. Dura
14 is intact. Associated \\'ith tuft of hair or skin dimple at level of bony defect.
15 Meningocele \leninges (but no neural tissue) herniate lhrough bony defect. Associated with spina bifida cystica.
16 Meningomyelocele ~ leninges and neural tissue (eg, cauda equina) herniate through bony defect.
• 17
_, , / +1- Tuft of hair / Skin defecVthinning / Skin thin or absent
• 18
Skin ~ ~ +1- Skin dimple
• 19
• 20
{ /;Dura
• 21 y-Leptomeninges

• 22
~~
Sptnal ~
• 23
\_Transverse
• 24 cord process
• 25

a
Lock
s
Suspend
8
End Bl ock
Item: 16 of 25 ~ 1 • M k -<:J 1>- Jil ~· !:';-~
QIO: 3174 ..L ar Pre v ious Next Labfli!llues Not es Calcula t o r
A
.. .. .. ..
~ "' .. .. .. .. .. A
5 during pregnancy. f a -fetoprotein (AFP) in amniotic Auid and maternal serum (except spina
6 bifida occulta =norma l FP). f acetyleholinesterase (AChE) in amniotic Auid is a helpful
7 confirmatory test (fetal AChE in CSF Aow~ through defect into amniotic Auid).
8 Anencephaly Failure of rostral neuropore to close ..... no forebrain, open cah-arium. Clinical findings:
9 polyhydramnios (no swallowing center in brain).
10 Spina bifida occulta Failure of caudal neuropore to close, but no herniation. Usually seen at lower ,·ertebrallevels. Dura
11
is intact. Associated with tuft of hair or skin dimple at Je,·el of bony defect.

12
Meningocele \leninges (but no neural tissue) herniate through bony defect. Associated with spina bifida C}Stica.
13 Meningomyelocele ;\leninges and neural tissue (eg, cauda equina) herniate through bony defect.
14 _, . r +1- Tuft of hair / Skin defect/thinning r Skin thin or absent
~ +1- Sk1n d1mple
15 Skm' \
Subarachnoid
16 space

• 17 ( / : - Dura
f f leptomeninges
• 18

Spinal~0~
• 19
• 20 \_Transverse
cord process
• 21
• 22
• 23 Normal Spina bifida occulta Meningocele Meningomyelocele
(most common)
• 24
• 25

Lock
a s
Suspend
8
End Bl ock
Item: 17 of 25 ~ 1 • M k -<:J 1>- Jil ~· !:';-~
QIO: 3181 ..L ar Pre v ious Next Labfli!llues Notes Calcula t o r
A A
5
A 24-year-old man is brought to the emergency department by the police. He was found pacing outside a
6 loca l bar, holding a baseball bat, and muttering under his breath. On arrival, he appears agitated and is in
7 near continuous motion. He says his chest hurts but he does not know why. Physical examination reveals a
8
heart rate of 190/min, dilated pupils, and evidence of a recent episode of epistaxis . A urine drug screen is positive.
9 Tyrosine . ;- - Tyrosine
10 l
DOPA ~--\---____,
11
12 ~
Dopamine
13
14
15
16
• 17
• 18


Reuptake
• 19
• 20 •
NE - - - -- Diffusion, metabolism
• 21
• 22
·~· !.
~:::::==00.,....,. ,., .,
• 23
• 24
• 25

a
Lock
s
Suspend
8
End Bl ock
Item: 17 of 25 ~ 1 • M k -<:J 1>- Jil ~· !:';-~
QIO: 3181 ..L ar Pre v ious Next Labfli!llues Notes Calcula t o r
- ... I I I I I I - .. • - I I I - I I - I ~ I tt tt T - • ... - · ... .. 1 - ' - ... ..
5
heart rate of 190/min, dilated pupils, and evidence of a recent episode of epistaxis . A urine drug screen is positive.
6
Tyrosine 1-t - - - Tyrosine
7
8 ~
DOPA ~-\-------1
9
10
t
Dopamine
11
12
13
14
15
16 Reuptake

• 17 f)- -·
• 18 NE - - - -- Diffusion, metabolism

·~· !.
• 19
• 20

~:::::===={]).,,..,.,..,.,,.,
• 21
• 22
• 23
• 24
On the diagram above, which letter corresponds to the location of action of the drug this man has taken?
• 25
·----.- ...
Lock
a s
Suspend
8
End Bl ock
5
6
7
Reuptake

·--·
8
9
10 NE - - - --

Diffusion, metabolism
11
12 • f-------+-• !.
~:::::====00••.•..,...,,.,,.
13
14
15
16 On the diagram above, which letter corresponds to the location of action of the drug this man has taken?
• 17
:
• 18 A
• 19
8
• 20
• 21 c
• 22 D
• 23
E
• 24
• 25

a
Lock
s
Suspend
8
End Block
Item: 17 of 25 ~. I • M k <:] t> al ~· ~
QIO: 3181 .l. ar Previous Next lab 'lifllues Notes Calculator


5
The correct answer is D. 73°/o chose this.
6
This man has taken coca ine. He is paranoid and ag itated. In addition, the patient is tachyca rdic, his pupils are
7 dilated, and he appears to be experiencing angina. His recent nosebleed is a classic sign of significant cocaine
8 usage. Cocaine is an indirect sympathomimetic that acts presynaptica lly to inh ibit the reuptake of
9
neurotransmitters such as norepinephrine from the neurona l synapse. The primary site of cocaine's action is the
nucleus accumbens (the dorsal portion of the ventra l striatum).
10 Nucleus accumbens Ventral striatum Sympathomimetic drug Norepinephrine Synapse Cocaine Striatum Tachycardia Neurotransmitter Reuptake Paranoia
11 Angina pectoris Nosebleed
12
A is not correct. 3°/o chose this.
13
A corresponds to increasing the supply of dopamine in the presynaptic pathway. This is how levodopa works in
14
the t reatment of parkinsonism. Levodopa is converted by dopa deca r boxylase in the central nervous system to
15 dopamine, rep lenishing the supp ly of this neurotransmitter. Such a drug could lead to psychosis, but not the
16 above clinical presentation.
Central nervous system l-DOPA Neurotransmitter Parkinsonism Aromatic l-amina acid decarboxylase Dopamine Psychosis Chemical synapse Nervous system
17
• 18 B is not correct. 6°/o chose this .
• 19 8 corresponds to the location of action of reserpine, one of the earliest dr ugs used to treat hypertension. It is
also an antipsychotic agent. It acts by preventing the storage of catecho lamines such as norepinephrine in
• 20
vesicles, leading to their dep letion and thereby decreasing sympathetic nervous system tone .
• 21 Reserpine Sympathetic nervous system Antipsychotic Norepinephrine Catecholamine Hypertension Vesicle (biology and chemistry) Nervous system
• 22
C is not correct. 12% chose this .
• 23
C cor responds to amphetamines, another class of stimu lants producing symptoms similar to those of coca ine
• 24 when abused. Amphetamines are also indirect general agonists at adrenergic synapses, but exert their effects
• 25 ~
by releasing stored catecho lam ines. Given the similar symptom complex, the recent nose bleed is the biggest ~

6
lock
s
Suspend
0
End Block
Item: 17 of 25 ~. I • M k <:] t> al ~· ~
QIO: 3181 .l. ar Previous Next Lab 'lifllues Notes Calculator
:, I I ... • • I ...
5
8 correspo nds to t he location of action of reserpine, one of the earliest dr ugs used to t reat hypertension. I t is
6 also an antipsychotic agent. It acts by preventing the sto rage of catecho lamines such as no repinephrine in
7 vesicles, leading to their dep letion and thereby decreasing sympathetic nervous system tone.
Reserpine Sympathetic nervous system Antipsychotic Norepinephrine Catecholamine Hypertension Vesicle (biology and chemistry) Nervous system
8
9 C is not correct. 12% chose this.
10 C cor responds to amphetamines, another class of stimu lants producing symptoms simila r to those of coca ine
11 when abused. Amphetamines are also indi rect general agonists at ad renergic synapses, but exert their effects
by releasing stored catecho lam ines. Given the simila r symptom complex, the recent nose bleed is the biggest
12
hint as to why th is patient more likely took cocaine ( ie, intranasal route of administration) versus amphetamines
13 ( ie, oral administration).
14 Cocaine Substituted amphetamine Insufflation (medicine) Catecholamine Route of administration Nosebleed Stimulant Synapse Symptom Adrenergic

15 Oral administration

16 E is not correct. 6°/o chose this.


17 E corresponds to a direct ad renergic agon ist, such as epinephrine. Other agonists include phenylephrine and
• 18 norepinephine. Cocaine does not act upon this receptor di rectly; it blocks the reupta ke of adrenergic agonists
• 19 that in turn act on the recepto rs .
Phenylephrine Epinephrine Agonist Cocaine Adrenergic agonist Adrenergic receptor Reuptake Adrenergic Receptor (biochemistry)
• 20
• 21
• 22 Bottom Line:
• 23 Cocaine acts as an ind irect sympathomimetic agent by blocking reupta ke of neu rotransmitters .
Sympathomimetic drug Cocaine Neurotransmitter Reuptake
• 24
• 25 ~

6
lock
s
Suspend
0
End Block
Item: 17 of 25 ~ 1 • M k -<:J 1>- Jil ~· !:';-~
QIO: 3181 ..L ar Pre v ious Next Labfli!llues Notes Calcula t o r
A A
5 FA17 p231 .1
6 Autonomic drugs Release of norepinephrine from a sympathetic ncn c ending is modulated by NE itself, acting on
7 presynaptic a 2-autoreceptors - negative feedback.
8 Amphetamines use the 1 E transporter ('J ET ) to enter the presynaptic terminal, where they utilize
9 the ,·esicular monoamine transporter (V\ I T ) to enter neurosecretory ,·esicles. This displaces E
from the ,·esicles. Once 1 E reaches a concentration threshold within the pres)11aptic terminal,
10
the action of P:ET is reversed , and 1\ E is expelled into the synaptic cleft, contributing to the
11
characteristics and effects of t 1\'E observed in patients taking amphetamines.
12
CHOLINERGIC NORAORENERGIC
13
AXON I)N
14
Tyrosine ~
15 Choline
Tyrosine
16
17
!
DOPA
Choline+
• 18
Acetyl-Co A !
Dopamine
ChAT !
• 19
Reserpine - - - - iQ )-- ----1

/
Release-modulating
ACh tJ'
0 20 receptors
Ca2+ AT II
• 21 %
NE--4)
J
• 22
• 23
• 24
Botulinum
0 25

a
Lock
s
Suspend
8
End Bl ock
Item: 17 of 25 ~ 1 • M k -<:J 1>- Jil ~· !:';-~
QIO: 3181 ..L ar Pre v ious Next Labfli!llues Notes Calcula t o r

5 p g p
6 CHOLINERGIC NORADRENERGIC
7
AXON ON
8 Tyrosine --~--
Choline
9 Tyrosine
10 !
DOPA
Choline+
11 Acetyl-<:oA l
Dopamine
12 ChAT !
13
14
15
J ACh tJ'
Reserpine ---10~-..--...j

Ca2+
NE ~%
Release-modulating
receptors

16 Ca2+

17
• 18 Botulinum

• 19 Cocaine, TCAs,
amphetamine
0 20 NEQ
~ Diffusion,
• 21 metabolism
• 22
AChE inhibitors
• 23
AChE Ad reno receptors a or p
• 24
1--
0 25 POSTSYNAPTIC MEMBRANE -:-s 'NnPI C IIEMBRANE
r ..;~

a
Lock
s
Suspend
8
End Bl ock
Item: 17 of 25 ~. I • M k <:] t> al ~· ~
QIO: 3181 .l. ar Previous Next Lab 'lifllues Notes Calculator

• •
5
6 FA17 p 540.1
7 Psychoactive drug intoxication and withdrawal
8 DRUG ltiTOXICATION WITHDRAWAL

9 Depressants
10 onspecific: mood elevation, l anxiety, onspecific: anxiety, tremor, seizures,
. .
sedation, behavioral disinhibition, respiratory lllSOmnaa.
11
depression.
12
Alcohol Emotional labil ity, slurred speech, ataxia, Time from last drink:
13
coma, blackouts. Serum y-glutamyltransferase 3-36 hr: minor symptoms similar to other
14 (GGT)-sensitive indicator of alcohol usc. depressants
15 AST value is twice ALT value. 6- 48 hr: withdrawal seizures
16 12-48 hr: alcoholic hallucinosis (usually visual)
17 48-96 hr: delirium tremens (DTs) in 5% of
cases
• 18
Treatment: benzodiazepines.
• 19
Opioids Euphoria, respiratory and CNS depression, Sweating, dilated pupils, piloerection ("cold
0 20
l gag reAex, pupillary constriction (pinpoint turkey"), fever, rhinorrhea, yawning, nausea,
0 21 pupils), seizures (overdose). Most common stomach cramps, diarrhea {"Au-like" symptoms).
• 22 cause of drug overdose death. Treatment: Treatment: long-term support, methadone,
0 23 naloxone. buprenorphine.
• 24 Barbiturates Low safety margin, marked res piratory Delirium, life-threatening cardiovascular
0 25 depression. Treatment: symptom management collapse. •

6
lock
s
Suspend
0
End Block
Item: 17 of 25 ~ 1 • M k -<:J 1>- Jil ~· !:';-~
QIO: 3181 ..L ar Pre v ious Next Labfli!llues Notes Calcula t o r
A A
5 Benzodiazepines Greater safety margin. laxia, minor Jeep disturbance, depression, rebound anxiety,
6 respiratory depression. Treatment: Ammwcn il Seizure.
7 (benzodiazepine receptor antagonist, but
rarely used as it can precipitate seizures).
8
Stimulants
9
10
l\onspecific: mood elen1tion, ps)chomotor 'onspecific: post-use "crash," including
agitation, insomnia, cardiac arrh} thmias, depression, lethargy, t appetite, sleep
11
taclwcardja,

a nxietv.
• disturbance. ,·ivid nightmares.
12
Amphetamines Euphoria, grandiosity, pupillary dilation,
13 prolonged wakefulness and attention,
14 hypertension, tachycardia, anorexia, paranoia,
15 fever. se,·ere: cardiac arrest, seizures.
16 Treatment: benzodiazepines for agitation and
se1zures.
17
• 18
Cocaine Impaired judgment, pupillary dilation,
hallucinations (including tactile), paranoid
• 19
ideations, angina, sudden cardiac death.
0 20 Treatment: a -blockers, bcnzodiazcpincs.
• 21 P-blockers not recommended.
• 22 Caffeine Restlessness, t diuresis, muscle twitching. Headache, difficulty concentrating, Au-like
• 23 symptoms.
• 24 Nicotine Restlessness. Irritability, anxiety, restlessness, difficulty
0 25 concentrating. Treatment: nicotine patch,

a
Lock
s
Suspend
8
End Bl ock
Item: 17 of 25 ~ 1 • M k -<:J 1>- Jil ~· !:';-~
QIO: 3181

5
..L
.
ar Pre v ious Next Labfli!llues Notes Calcula t o r
. . . " .
concentrating. Treatment: nicotine patch,
6 gum, or lozenges; bupropion/varcniclinc.
7 Phencyclidine iolence, impulsivity, psychomotor agitation,
8 nystagmus, tachycardia, h) pertension,
9 analgesia, psychosis, delirium, sci.wres.
10
Trauma is most common complication.
Treatment: benzodiazcpincs, rapid-acting
11
antipsychotic.
12
lysergic acid Perceptual distortion (,·isual, auditory),
13 diethylamide (LSD) depersonalization, am.iet), paranoia,
14 psychosis, possible Aashbacks.
15 Marijuana Euphoria, anxiety, paranoid delusions, Irritability, anxiety, depression, insomnia,
16 (cannabinoid) perception of slowed time, impa ired judgment, restlessness, l appetite.
17 social withdrawal, f appetite, dry mouth,
• 18
conjunctival injection, hallucinations.
Pharmaceutical form is clronabinol: used
• 19
0

• 21
20

MDMA (ecstasy)
as antiemetic (chemotherapy) and appetite
stimulant (in AIDS).
Hallucinogenic stimu lant: euphoria, Depression, fa tigue, change in appetite, difficulty
I
• 22 disinhibition, hyperactivil), distorted sensory concentrating, anxiety.
• 23 and time perception, teeth clenching. Life-
• 24 threatening effects include hypertension,
tachycardia, hyperthermia, hypo nat rcmia,
0 25 ..._ _ _ __ ._ ... _: _ _ __ .J_ ... _ _

a
Lock
s
Suspend
8
End Bl ock
Item: 17 of 25 ~. I • M k <:] t> al ~· ~
QIO: 3181 .l. ar Previous Next Lab 'lifllues Notes Calculator

• •
5
6 FA17p234.1
7 Sympathomimetics
8 DRUG ACTION APPLICATIONS

9 Direct sympathomimetics
10 Albuterol, salmeterol l3z > p, Albuterol for acute asthma or COPD. Salmeterol
11 for long-term asthma or COPD control.
12 Dobutamine p, > 13z, a Heart failure (llF) (inotropic > chronotropic),
13
cardiac stress testing.
14 Dopamine 0 1 = D2 > P>a Unstable bradycardia, HF, shock; inotropic and
chronotropic effects at Jo,,·er closes due to P
15
effects; vasoconstriction at high doses due to a
16
effects.
17
Epinephrine P> a Anaphylaxis, asthma, open-angle glaucoma;
• 18 a effects predominate at high doses.
• 19 Significantly stronger effect at 13z-receptor than
• 20 norepinephrine.
• 21 Fenoldopam o, Postoperative hypertension, hrpertensive crisis.
• 22 Vasodilator (coronary, peripheral, renal, and
splanchnic). Promotes natriuresis. Can cause
• 23
hypotension and tachycardia .
• 24
• 25
Isoproterenol p, = l3z Electrophysiologic evaluation of

• ........... ,., ........ l._uj.l., •...,.. ;.-,r r . . _.. u u"\ ..C"Ot"'o ;rrol"\.o. ...."l; ....

6
lock
s
Suspend
0
End Block
Item: 17 of 25 ~ 1 • M k -<:J 1>- Jil ~· !:';-~
QIO: 3181 ..L ar Pre v ious Next Labfli!llues Notes Calcula t o r

) •
5
hypotension and tachyca rdia.
6
Isoproterenol Eleclrophysiologic e,·alualion of
7
lachyarrhythmias. Can worsen ischemia.
8
Midodrine Autonomic insufficiency and postural
9
hypotension. ~ fay exacerbate supine
10 h) pertension.
11 Mirabegron Urinary urge incontinence or O\'eractive bladder.
12 Norepinephrine Hypotension. septic shock.
13
14 Phenylephrine Hypotension (,·asoconstrictor), ocular procedures
15 (mydriatic), rhinitis (decongestant).
16 Indirect sympathomimetics
17 Amphetamine i ndirect general agonist, rcupta kc inhibitor, also arcolepsy, obesity, ADHD.
• 18 releases stored catecholamines
• 19 Cocaine i ndirect general agonist, rcupta kc inhibitor Causes vasoconstriction and local anesthesia.
0 20 e,·er give ~-blockers if cocaine intoxication is
suspected (can lead to unopposed a 1 activation
• 21
and extreme hypertension).
• 22
Ephedrine Indirect general agonist, releases stored 'asal decongestion (pseudoephedrine), urinar}

I
• 23
catecholamines incontinence, hrpotension.
• 24
0 25

a
Lock
s
Suspend
8
End Bl ock
Item: 18 of 25 ~ 1 • M k -<:J 1>- Jil ~· !:';-~
QIO: 25 79 ..L ar Pre v ious Next Lab fli!ltues Notes Calcula t o r

IAA]
A A
5
In the m id-1990s, the Food and Drug Administration mandated the supplementation of grain products with a
6 compound in an attempt to reduce the incidence of a specific category of birth defects. This compound is a
7 coenzyme for 1-carbon transfers, and is absorbed in the jejunum.
8
9 This compound exerts its greatest effect on which area of the developing embryo?
10 :
11 A . Midgut
12 B. Neural folds
13
C. Notochord
14
D. Primitive atrium
15
16 E. Truncus arteriosus
17
• 18
• 19
• 20
• 21
• 22
• 23
• 24
• 25

a
Lock
s
Suspend
8
End Bl ock
Item: 18of25 ~. , . M k <:] t> al ~· ~
QIO: 2579 .l. ar Previous Next lab 'lifllues Notes Calculator


5
6
The correct answer is B. 68°/o chose this.
The U.S. Food and Drug Administration mandated in 1998 that cereal gra in products be fortified with folic acid
7
to decrease neural tube defects, as before this t ime, only one in three pregnant women were consuming enough
8 folic acid . Neural tube defects are the resu lt of a fa ilure of the neural fo lds to close. The neural plate fo lds to
9 become the neural tube, which has openi ngs at either end known as the neuropores. Usual ly, it is the
10
neuropores that are responsible for the neural tube defects that are reduced by fo lic acid, although neura l tube
defects may rarely occu r at other places in the neural fo lds, even when the neuropores close. Fa ilure of the
11 anter ior neural folds to close leads to anencephaly, whereas fai lure of the posterior neura l folds to close leads to
12 spina bifida. Closure of the neural folds usua lly happens in wee k 4 of the pregnancy.
Spina bifida Anencephaly Folic acid Food and Drug Administration Neural tube defect Cereal Neural fold Neural plate Neural tube Pregnancy
13
14 A is not correct. 4°/o chose this.
15 The birth defect that is prevented by fo lic acid is a neura l tube defect. The midgut gives rise to the intestinal
16 tract f rom the lower duodenum through the proximal two thirds of the transve rse colon; thus it is not involved
in the development of neura l tube defects and is unaffected by fo lic acid .
17
Neural tube defect Folic acid Duodenum Transverse colon Neural tube Colon (anatomy) Gastrointestinal tract Anatomical terms of location
18
• 19
C is not correct. 26% chose this .
The notochord is responsible for inducing ectoderm to fo rm the neuroectode rm, wh ich subsequently deve lops
• 20
into the neural plate. The notochord is not directly invo lved in the deve lopment of neu ral tube defects .
• 21 Notochord Ectoderm Neuroectoderm Neural plate Neural tube Neural tube defect

• 22
Dis not correct. 1°/o chose this .
• 23
The birth defect refer red to in the question is a neura l tube defect, wh ich can be prevented by ta king adequate
• 24 amounts of fo lic acid. The primitive atrium gives rise to the t rabeculated portions of the left and right atria; thus
• 25 ~
it is not involved in the development of neural tube defects and is unaffected by fol ic acid .

6
lock
s
Suspend
0
End Block
Item: 18of25 ~. , . M k <:] t> al ~· ~
QIO: 2579

5
.l. ar
-.-:.:-••-.
Previous
-·- .. -.-- -· ·-- - ..... -·
Next lab 'lifllues Notes Calculator

Neural tube defect Folic acid Duodenum Transverse colon Neural tube Colon (anatomy) Gastrointestinal tract Anatomical terms of location
6
C is not correct. 26% chose this.
7
The notochord is responsible for inducing ectoderm to form the neuroectoderm, wh ich subsequently deve lops
8 into the neural plate. The notochord is not directly invo lved in the deve lopment of neu ral tube defects.
9 Notochord Ectoderm Neuroectoderm Neural plate Neural tube Neural tube defect

10
Dis not correct. 1°/o chose this.
11
The birth defect refer red to in the question is a neura l tube defect, wh ich can be prevented by ta king adequate
12 amounts of fo lic acid. The primitive atrium gives rise to the t rabeculated portions of the left and right atria; thus
13 it is not involved in the development of neural tube defects and is unaffected by fol ic acid .
Neural tube defect Folic acid Neural tube Atrium (heart) Trabecula
14
15 E is not correct. 1°/o chose this.
16 The birth defect that is prevented by fo lic acid is a neura l tube defect. The t runcus arteriosus gives rise to the
aorta and pulmonary trun k; thus it is not involved in the development of neural tube defects and is unaffected
17
by folic acid .
18 Neural tube defect Folic acid Pulmonary artery Aorta Neural tube
• 19
• 20
Bottom Line:
• 21
• 22
Folic acid, a coenzyme fo r 1-carbon transfers, decreases neu ral tube defects in the fetus when taken in
adequate amounts by the mothe r ea rly in pregnancy .
• 23 Folic acid Neural tube Pregnancy Cofactor (biochemistry) Neural tube defect Fetus
• 24
• 25 ~

6
lock
s
Suspend
0
End Block
Item: 18 of 25 ~ 1 • M k -<:J 1>- Jil ~· !:';-~
QIO: 25 79 ..L ar Pre v ious Next Lab fli!ltues Notes Calcula t o r
A A
5
6 FA17 p 461 .1

7 Neural tube defects leuropores fail to fuse (4th week) - persistent connection bet\\'een amniotic cavity and spinal
8 canaL ssociated '' ith maternal diabetes as ''ell as low folic acid intake before conception and
9 during pregnancy. t a-fetoprolein (AFP) in amniotic Auid and maternal serum (except spina
bifida occulta = normal AFP). t acetylcholinesterase ( ChE) in amniotic Auid is a helpful
10
confirmatory test (fetal AChE in CSF AO\\S through defect into amniotic Auid).
11
Anencephaly Failure of rostral neuropore to close - no forebrain, open cah-arium. Clinical findings:
12
polyhydramnios (no swallowing center in brain).
13
Spina bifida occulta Failure of caudal neuropore to close, but no herniation. Usually seen at lower ,·ertebral b ·cls. Dura
14 is intact. Associated with tuft of hair or skin dimple at level of bony defect.
15
Meningocele 1\ leninges (but no neural tissue) herniate through bon} defect. Associated with spina bifida cystica.
16
Meningomyelocele 1\ len inges and neural tissue (cg, cauda equina) herniate through bony defect.
17
..../ +1- Tuft of hair / Skin defectlthinning
18 ~ +1- Skin d1mple
Skin ~
• 19 Subarachnoid
space
• 20
• 21
• 22
~:., ,., ...
• 23
Sptnal ~ \_Transverse
• 24 cord process
• 25

a
Lock Suspend
s 8
End Bl ock
Item: 18 of 25 ~ 1 • M k -<:J 1>- Jil ~· !:';-~
QIO: 25 79 ..L ar Pre v ious Next Lab fli!ltues Notes Calcula t o r
A A
5
6 FA17 p64.2
7 Vitamin 8 9 (folate)

8 FUNCTION Converted to tetrahydrofolic acid (T il F), a Found in leafy green ,·egetables. Absorbed in
9 coenzyme for !-carbon transfer/methylation jejunum. Folate from foliage.
reactions. Small resen·e pool stored primarily in the Ji,·er.
10
Important for the synthesis of nitrogenous bases
11 in OJ A and Rl'\/A.
12
D£FICIENCY ~ lacrocrtic, megaloblastic anemia; Deficiency can be caused by several drugs (eg,
13 h~ persegmented polymorphonuclear cells phenytoin, sulfonamides, methotrexate).
14 (PN!, s); glossitis; no neurologic S) mptoms Supplemental maternal folic acid at least I month
15 (as opposed to vitamin B12 deficiencr). Labs: prior to conception and during earlr pregnancy
t homocysteine, normal mcthylmalonic acid to ~ risk of neural tube defects.
16
le,·cls. .VIost common vitamin deficiency in
17
the United States. Seen in alcoholism and
18 pregnancy.
• 19
• 20
FA17 p 4 60.1
• 21
Neural development Notochord induces overlying ectoderm to differentiate into neuroectoderm and form neural plate.
• 22 1 eural plate gives rise to neural tube and neural crest cells.
• 23 Day IS 1 otochord becomes nucleus pulposus of intervertebral disc in adults .
• 24
Notochord
Neoralfold Alar plate (dorsal): sensory J . . .
Same oncntallon as spmal cord.
• 25 ~ Basa I pIate (,·entra I): motor

a
Lock
s
Suspend
8
End Bl ock
Item: 18 of 25 ~ 1 • M k -<:J 1>- Jil ~· !:';-~
QIO: 25 79 ..L ar Pre v ious Next Lab fli!ltues Notes Calcula t o r

Small reserve pool stored primarily in the li\'er.


A A
5 reactions.
6 Important for the synthesis of nitrogenous bases
in D A and RNA.
7
DEFICIENCY \ lacrocytic, megaloblastic anemia; Deficiency can be caused by se,·eral drugs {eg,
8
h) persegmented polymorphonuclear cells phenytoin, sulfonamides, methotrexate).
9
(PM, s); glossitis; no neurologic S) mptoms Supplemental maternal folic acid at least I month
10 {as opposed to ,·itamin 8 12 deficiency). Labs: prior to conception and during early pregnancy
11 f homocysteine, normal methrlmalonic acid to l risk of neun1l tube defects.
12 le\'cls. .\ lost common vitamin deficiencr in
13
the United States. Seen in alcoholism and
pregnancy.
14
15
FA17 p460.1
16
17
Neural development otochord induces overlying ectoderm to d ifferenti<~te into neuroectoderm and form neural plate.
Neuralplate eural plate gives rise to neural tube and neural crest cells.
18
Dayt8 otochord becomes nucleus pulposus of intervertebra l disc in adults .
• 19
• 20
Alar plate (dorsal): sensory
Basa I pIate (ventra I): motor
J . . .
Same onentatton as spma l cord .
• 21
• 22
trest
• 23 cells
Day21
• 24
• 25

a
Lock
s
Suspend
8
End Bl ock
Item: 19 of 25 ~ 1 • M k -<:J 1>- Jil ~· !:';-~
QIO: 3347 ..L ar Pre v ious Next Lab fli!ltues Not es Calcula t o r
A A
5
A 68-year-old man suffers from a resting tremor and rigid ity during his last 5 years of life. He is followed in
6 neurology clinic and treated with dopaminergic therapy. His symptoms eventually worsen and he develops
7 postural instability. The neuropathology shown in the image was observed during postmortem examination.
8
9

10
11
12
13
14
15
16
17
18
• 19
0 20
• 21
• 22
Mutations in which protein are genetically linked to the disease from wh ich this man suffered?
• 23
:
• 24
A. a-Synuclein
0 25

a
Lock
s
Suspend
8
End Bl ock
15
16 Mutations in wh ich protein are genetica lly linked to the d isease from which this man suffered?
17
:
18 A. a-Synuclein
• 19
B. Dystrophin
0 20
• 21 C. Huntingtin
• 22 D. Presenilin
• 23
E. Tau
• 24
0 25

a
Lock
s
Suspend
8
End Block
Item: 19 of 25 ~. I • M k <:] t> al ~· ~
QIO: 3347 .l. ar Previous Next lab 'lifllues Notes Calculator


5
6
The correct answer is A. 74°/o chose this.
This patient suffered from Parkinson disease (PD), a neurodegenerative disorder of the substantia nigra and
7
locus ceruleus that is most often characterized by Trem or, Rigidity, Akinesia, an d Postural instability (TRAP) .
8 The photomicrog raph shows several neurons from the substantia nigra stained with haematoxyl in and eosin at
9 500 times magnification, at least two of which (a r rows) exhibit large Lewy bodies (eosinoph ilic cytoplasmic
10 inclusions that consist of a dense core surrounded by a halo of 10-nm wi de radiating fibrils). These aggregates
are the major patho log ical f eatu re of PD and are composed primarily of the protein a-synuclein. Some inher ited
11
for ms of PD have been shown to be caused by mutations in the a-synuclein gene.
12 locus coeruleus Substantia nigra Haematoxylin Neurodegeneration Parkinson' s disease Gene Protein lewy body Neuron Eosin Eosinophilic H&E stain Mutation

13 Pathology Cytoplasm

14
B is not correct. 4°/o chose this.
15
Dystrophin is the gene disrupted in Duchenne muscu lar dystrophy . This disease is character ized by progressive
16 muscle weakness and does not have any significant neuropathology.
17 Dystrophin Duchenne muscular dystrophy Gene Muscular dystrophy Muscle weakness Neuropathology Muscle

18 C is not correct. 5°/o chose this.


19 Trinucleotide repeat expansions inside the huntingtin protein ca use Huntington disease (HD), a
0 20 neurodegenerative disease that pr imari ly affects the caudate nucleus and clinica lly manifests with chorea and
0
21
dementia. This patient's cl inica l symptoms and pathology are not consistent with HD.
Huntington' s disease Caudate nucleus Neurodegeneration Huntingtin Chorea Trinucleotide repeat disorder Protein Dementia Cell nucleus Pathology
0
22
0
23 D is not correct. 6°/o chose this.
0 24
Mutations in the presenilin gene have been linked to early for ms of Alzheimer disease (AD), the most common
cause of dementia in the elder ly . Neuropathologica lly, AD manifests with darkly staining neurofi brillary tang les
0 25 ~ ;::~nri PYtr;o~rPi h d ;o~r ;::~mvln i ri n l ;o~n1 1 P<: RPc:tinn trPmnr ;::~nri I PWV hnri iPc: ;o~rp nnt ;o~c:c:nri ;o~tPri with A f)

6
lock
s
Suspend
0
End Block
Item: 19 of 25 ~. I • M k <:] t> al ~· ~
QIO: 3347 .l. ar Previous Next lab 'lifllues Notes Calculator

5
C is not correct. 5°/o chose this.
6
Trinucleotide repeat expansions inside the huntingtin protein cause Huntington disease (HD), a
7 neurodegenerative disease that pr im ari ly affects the cau date nucleus and clinica lly m anifests with chorea and
8 dementia. This patient's cl inica l symptoms and pathology are not consistent with HD.
Huntington's disease Caudate nucleus Neurodegeneration Huntingtin Chorea Trinucleotide repeat disorder Protein Dementia Cell nucleus Pathology
9
10 D is not correct. 6°/o chose this.
11 Mutations in the presenilin gene have been linked to early for ms of Alzheimer disease (AD), the most common
12 cause of dementia in the elder ly . Neuropathologica lly, AD m anifests with darkly staining neurofi brillary tang les
and extracellular amyloid plaq ues. Resting tremor and Lewy bodies are not associated with AD .
13 Gene Presenilin Amyloid Alzheimer's disease Neurofibrillary tangle Dementia lewy body Mutation Tremor
14
E is not correct. 11% chose this.
15
Mutations in tau cause frontotempora l dem entia with parkinsonism linked to chrom osome 17 (FTDP-17) .
16
Severa l other neu rodegenerative disorders includ ing Alzheimer disease, Pick disease, and progressive
17 supranuclear palsy manifest with abnorma l cytoplasmic accumu lations of tau protein. Lewy bodies are not
18 associated with these neu rodegenerative diso rders.
Progressive supranuclear palsy Tau protein Frontotemporal dementia Parkinsonism Dementia lewy body Chromosome 17 (human) Alzheimer's disease
19
Neurodegeneration Protein Chromosome Mutation Tau Cytoplasm
0 20
0
21
0
22 Bottom Line:
0
23 a-Synuclein is accumulated in cells of patients with Parkinson disease.
0 24 Parkinson's disease

0 25

6
lock
s
Suspend
0
End Block
Item: 19 of 25 ~. I • M k <:] t> al ~· ~
QIO: 3347 .l. ar Previous Next lab 'lifllues Notes Calculator
FIRST AID FACTS
5
6
FA17 p490.1
7
Neurodegenerative l in cognitive ability, memory, or function with intact consciousness.
8 disorders
9 DISEASE DESCRIPTION HISTOLOGIC/GROSS FINDINGS
10 Parkinson disease Parkinson TRAPS your body: Loss of dopam inergic neurons (ie,
11 Tremor (pill-rolling tremor at rest) depigmentation) of substantia nigra pars
12 Rigidity (cogwheel) compacta.
Akinesia (or bradykinesia) Lewy bodies: composed of a-synuclein
13
Postural instability (intracellular eosi noph ilic inclusions t:i.l).
14
ShufAing gait
15 MPTP, a contaminant in illegal drugs, is
16 metabolized to lPP+, which can cause
17 parkinsonian symptoms.
18 Huntington disease Autosomal dominant trinucleotide (CAG)n Atrophy of caudate and putamen with ex vacuo
19 repeat disorder on chromosome 4. Symptoms ventriculomega ly.
manifest between ages 20 and 50: chorea, t dopamine, l GABA, l ACh in brain. 1 euronal
0 20
athetosis, aggression, depression, dementia death via MDA-R binding and glutamate
0
21 (sometimes initially mistaken for substance excitotoxicity.
0
22 abuse).
0
23 Anticipation results from expansion of CAG
0 24 repeats. Caudate loses ACh and GABA.
0 25 Alzheimer disease Most common cause of dementia in elderly. Widespread cortical atrophy (normal cortex IJ);

6
lock
s
Suspend
0
End Block
Item: 19 of 25 ~ 1 • M k -<:J 1>- Jil ~· !:';-~
QIO: 3347 ..L ar Pre v ious Next Lab fli!ltues Not es Calcula t o r

5
A
repeats. Caudate loses ACh and GABA. A

6 Alzheimer disease lost common cause of dementia in elder!\'. Widespread cortical atrophy (norma l cortex [);
'
7
00\m syndrome patients ha,·e t risk of cortex in Alzheimer disease ), especially
developing Alzheimer disease, as APP is hippocampus (arrows in [) and ). l arrO\\ ing
8
located on chromosome 21. of g) ri and widening of sulci.
9
Associated with the folio" ing altered proteins: enile plaques m in gray matter: extracellular
10 poE2: ' risk of sporadic form 13-amyloid core; may cause am) loid angiopathy
11 poE4: t risk of sporadic form - intracranial hemorrhage; Al3 (amyloid-13)
12 • APP, presenilin-1, presenilin-2: familial synthesized by clea,·ing amyloid precursor
forms (10%) with earlier onset protein (APP).
13
' ACh 'eurofibrillary tangles 1]: intracellular,
14
hrperphosphorylated tau protein = insoluble
15 cytoskeletal elements; number of tangles
16 correlates with degree with dementia.
17 Frontotemporal Early changes in persona lit}' and behavior Frontotemporal lobe degeneration 0 .
18 dementia (Pick (behavioral variant), or aphasia (primary Inclusions of hyperphosphorylaled tau (round
19
disease) progressive aphasia). Pick bodies (!I) or ubiquitinated TDP-43.
May ha,·e associated movement disorders
0 20
(eg, parkinsonism, ALS-I ike UMN/Ll'vl
0
21 degeneration).
0
22 Lewy body dementia Dementia and visual hallucinations Intracellular Lew) bodies rJ primarily in corte>...
• 23 ("ha Lewycinations") - parkinsonian features
• 24 Vascular dementia Result of multiple arterial infarcts and/or lRI or CT shows multiple cortical and/or
0 25 chronic ischemia. subcortical infarcts.

a
Lock Suspend
s 8
End Bl ock
Item: 19 of 25 ~ 1 • M k -<:J 1>- Jil ~· !:';-~
QIO: 3347 ..L ar Pre v ious Next Lab fli!ltues Not es Calcula t o r
A A
5 Vascular dementia Result of multiple arterial infarcts and/or l\IRI or CT shows multiple cortical and/or
6 chronic ischemia. subcortical infarcts.
Step-wise decline in cognitive ability with late-
7
onset memory impairment. 2nd most common
8
cause of dementia in elderly.
9
Creutzfeldt-Jakob Rapidly progressi,·e (weeks to months) dementia Spongiform cortex.
10 disease with myoclonus ('"startle myoclonus"}. Prions (Prpc - Pd>sc sheet [f3-pl eated sheet
11 Commonly see per1odic sharp waves on EF.:C resistant to proteases]) CJ.
12 and t 14-3-3 protein in CSF.
13
14
15
16
17
18
19
0 20
• 21
• 22
• 23
• 24
0 25

a
Lock
s
Suspend
8
End Bl ock
Item: 19 of 25 ~ 1 • M k -<:J 1>- Jil ~· !:';-~
QIO: 3347 ..L ar Pre v ious Next Lab fli!ltues Not es Calcula t o r
- - a - - ~ A • •

A A
5
6 FA17 p 489.1
7 Movement disorders
8 DISORDER PRESENTATION CHARACTERISTIC lESION NOTES

9 Akathisia Restlessness and intense urge Can be seen with neuroleptic


10 to move use or in Parkinson disease.
11 Asterixis Extension of wrists causes Associated with hepatic
~Aapping" motion encephalopathy, Wilson
12
disease, and other metabolic
13
derangements.
14
Athetosis Slow, snake-like, \\'rithing Basal ganglia
15 movements; especial!) seen in
16 the fingers
17 Chorea Sudden, jerky, purposeless Basal ganglia Chorea = dancing.
18 movements Sydenham chorea seen in acntc
19 rheumatic fever.
0 20 Dystonia Sustained, involuntary muscle Writer's cramp, blepharospasm,
contractions torticoll is.

I
. 21
. 22 Essential tremor II igh-frequency tremor Often familial. Patients often
with sustained posture self-medicate with alcohol,
. 23
(eg, outstretched arms), which ! tremor amplitude.
. 24 worsened with mo,·ement or Treatment: nonselective
0 25 when anxious 8-blockers (eg, propranolol),

a
Lock
s
Suspend
8
End Bl ock
Item: 19 of 25 ~ 1 • M k -<:J 1>- Jil ~· !:';-~
QIO: 3347 ..L ar Pre v ious Next Lab fli!ltues Not es Calcula t o r

5
Dystonia Sustained, involuntary muscle Writer's cramp, blepharospasm,
6
contractions torticollis.
7
Essential tremor High-frequency tremor Often familial. Patients often
8
with sustained posture self-medicate with alcohol,
9 (eg, outstretched arms}, which l tremor amplitude.
10 worsened with mo,·ement or Treatment: nonselective
11 when anxious P-blockers (eg, propranolol),
primidonc.
12
13
Hemiballismus Sudden, wild Aailing of I arm Coni ra latera I subtha Iamie Pronounce "Half-of-body
+1- ipsilateral leg nucleus (eg, lacunar stroke) ball istic."
14
Contralateral lesion.
15
Intention tremor Slow, zigzag motion when Cerebellar d)sfunction
16
pointing/extending toward a
17 target
18 Myoclonus Sudden, brief, uncontrolled Jerks; hiccups; common in
19 muscle contraction metabolic abnormalities such
0 20 as renal and liver fai lure.
. 21 Resting tremor Uncontrolled movement of distal Subst·anlia nigra (Parkinson Occurs at rest; "pill-rolling
. 22 appendages (most noticeable disease) tremor" of Parkinson disease.
in hands}; tremor alleviated b)' When you park your car, it is

I
. 23
intentionaJ movement at rest.
. 24
0 25

a
Lock
s
Suspend
8
End Bl ock
Item: 20 of 25 ~ 1 • M k -<:J 1>- Jil ~· !:';-~
QIO: 1668 ..L ar Pre v ious Next Lab fli!ltues Notes Calcula t o r

IAA]
A A
5
A 2-week-old girl is brought to t he pediatrician for her fi rst doctor's appointment . Physica l examination
6 revea ls a tuft of ha ir on her back covering a dimple at the level of her LS vertebrae. Neurolog ic examination
7 shows no deficit.
8
9 Which of the following conditions does this baby have?
10 :
11 A. Anencephaly
12 B. Encephalocele
13
C. Meningocele
14
D. Meningomyelocele
15
16 E. Spina bifida occulta
17
18
19
0 20
0 21
0 22
. 23
. 24
0 25

a
Lock
s
Suspend
8
End Bl ock
Item: 20 of 25 ~. I • M k <:] t> al ~· ~
QIO: 1668 .l. ar Previous Next lab 'lifllues Notes Calculator


5
6
The correct answer is E. 88°/o chose this.
Spina bifida occu lta is the most minima l defect in the spectrum of neural tube malformations. Spina bifida
7
occu lta is caused by fa ilure of the posterior vertebral arches to close med ially and is cha racterized by the
8 absence of any protrusion th rough this deficit. Th is abnormality occurs at the LS or Sl vertebrae and is often
9 ma rked by a lipoma or tuft of hai r. Since the re is no hern iation of the meninges or spina l cord, th is defect is
10
general ly asymptomatic.
lipoma Spina bifida Meninges Spinal cord Vertebra Neural tube Spina bifida occulta Spinal disc herniation
11

12 A is not correct. 1 °/o chose this.


13
Anencepha ly occurs when the cepha lic end of the neu ral tube fa ils to close, resu lt ing in the absence of a majo r
portion of the brain, sku ll, and sca lp. I nfants with this disorde r are born without a forebrain and a cereb rum.
14
The rema ining brain tissue is often exposed ( ie, not covered by bone or skin) .
15 Anencephaly Cerebrum Neural tube Forebrain Scalp Human skull Human brain Skull Bone Brain Head

16
B is not correct. 1 °/o chose this.
17
Encepha locele is a neu ral tube defect that occurs when the cephalic end of the neural tube fai ls to close.
18 Encepha locele invo lves protrusion of a sac-like structure th rough the skul l, into which the meninges or brain and
19 meninges herniate. I t wou ld be very unusua l fo r such a malformation to be asymptomatic.
Neural tube defect Encephalocele Meninges Neural tube Human skull Head Skull
20
• 21 C is not correct. 7°/o chose this .
• 22 Spina bifida cystica is a fami ly of malformations caused by failure of the poster ior vertebral arches to close
medial ly, along with external protrusion of a sac-like structu re. Spina bifida cystica is furthe r classified acco rding
• 23
to the contents of the sac into meningoce le (men inges and ce rebrospinal fluid), meningomyeloce le (men inges,
• 24 cerebrospinal fluid, and spina l cord), or myeloschisis (exposed f lat mass of neu ral tissue from failed closu re).
• 25 ~
The extent of neural involvement progresses with the different forms of spina bifida cystica; skin sensation is ~

6
lock
s
Suspend
0
End Block
Item: 20 of 25 ~. I • M k <:] t> al ~· ~
QIO: 1668 .l. ar Previous Next lab 'lifllues Notes Calculator
. .

5
6
C is not correct. 7°/o chose this.
Spina bifida cystica is a fami ly of malformations caused by failure of the poster ior vertebral arches to close
7
medially, along with external protrusion of a sac-like structure . Spina bifida cystica is further classified accord ing
8 to the contents of the sac into meningocele (men inges and cerebrospinal fluid), meningomyelocele (men inges,
9 cerebrospinal fluid, and spinal cord), or myeloschisis (exposed flat mass of neu ral tissue from failed closu re).
10 The extent of neural involvement progresses with the different forms of spina bifida cystica; skin sensation is
usually lost from the affected region, and there is some degree of muscle pa ralysis. In this patient, men ingocele
11
is unlikely due to the lack of protrusion of a sac-li ke structure (therefore no herniation of meninges) and the
12 absence of neurolog ic deficit.
Spina bifida Cerebrospinal fluid Meninges Vertebra Spinal cord Paralysis Nervous tissue Somatosensory system Neurology Muscle Spinal disc herniation
13
14 D is not correct. 3°/o chose this.
15 Men ingomyelocele, a type of spina bifida cystica, occurs when there is herniation of men inges and the spinal
16 cord through the spinal canal defect. In this patient, meningomyelocele is unlikely due to the lack of protr usion
of a sac-like structure (therefore no hern iation of meninges and spinal cord) and the absence of neurologic
17
deficit.
18 Spina bifida Meninges Spinal canal Spinal cord Neurology Spinal disc herniation

19
20
Bottom Line:
• 21
• 22
Spina bifida occulta is a malformation in which one or more vertebrae do not completely fuse and leave a smal l
opening in the back. Fortunately, neither the thecal sac nor its contents extrude through th is opening . This
• 23 defect is often ma r ked externally by dimpling of the skin, a der mal sinus, or a tuft of ha ir .
• 24 Spina bifida Thecal sac Spina bifida occulta Vertebra

• 25 ~

6
lock
s
Suspend
0
End Block
Item: 20 of 25 ~ 1 • M k -<:J 1>- Jil ~· !:';-~
QIO: 1668 ..L ar Pre v ious Next Lab fli!ltues Notes Calcula t o r
A A
5
6 FA17 p 461 .1

7 Neural tube defects Neuropores fail to fuse (4th week) - persistent connection between amniotic cavity and spinal
8 canaL Associated'' ith maternal diabetes as ''ell as low folic acid intake before conception and
9 during pregnancy. f a-fetoprotein (AFP) in amniotic Auid and maternal serum (except spina
bifida occulta = normai ,\FP). f acctylcholinestcrasc ( ChE) in amniotic Auid is a helpful
10
confirmatory test (fetal AChE in CSF Aows through defect into amniotic Auid).
11
Anencephaly Failure of rostral neuropore to close - no forebrain, open cah-arium. Clinical findings:
12
polyhrdramnios (no swallowing center in brain).
13
Spina bifida occulta Failure of caudal neuropore to close, hut no herniation. Usually seen at lower ,·ertcbral b ·cls. Dura
14 is intact. Associated with tuft of hair or skin dimple at level of bony defect.
15 Meningocele 1\ leninges (but no neural tissue) herniate through bon) defect. Associated with spina bifida cystica.
16
Meningomyelocele 1\ len inges and neural tissue (cg, cauda ec1uina) herniate through bony defect.
17
·'"~ +1- Tuft of hair
18 ~ +I- Skin d1mple
Skin ~
19
20
. 21 !{~:.,.;.,.,
. 22
. 23
Sprnal _____:? \_Transverse
. 24 cord process
• 25

a
Lock Suspend
s 8
End Bl ock
Item: 21 of 25 ~ 1 • M k -<:J 1>- Jil ~· !:';-~
QIO: 2622 ..L ar Pre v ious Next Lab fli!ltues Notes Calcula t o r
A A
5
A 47-year-old man complains of episodic diaphoresis, palpitations, and headache . These spells have been
6 occurring more frequently and are lasting longer. Physical examination reveals a patient in obvious distress
7 with facial pallor, sweating, heart rate of 105 BPM and a blood pressure of 150/ 95 mm Hg. Urinary and
8
plasma catecholamine and metanephrine levels are increased. A coronal view of the patient's CT scan is shown in
the image .
9
10
11
12
13
14
15
16
17
18
19
20
. 21
. 22
. 23
. 24
• 25

a
Lock
s
Suspend
8
End Bl ock
Item: 21 of 25 ~ 1 • Ma r k -<:J I> ~ £!1}>'
• !!":-~
QIO: 2622 ..L Prev ious Next Lab lues Notes Cal culat o r
I • • ,.. - I • - • • - .. - I - - J - • - - .. • I • e I • :t' - It I - I • - .. I fl
5
the image .
6
7
8
9
10
11
12
13
14
15
16
17
18
19
20
. 21
. 22
. 23
. 24 Image courtesy of Radiopaedia. Dr. Paresh K. Desai
• 25

Lock
a s
Suspend
8
End Bl ock
Image courtesy of Radiopaedia. Dr. Paresh K. Desai
15
16 What is the direct embryologic origin of the cells that make up this mass?
17
:
18 A. Endoderm
19
B. Mesoderm
20
. 21 C. Neural crest
. 22 D. Neuroectoderm
. 23
E. Surface ectoderm
. 24
• 25

a
Lock
s
Suspend
8
End Block
Item: 21 of 25 ~ 1 • M k -<:J 1>- Jil ~· !:';-~
QIO: 2622 ..L ar Pre v ious Next Lab fli!ltues Notes Calcula t o r
A A
5
The correct answer is C. 68°/o chose this.
6
The patient has a pheochromocytoma, a tumor of the adrena l medulla. The
7
CT image shows an enlarged right adrenal gland (indicated by the red circle)
8 directly above the kidney. Pheochromocytomas can secrete catecholamines
9 (norepinephrine and epinephrine), resulting in episodes characterized by
headaches, diaphoresis, palpitations, and severe hypertension. Urinary
10
catecholamine metabolites (vanillylmandelic acid, or VMA) and plasma
11 catecholamine levels are elevated in these patients. Most of these tumors are
12 benign, unilateral, and located in the adrenal gland. Pheochromocytomas
13 arise from the chromaffin cells of the adrenal medulla , which are derived from
neural crest cells. A mnemonic to remember some features of
14
pheochromocytomas is the Rule of lOs: 10°/o are extra-adrenal (in the
15 urinary bladder and organ of Zuckerkandl), 10°/o are bilateral, 10°/o are
16 pediatric, 10°/o are malignant, 10°/o are not associated with hypertension,
10°/o are familia l, and 10°/o contain calcification .
17
Adrenal gland Catecholamine Pheochromocytoma Adrenal medulla Epinephrine Perspiration
18
Image courtesy of Radiopaedia,
Organ of Zuckerkandl Norepinephrine Urinary bladder Hypertension Neural crest Palpitations Chromaffin cell Dr. Paresh Desai
19
Kidney Blood plasma Neoplasm Malignancy Medulla oblongata Gland Cancer Benign tumor Metabolite
20
21 A is not correct. 4°/o chose this.
. 22 The chromaffin cells of the adrenal medul la are derived from neura l crest cells not the endoderm . Adu lt
endoderm derivatives include the gut epithelium and its derivatives .
. 23 Endoderm Adrenal medulla Neural crest Chromaffin cell Epithelium Medulla oblongata Adrenal gland Gastrointestinal tract
. 24
B is not correct. 7 °/o chose this .
• 25
• "'rl .... - -1 • • • · · - 1 .................... . . . 1... ! ... 1... .............. .L- ...... - 1 - l ...... L ... . . . . . . . . . . _ I , .............. ...I..!.., ... ! .J ... _,. J ....... . I .... .. - ...... .... _ ... . ... J ... .. : . . ... J .C..... .. . Ll ....

a
Lock
s
Suspend
8
End Bl ock
Item: 21 of 25 ~. I • M k <:] t> al ~· ~
QIO: 2622 .l. ar Previous Next lab 'lifllues Notes Calculator


5
B is not correct. 7°/o chose this.
6
The adrenal cortex, wh ich secretes aldosterone, glucocorticoids, and sex hormones, is derived from the
7 mesoderm. The adrena l cortex does not secrete catecholamines, which are responsible for the hyperadrenergic
8 episodes seen in patients with pheochromocytomas .
Aldosterone Adrenal cortex Mesoderm Catecholamine Glucocorticoid Sex steroid Pheochromocytoma Cerebral cortex Hormone Cortex (anatomy)
9
10 D is not correct. 19% chose this.
11 The chromaffin cells of the adrena l medul la arise from neura l crest cells wh ich are derived from the ect oderm;
12
however, of the possible answer options this is not the most specific. Adult neuroect oderm derivatives include
the neurohypophysis, central nervous system neurons, oligodendrocytes, astrocytes, ependyma l cel ls, retina,
13 and the pinea l gland .
14 Pineal gland Ectoderm Adrenal medulla Central nervous system Neural crest Neuroectoderm Retina Ependyma Posterior pituitary Oligodendrocyte Astrocyte

15 Chromaffin cell Medulla oblongata Nervous system Neuron Adrenal gland Gland

16
E is not correct. 2°/o chose this.
17
The chromaffin cells of the adrena l medul la are not derived from surface ectoderm . Surface ectoderm gives rise
18 to the adenohypophysis, lens of the eye, epiderm is, and the epithel ial linings of the skin, ear, eye, and nose.
19 Ectoderm Adrenal medulla Anterior pituitary Epidermis Epithelium Chromaffin cell Surface ectoderm lens (anatomy) Adrenal gland Medulla oblongata

20
21 Bottom Line:
• 22
Pheochromocytomas arise from chromaffin cells in the adrenal medul la, wh ich is derived from neura l crest
• 23 cells.
• 24 Adrenal medulla Neural crest Chromaffin cell Pheochromocytoma Medulla oblongata

• 25 ~

6
lock
s
Suspend
0
End Block
Item: 21 of 25 ~ 1 • M k -<:J 1>- Jil ~· !:';-~
QIO: 2622 ..L ar Pre v ious Next Lab fli!ltues Notes Calcula t o r
A A
5 FA17 p 326.1
6 Pheochromocytoma
7 ETIOLOGY lost common tumor of the adrenal medulla in Rule of IO's:
8 adults · . Derived from chromaffin cells (arise 10% malignant
9 from neural crest). IO'lc bilateral
:\lay be associated with germline mutations (eg, 10% extra-adrenal (eg, bladder wall, organ of
10
1 F-1, VHL, RET [\l Et\ 2 , 28]). Zuckerkandl)
11
IO'lc calci fy
12 10% kids
13
14
15 Episodic hyperadrenergic symptoms (5 P '~):
SYMPTOMS !\lost tumors secrete epinephrine,
16 norepinephrine, and dopamine, which can Pressure (t BP)
17 cause episodic hypertension. Pain (headache)
18 Symptoms occur in "spells" -relapse a11d rc111il. Perspiration
Palpitations (tachycardia)
19
Pallor
20
FINDINGS t catecholamines and metancphrincs in urine
21
and plasma.
. 22
TREATMENT Irreversible a-antagonists (eg, Phenox} benzamine (16 letters) is given for
• 23
phenoxybenzamine) followed by ~-blockers pheochromocytoma (also 16 letters).
• 24 prior to tumor resection. a-blockade must be
• 25 achieved before giving ~-blockers to a\oid a

a
Lock
s
Suspend
8
End Bl ock
Item: 21 of 25 ~ 1 • M k -<:J 1>- Jil ~· !:';-~
QIO: 2622 ..L ar Pre v ious Next Lab fli!ltues Notes Calcula t o r
A A
5 TREATMENT Irre,•ersible <X-antagon ists (eg, Phenoxy benzamine (16 letters) is given for
6 phenoxybenzamine) followed by P-blockers pheochromocytoma (also 16 letters).
7
prior to tumor resection. a -blockade must be
achieved before giving P-blockers to a\ oid a
8
hypertensive crisis.
9
10
FA17 p 581 .1
11
Embryologic derivatives
12
Ectoderm E-.ternallouter layer
13
Surface ectoderm Epidem1is; adenohypophysis (from Rathke Craniopharyngioma -benign Rathke pouch
14
pouch); lens of eye; epithelial linings of om I tumor with cholesterol crystals, calcifications.
15 cavity, sensory organs of ear, and olfactory
16 epithelium; anal canal below the pectinate line;
17 parotid, sweat, mammary glands.
18 Neural tube Brain (neurohypophysis, C S neurons, oligo- 'curoectoderm-think C S.
19
dendrocytes, astrocytes, ependymal cells, piucal
gland), retina, spinal cord.
20
Neural crest P S (dorsal root ganglia, cranial nerves, cural crest- think P Sand non-neural
21
autonomic ganglia, Schwann cells), structures nearby.
. 22
melanocytes, chromaffin cells of adrenal
• 23 medulla, para follicu lar (C) cells of thyroid,
• 24 pia and arachnoid, bones of the sJ..ull,
• 25 odontoblasts, aorticopulmonary septum,
. .

a
Lock
s
Suspend
8
End Bl ock
Item: 21 of 25 ~ 1 • M k -<:J 1>- Jil ~· !:';-~
QIO: 2622 ..L ar Pre v ious Next Lab fli!ltues Notes Calcula t o r
A A
5
6 FA17p581 .1

7
Embryologic derivatives

8 Ectoderm E'tcrnal/ouler layer


9 Surface ectoderm Epidem1is; adenohypophysis (from Rathke Craniopharyngioma-benign Rathke pouch
10
pouch}; lens of e)e; epithelial linings of oral tumor with cholesterol cr) stals, calcifications.
cavity, sensory organs of ear, and olfactOI)
11
epithelium; anal canal below the pectinate line;
12 parotid, sweat, mammal) glands.
13
Neural tube Brain (neurohypophysis, CNS neurons, oligo- Neuroectoderm-think CKS.
14 dendrocytes, astroC) tcs, epcnd} mal cells, pineal
15 gland), retina, spinal cord.
16 Neural crest P S (dorsal root ganglia, cranial nerves, 'cural crest- think Pi Sand non-neural
17 autonomic ganglia, Schwann cells), structures nearby.
18
melanocytes, chromaffin cells of adrenal
medulla, para follicu lar (C) cells of thyroid,
19
pia and arachnoid, bones of the skull,
20 odontoblasts, aortieopulmonary septum,
21 endocardial cushions, myenteric ( ucrbach)
. 22 plexus.
• 23 Mesoderm l\'luscle, bone, connective tissue, serous 1\ Iiddlet'meat" layer.
• 24 linings of body cavities (eg, peritoneum), lesodermal defects = VACTERL:
spleen (deri,·ed from foregut mesentery), \ ertebral defects
• 25
•• 1 •• 1 I ' I

a
Lock
s
Suspend
8
End Bl ock
Item: 21 of 25 ~ 1 • M k -<:J 1>- Jil ~· !:';-~
QIO: 2622 ..L ar Pre v ious Next Lab fli!ltues Notes Calcula t o r

5 g g ' ),
melanocytes, chromaffin cells of adrena l
6
medulla, para follicu lar (C) cells of thyroid,
7 pia and arachnoid, bones of I he skull,
8 odontoblasts, aorticopulmonary septum,
9 endocardial cushions, myenteric (Auerbach)
10
plexus.
11 Mesoderm luscle, bone, connective tissue, serous \1 iddlermeat" lm·er.
linings of body cavities (eg, peritoneum), ~ l esodcrma ldefects= VACTERL:
12
spleen (deri\ ed from foregut mesentery), \ ertebral defects
13 cardio,-ascular structures, lymphatics, blood, Anal atresia
14 wall of gut tube, upper ,·agina, l..idneys, Cardiac defects
15 adrenal cortex, dermis, testes, ovaries. Tracheo-Esophageal fi stula
16 'otochord induces ectoderm lo form Renal defects
neuroectoderm (neural plate). Its only Limb defects (bone and muscle)
17
postnatal derivative is the nucleus pulposus of
18 the intervertebral disc.
19
Endoderm Gut tube epithelium (including ana l canal ")'•.nterna 1" 1ayer.
20 above the pectinate line), most of urethra and
21 lower vagina (derived from urogenital sinus),
. 22 luminal epithelial derivatives (eg, lungs,
. 23
li\'er, gallbladder, pancreas, eustachian lube,
thr mus, parathyroid, thyroid follicular cells).
. 24
• 25

a
Lock
s
Suspend
8
End Bl ock
Item: 22 of 25 ~ 1 • M k -<:J 1>- Jil ~· !:';-~
QIO: 1667 ..L ar Pre v ious Next Labfli!llues Notes Calcula t o r

IAA]
A A
5
A 2-week-old girl is brought to the pediatrician for her first doctor's appointment after receiving no prenata l
6 care . Physical examination reveals a tuft of hair on her lower back, and an abnormality is noted on x-ray, as
7 shown in the image. Ultrasound shows no herniation of any kind .
8
9
10
11
12
13
14
15
16
17
18
19
20
21
. 22
Wh ich of the following cond itions does this baby have?
• 23
:
• 24
A. Anencephaly
• 25

a
Lock
s
Suspend
8
End Bl ock
15
16 Which of the fol lowing cond itions does this baby have?
17
:
18 A. Anencephaly
19
B. Men ingocele
20
21 C. Men ingomyelocele
. 22 D. Spina bifida cystica
. 23
E. Spina bifida occulta
. 24
• 25

a
Lock
s
Suspend
8
End Block
Item: 22 of 25 ~. I • M k <:] t> al ~· ~
QIO: 1667 .l. ar Previous Next lab 'lifllues Notes Calculator


5
The correct answer is E. 81 °/o chose this.
6
Spina bifida occu lta is caused by the failure of the posterior vertebral arches to close in utero, as shown by the
7
circle in image. Genera lly there are no associated clinica l abnorma lities, or a tuft of hair may be present over
8 the site of the defect. Folic acid given before the 28th day of pregnancy can help prevent its occurrence.
9 Spina bifida Folic acid In utero Vertebra Pregnancy Spina bifida occulta

10 A is not correct. 1 °/o chose this.


11 Anencepha ly occurs when the cepha lic end of the neura l tube fa ils to close, resu lt ing in the absence of a major
12 portion of the brain, sku ll, and sca lp. Infants with this disorder are born without a forebrain and a cerebrum .
The rema ining brain tissue is often exposed . This condition is incompatible with life.
13
Anencephaly Cerebrum Neural tube Forebrain Scalp Human skull Human brain Skull Brain Head
14
15 B is not correct. 5°/o chose this.
Men ingocele is the herniation of meninges (but no nerve tissue) through a bony defect, often located at the
16
sacrum. The ultrasound revealed no herniation, making th is diagnosis un likely.
17 Meninges Spina bifida Sacrum Ultrasound Medical ultrasound Nerve Spinal disc herniation Nervous tissue

18
C is not correct. 4°/o chose this.
19
Men ingomyelocele occurs when there is hern iation of both the meninges and spinal cord through a spinal canal
20 defect, wh ich has not happened in th is patient.
21 Meninges Spina bifida Spinal canal Spinal cord

22
D is not correct. 9°/o chose this .
• 23 Spina bifid a cystica is caused by the fai lure of the posterior vertebral arches to close, along with external
• 24 protrusion of a saclike structu re. This is further classified according to the extent of neural involvement ( eg,
• 25 meningocele or meningomye locele) . Note that hern iation has not taken place in this patient, ma king this answer
~

6
lock
s
Suspend
0
End Block
Item: 22 of 25 ~. I • M k <:] t> al ~· ~
QIO: 1667 .l. ar Previous Next lab 'lifllues Notes Calculator
-- - . ••- •- •- -·•• -• • • • . - ._ .-u -
5
Anencephaly Cerebrum Neural tube Forebrain Scalp Human skull Human brain Skull Brain Head
6
B is not correct. 5°/o chose this.
7
Men ingocele is the herniation of meninges (but no nerve tissue) through a bony defect, often located at the
8
sacrum. The ultrasound revealed no herniation, making th is diagnosis un likely.
9 Meninges Spina bifida Sacrum Ultrasound Medical ultrasound Nerve Spinal disc herniation Nervous tissue

10
C is not correct. 4°/o chose this.
11
Men ingomyelocele occurs when there is hern iation of both the meninges and spinal cord through a spinal canal
12 defect, wh ich has not happened in th is patient.
13 Meninges Spina bifida Spinal canal Spinal cord

14 D is not correct. 9°/o chose this.


15 Spina bifid a cystica is caused by the fai lure of the posterior vertebral arches to close, along with external
16 protrusion of a saclike structure. This is further classified according to the extent of neural invo lvement ( eg,
17
meningocele or meningomye locele) . Note that hern iation has not taken place in this patient, making this answer
incorrect.
18 Spina bifida Vertebra Spinal disc herniation
19
20
Bottom Line:
21
22
Spina bifida occu lta is a mild neural-tube defect, usual ly found incidental ly, that is caused by failure of the
dorsa l segments of the vertebra to fuse during deve lopment .
• 23 Spina bifida Vertebra Neural tube defect Spina bifida occulta Anatomical terms of location Dorsum (biology)
• 24
• 25 ~

6
lock
s
Suspend
0
End Block
Item: 22 of 25 ~ 1 • M k -<:J 1>- Jil ~· !:';-~
QIO: 1667 ..L ar Pre v ious Next Labfli!llues Notes Calcula t o r
A A
5
6 FA17 p 461 .1

7 Neural tube defects Neuropores fail to fuse (4th week) - persistent connection between amniotic cavity and spinal
8 canaL ssociated '' ith maternal diabetes as ''ell as low folic acid intake before conception and
9
during pregnancy. f a-fetoprotein (AFP) in amniotic Auid and maternal serum (except spina
bifida occulta = normai ,\FP). f acetylcholinesterase ( ChE) in amniotic Auid is a helpful
10
confirmatory test (fetal AChE in CSF Aows through defect into amniotic Auid).
11
Anencephaly Failure of rostral neuropore to close - no forebrain, open cah-arium. Clinical findings:
12 polyhydramnios (no swallowing center in brain).
13 Spina bifida occulta Failure of caudal neuropore to close, but no herniation. Usually seen at lower ,·ertcbral b ·cls. Dura
14 is intact. Associated \\'ith tuft of hair or skin dimple at le\'el of bony defect.
15 Meningocele 1\ leninges (but no neural tissue) herniate through bon} defect. Associated with spina bifida crstica.
16 Meningomyelocele 1\ len inges and neural tissue (cg, cauda equina) herniate through bony defect.
17
·'"~ +1- Tuft of hair
18 ~ +I- Skin d1mple
Skin ~
19
20
21
22
!f:;.,.;.,.,
. 23
Sptnal ...,.;? \_Transverse
. 24 cord process
• 25

a
Lock Suspend
s 8
End Bl ock
Item: 22 of 25 ~ 1 • M k -<:J 1>- Jil ~· !:';-~
QIO: 1667 ..L ar Pre v ious Next Labfli!llues Notes Calcula t o r
A A
5
6 FA17 p65. 1

7 Vitamin 8 12 (cobalamin)
8 FUNCTION Cofactor for methionine synthase (transfers Found in animal products.
9 C H3 groups as meth) !cobalamin) and S~ nthesized only by microorganisms. Ver) large
methylmalonyl-CoA mutase. Important for reserve pool (se,·eral years) stored primarily in
10
DNA sp1thesis. the Ji,·er. Deficiency caused by malabsorption
11 (eg, sprue, enteritis. Diphyllobothrium latum),
DEFICIENCY ~lacrocytic, megaloblastic anemia;
12
hrpersegmented P I 's; paresthesia~ lack of intrinsic factor (pernicious anemia,
13 and subacute combined degeneration gastric bypass surgery), absence of terminal
14 (degeneration of dorsal columns, lateral ileum (~urgica l resection, cg, for Crohn
corticospinal tracts, and spinocerebellar tracts) disease), or insufficient intake (eg, veganism).
15
due to abnormal myelin. Associated with Anti-intrinsic factor antibodies diagnostic for
16 . . .
t serum homocysteine and mcthylmalonic perniCIOUS anem1a.
17
acid le,·els, along with zofolate deficiency.
18 Prolonged deficiency - irreversible nerve
19 damage.
20
Protein Fatty acids with odd number of
21 t carbons, branched·chain amino acids
22 THF Methionine SAM l
CH1 to anabol1c •
. 23 pathways Methylmalonyi·CoA
. 24 S-adenosyl B Mel.hytm ton ti..CoA
Methiornne synthase muta:.c
homocysteine
• 25

a
Lock Suspend
s 8
End Bl ock
Item: 22 of 25 ~ • -<:J. I> ~ ~· ..,-,,n
1
QIO: 1667 ..L 1
M a rk
Next Lab lues
, ;r
Notes
,. . .
Cal culat o r
Prev10us

5 DNA synthes1s. the liver. Deficiency caused by malabsorpl ion •


DEFICIENCY lacrOC)tic, megaloblastic anemia; (eg, sprue, enteritis, Diphyllobothrium /alum),
6
hypersegmented P 11 s; paresthesias lack of intrinsic factor (pernicious anemia,
7 gastric bypass surgery), absence of term ina I
and subacute combined degeneration
8
(degeneration of dorsal columns, lateral ileum (surgical resection, eg, for Crohn
9 corticospinal tracts, and spinocerebellar tracts) disease), or insufficient intake (eg, veganism).
10 due to abnormal myelin. Associated with Anti-intrinsic factor antibodies diagnostic for
t serum homocysteine and mel h) Imalonic pernicious anemia.
11
12
acid le,·els, along'' ith 2° folate deficienc\.
Prolonged deficiency - irre,·ersiblc ncrv~
13
damage.
14
Protetn Fatty acids with odd number of
15
carbons. branched-chain amino acids
16 Methionine SAM ;

17
~ CH1to anaboliC ;
\ patnways Methytmalonyi·CoA
18
Methionme synthase
S·adenosyt B 1Methylmatonyi·CoA
mutase
19 homocysteine

~
Succinyt·CoA
20
21
22
Homocysteine

B Adenosine
V'\
Heme TCA
. 23
Cysteine
. 24
• 25

! s
Suspend
8
End Block
Item: 24 of 25 ~ 1 • M k -<:J 1>- Jil ~· !:';-~
QIO: 2937 ..L ar Pre v ious Next Labfli!llues Not es Calcula t o r

IAA]
A A
5
A 24-year-old man presents to the neurologist's office ho lding his arm. He ind icates that he was invo lved in a
6 gang fight 1 week ago, during wh ich time he was stabbed from behind in his right arm. On physical
7 examination, he is unable to extend his right wrist and has decreased sensation to the majority of the
dorsum of his right hand.
8
9
What observations would be expected on a muscle biopsy distal to the patient's nerve transection?
10
11 :
A. CD4+ mediated perimysial inflammation
12
13 B. Degenerative myocytes with interstitial fibrofatty infi ltrate
14 C. Ragged-red fibers on trichrome stain
15
D. Small, angulated muscle fibers
16
17
E. Transfasicu lar and intracellular CDS+ mediated inflammation

18
19
20
21
22
. 23
. 24
• 25

a
Lock
s
Suspend
8
End Bl ock
Item: 24 of 25 ~. I • M k <:] t> al ~· ~
QIO: 2937 .l. ar Previous Next lab 'lifllues Notes Calculator


5
The correct answer is D. 42°/o chose this.
6
This patient is most likely suffer ing from a t ransection of his
7 radia l nerve, resu lt ing in wrist drop. After transection of a
8 peripheral nerve fiber, degeneration of the dista l nerve
section begins almost immediately. Wa ller ian degeneration
9
(a lso cal led orthog rade degeneration) refers specifical ly to
10 the process whereby nerve fibers (general ly the axon) dista l
11 to the site of t ransection are lost as they are cut off from the
12 soma (as shown in image A, arrows), the sou rce of metabol ic
nou r ishment. As a resu lt of denervation, skeletal muscles
13
atrophy. Microscopically, denervation atrophy of skeletal Images copyright © 2013 Lai eta!.; licensee
14 muscle appears as sma ll muscle fibers with a decreased BioMed Central Ltd.
15 cross-sectional area, which are angulated or t riangu lar and
ar ranged in small groups (as shown in image B, circles) .
16
Wallerian degeneration Axon Radial nerve Nerve fiber Soma (biology) Skeletal muscle Denervation Atrophy Anatomical terms of location Wrist drop Muscle
17
18
A is not correct. 10% chose this.
19
Der matomyositis is an inflammatory, autoimmune condition character ized cl inical ly by proximal muscle
wea kness. Biopsy of affected tissue reveals infiltration of CD4 + T lymphocytes in the pe rimysium of muscle
20 tissue.
21 Dermatomyositis Biopsy lymphocyte T cell Autoimmunity Muscle weakness T helper cell Muscle Autoimmune disease Anatomical terms of location

22 B is not correct. 28% chose this .


• 23 This histopatholog ic description corresponds to Duchenne muscu lar dystrophy (DMD), an X- linked muscular
24 dystrophy that is often associated with Gowe r sign (a pattern of using upper extremit ies to rise from the ground
rather than lower extremities) and pseudohypertrophy of the ca lves. Biopsy revea ls significant fat and
• 25 ~ . . . . . . ... .
6
lock
s
Suspend
0
End Block
Item: 24 of 25 ~. I • M k <:] t> al ~· ~
QIO: 2937 .l. ar Previous Next lab 'lifllues Notes Calculator


5
B is not correct. 28% chose this.
6
This histopatholog ic description corresponds to Duchenne muscu lar dystrophy (DMD), an X- linked muscular
7
dystrophy that is often associated with Gower sign (a pattern of using upper extremit ies to rise from the ground
8 rather than lower extremities) and pseudohypertrophy of the ca lves. Biopsy revea ls significant fat and
9 connective t issue in place of muscle.
Ouchenne muscular dystrophy Muscular dystrophy Connective tissue Biopsy Histopathology Sex linkage Fat Calf (leg) Muscle
10
11 C is not correct. 11% chose this.
12 Mitochondrial myopathies are debi litating and often lethal conditions that manifest with weakness and other
13
abnorma lities. Histo log ic examination reveals " ragged-red" fibers in disa rray with mitochond r ial changes, some
described as " parking-lot" inclusions based on the crystal line deposits seen intrace llular ly. Mitochondrial
14
encepha lomyopathy, lactic acidosis, and stroke- like episodes (MELAS) is one such disease.
15 Mitochondrial myopathy MELAS syndrome lactic acidosis Myopathy Mitochondrion Acidosis Encephalomyopathy Mitochondrial DNA

16
E is not correct. 9°/o chose this.
17
This histopatholog ic description corresponds to po lymyositis, an autoimmune condition with features simila r to
18 de rmatomyositis. T lymphocytes that are CDS+ are found in muscle t issue upon histologic examination.
19 Polymyositis Dermatomyositis lymphocyte Autoimmune disease T cell Autoimmunity Histopathology Histology cos Muscle

20
21 Bottom Line:
22
Deinne rvation leads to muscula r atrophy . Histologic findings include smal l groups of angulated muscle fibe rs
• 23 with decreased cross-sectional area .
24 Histology Atrophy Muscle atrophy Skeletal muscle Muscle Myocyte

• 25 ~

6
lock
s
Suspend
0
End Block
Item: 24 of 25 ~. I • M k <:] t> al ~· ~
QIO: 2937 .l. ar Previous Next Lab 'lifllues Notes Calculator

6
lock
s
Suspend
0
End Block
Item: 24 of 25 ~ 1 • M k -<:J 1>- Jil ~· !:';-~
QIO: 2937 ..L ar Pre v ious Next Labfli!llues Not es Calcula t o r
A A
5 Median (CS-Tl ) Supracondylar fracture of humerus (proximal ''Ape hand" and "Pope's blessing"
6 lesion); carpal tunnel syndrome and wrist Loss of \\'risl Aexion, Aex ion of lateral fingers,
laceration (distal lesion) thumb opposition, lumbricals of 2nd and 3rd
7
digits
8 Loss of sensa~ion o,·er thenar eminence and
9 dorsal and palmar aspects of lateral 3 ~ fingers
10 "ith proximal lesion
11 Ulnar (CS-Tl) Fracture of medial epicondyle of humerus "Ulnar claw" on digit extension
"funnr bone" (pro>.imallcsion); fractured Radial de' iation of" rist upon fl exion (proximal
12
hook of hamate {distal lesion) from fall on lesion)
13 Loss of \\Tist Aexion, Aexion of medial fingers,
outstretched hand
14 abduction and adduction of fingers (interossei),
15 actions of medial 2 lumbrical muscles
16
Loss of sensation over medial Jlh fingers
including hypothenar eminence
17
Recurrent branch of Superficial laceration of pa lm " pe han d"
18
median nerve (CS-Tl ) Loss of thenar muscle group: opposition,
19 abduction, and Acxion of thumb
20 o loss of sensation
21 l lumerus fractures, proximallr to distally, follow the ARI\ 1 ( \ xillary- Radia l - \1edian)
22 cs
(6
0
• 23 an
~Median nerve
24 Axillary nerve
• 25 Ulnar nerve --.........
.;- lntercostobrach~al

a
Lock
s
Suspend
8
End Bl ock
5 • .....__Radial nerve
Rad1al nerve ~
_....- Medial brachial
6 cutaneous nerve Palmof hand
7
Median nerve _ / ' - Ulnar nerve Medial antebrachial
8 Musculocutaneous nerve--....
cutaneous nerve Median nerve --......__,.
9
Racf1al nerve /
~ Ulnar nerve
10 Radial nerve --......__,.
Recurrent branch """"""
11 of med1an nerve
12 Dorsum of hand

13
14
15 FA17 p 4 65.1

16 Peripheral nerve Endoneurium- invests single nerve fi ber layers F:ndo = inner.
17 ~ Nerve trunk (inAammatory infiltrate in Cu illain-B<m c Peri = around.
syndrome). Epi = outer.
18 ~~Epineurium
Perineurium (blood-nerve Pcnneabi Iit y
19 ~Perineurium
barrier)-surrounds a fascicle of ncn·c fibers.
_.........-Endoneurium
.,.('
20 Must be rejoined in microsurgery for limb
21
~-Nerve fiber reattachment.
Ill
22 Epineurium-dense con necti\'C tissue that
• 23
surrounds entire ner\'e (fasc icles and blood
\'essels).
24
• 25

a
Lock Suspend
s 8
End Block
Item: 25 of 25 ~ 1 • M k -<:J 1>- Jil ~· !:';-~
QIO: 2658 ..L ar Pre v ious Next Lab fli!ltues Notes Calcula t o r

IAA]
A A
5
A 40-year-old woman presents to the physician because of progressive hearing loss and impaired balance.
6 MRI of the brain shows bilateral masses at the cerebellopontine angle.
7
8 The patient's intracran ial masses arise from cells serving which purpose?
9
:
10 A. Conduction of electrical impulses
11
B. Maintenance of the blood - brain barrier
12
C. Myelination of central nervous system axons
13
14 D. Myelination of periphera l nervous system axons
15 E. Production of cerebrospinal flu id
16
17
18
19
20
21
22
. 23
24
• 25

a
Lock
s
Suspend
8
End Bl ock
Item: 25 of 25 ~. I • M k <:] t> al ~· ~
QIO: 2658 .l. ar Previous Next lab 'lifllues Notes Calculator


5
6 The correct answer is D. 60°/o chose this.
7 The patient has ne urofibromatosis type 2 (N F- 2), an autosomal dominant disorder in which bilatera l acoustic
8
neuromas (the masses mentioned in the vignett e) develop in > 90% of affected individua ls. Character istics of
NF- 2 include schwannomas (tumors of Schwann cell origin), mening iomas, and ependymomas . Reca ll that
9
Schwann cel ls are der ived from neural crest cells and function to myelinate periphera l nervous system axons.
10 Although NF- 2 patients do not genera lly have neu rofibromas (wh ich are seen in NF- 1), they can have any
11 number of cutaneous find ings, such as plaque-l ike lesions, subcutaneous nodules, and/or schwanomas, like this
patient. Patients who develop schwannomas of the vestibu lococh lear nerve (also cal led acoustic neuromas) may
12
present with symptoms of hear ing loss or deter iorating balance. The presence of bilateral acoustic neuromas is
13 v irtua lly pathognomon ic of NF-2 .
14 Schwann cell Vestibulocochlear nerve Neurofibromatosis type II Neurofibromatosis Dominance (genetics) Peripheral nervous system Neural crest Meningioma

15 Vestibular schwannoma Axon Neuroma Neurofibroma Hearing loss Nervous system Neoplasm Schwannoma

16
A is not correct. 9°/o chose this.
17 Neurons are cel ls that relay signals in both the per ipheral and centra l nervous systems . These signals are
18 transported via axons and dendrites, extensions of neurona l cel l bod ies. Neurons themselves are not affected in
19 neurofibromatosis type 2 .
Neurofibromatosis Dendrite Axon Soma (biology) Neuron
20
21 B is not correct. 7°/o chose this.
22 Astrocytes are critica l in the maintenance of the blood -bra in ba r rier. It would be very un like that bilateral
astrocytomas could account for this patient's presenting symptoms and imaging find ings .
• 23
Blood-brain barrier Astrocytoma Astrocyte
24
C is not correct. 18% chose this.
25 ~

6
lock
s
Suspend
0
End Block
Item: 25 of 25 ~. I • M k <:] t> al ~· ~
QIO: 2658 .l. ar Previous Next lab 'lifllues Notes Calculator

5 neurofibromatosis type 2.
Neurofibromatosis Dendrite Axon Soma (biology) Neuron
6
7 B is not correct. 7°/o chose this.
8 Astrocytes are crit ical in the maintenance of the blood-brain ba r rier. It would be very un like t hat bilateral
astrocyto mas could account for this patient's presenting symptoms and imaging findings.
9
Blood-brain barrier Astrocytoma Astrocyte
10
11
C is not correct. 18% chose this.
Ol igodendrocytes are cells t hat mye linate central nervous system neurons. Althoug h ol igodend rocytes and
12
Schwann are both involved in myelination, o ligodendrocytes are not affected in neurofibromatosis type 2.
13 Central nervous system Neurofibromatosis Myelin Oligodendrocyte Neuron Nervous system
14
E is not correct. 6°/o chose this.
15
Cereb rospina l fluid is produced by ependymal ce lls, which form the inner lining of the ventricles. Although
16 patients with neurofibromatosis type 2 may indeed deve lop ependymomas, the location (and bi late ral ity) of th is
17 patient's masses, along with her symptoms, suggest that the MRI masses are acoustic neuromas.
Cerebrospinal fluid Ependyma Neurofibromatosis type II Neurofibromatosis Vestibular schwannoma Ventricular system Magnetic resonance imaging Neuroma
18
Ventricle (heart)
19
20
21 Bottom Line:
22 Neu rofib romatosis type 2 is an autosomal dominant disorder in which >90% of patients develop bilateral
• 23 acoustic neuromas (Schwann cell tumo rs of the vestibu locochlear nerve) .
Schwann cell Vestibulocochlear nerve Neurofibromatosis type II Neurofibromatosis Dominance (genetics) Vestibular schwannoma Neuroma Neoplasm
24
25

6
lock
s
Suspend
0
End Block
Item: 25 of 25 ~- I • M k <:] t> al ~· ~
QIO: 2658 .l. ar Previous Next lab 'lifllues Notes Calculator
~ •
YP • I • • I. p • p •
5
acoustic neuromas (Schw ann cell tumo rs of the vestibu locochlear nerve) .
6 Schwann cell Vestibulocochlear nerve Neurofibromatosis type II Neurofibromatosis Dominance (genetics) Vestibular schwannoma Neuroma Neoplasm

7
8
9 1iU;fii!1J•J for year:[ 2017
FIRST AID FAC T S
"
10
11
FA17 p464.2
12
Schwann cells Each Schwann cell myclinatcs only 1 P 1S axon. May be injured in Guillain-Barre syndrome.
13 Also promote axonal regeneration. Derived
14 from nemal crest.
15
16
Myelin sheath
17
18
19 FA17p56.1
20
Autosomal dominant Achondroplasia, autosomal dominant polycystic kidney disease, fam ilial adenomatous polyposis,
21 diseases familial hypercholesterolemia, hereditary hemorrhagic telangiectasia, hereditary spherocytosis,
22 Huntington disease, Li-F'raumeni syndrome, Marfan syndrome, multiple endocrine neoplasias,
• 23 neurofibromatosis type I (,·on Reckl inghausen disease}, neurofibromatosis type 2, tuberous
24
sclerosis, von 1-1 ippei-Lindau disease.

25 ~

lock
6 s
Suspend
0
End Block
Item: 23 of 25 ~ 1 • M k -<:J 1>- Jil ~· !:';-~
QIO: 3346 ..L ar Pre v ious Next Lab fli!ltues Not es Calcula t o r

IAA]
A A
5
A postmortem examination of a 45-year-old patient with a known trisomy revea ls a specific neuropathology.
6 Hisopathology is shown below:
.,-----,.,...,...,~

7
8
9
10
11
12
13
14
15
16
17
18
19 I mage courtesy if Wikimedia Commons

20
21
What protein is found in the cytoplasmic inclusions of neurons in the image?
22 :

• 23 A . Amyloid precu rsor protein


24 B. Amyl o i d-~
25 r Ann li nnnrnt.,.i n I=

a
Lock
s
Suspend
8
End Bl ock
I mage courtesy if Wikimedia Commons
15
16 What prote in is found in the cytoplasm ic inclusions of neurons in the image?
17
:
18 A. Amyloid precursor protein
19
B. Amy l o i d-~
20
21 C. Apolipop rotein E
22 D. Glial fibrillary acid ic protein
• 23
E. Tau
24
25

a
Lock
s
Suspend
8
End Block
Item: 23 of 25 ~ 1 • M k -<:J 1>- Jil ~· !:';-~
QIO: 3346 ..L ar Pre v ious Next Lab fli!ltues Not es Calcula t o r
A A
5
6 The correct a nswer is E. 55°/o chose this.
7 The image shows a number of neurofibrillary tangles consistent w ith a
pathologic diagnosis of Alzheimer disease (AD). Neurofibrillary tangles are
8
intracellular inclusions formed primarily of the cytoplasmic microtubule
9 binding protein tau. They are found in AD and a number of other
10 neurodegenerative diseases known as tauopathies. The other primary
pathologic finding in AD is extracellular amyloid plaques, which are faintly
11
visible in the image as large homogeneous pink areas. Like other amyloid
12 protein deposits, amyloid plaques are best seen using the birefringence of the
13 Congo red stain (as shown here) . Interestingly, the precursor to the primary
14 component of these amyloid lesions has been loca lized to chromosome 21.
This finding is thought to explain why patients with Down syndrome, al l of
15
whom have three copies of chromosome 21, frequently develop AD before Image copyright © 2009 Briggs
16 age SO years. eta!.; licensee Cases Network
Congo red Down syndrome Microtubule Tauopathy Birefringence Amyloid Neurofibrillary tangle Ltd
17
18 Alzheimer's disease Neurodegeneration Protein Chromosome 21 (human) Cytoplasm Chromosome Tau protein

19 A is not correct. 12°/o chose this.


20 Many t issues conta in amyloid precursor protein, but it is concentrated in neu rons . It is the parent protein of the
21 amy l oid-~ peptide, and mut ations in the gene that generat e ~-amy l o i d resu lt in familial susceptibility to
Alzheimer disease.
22
Amyloid precursor protein Gene Amyloid Protein Peptide Alzheimer's disease Neuron Mutation Amyloid (mycology)
23
24 B is not correct. 23°/o chose this.
25
Accumulations of abnormally folded amylo id-~ peptide deposit outside of neurons as senile plaques, not as
~ --· ·- -~:L..- : 11--. ~- --1-- ... L.:-L - - - -'··- &.- &.L- - L - - - - - 1 - - - ··-··1 - &.:-- -~ ..__ .. ___ ...__:_

a
Lock
s
Suspend
8
End Bl ock
Item: 23 of 25 ~. I • M k <:] t> al ~· ~
QIO: 3346 .l. ar Previous Next lab 'lifllues Notes Calculator

5
• amylo i d-~
peptide, and mutations in the gene that generate ~-amylo i d result in familia l susceptibi lity to
Alzheimer disease.
6
Amyloid precursor protein Gene Amyloid Protein Peptide Alzheimer' s disease Neuron Mutation Amyloid (mycology)
7
B is not correct. 23% chose this.
8
Accumulations of abnorma lly folded amylo i d-~ peptide deposit outside of neurons as seni le plaques, not as
9
neurofibr illary tangles, which are due to the abnormal accumu lation of tau protein.
10 Tau protein Senile plaques Neurofibrillary tangle Protein Neuron Peptide Dementia

11
C is not correct. 5°/o chose this.
12
Apolipoprotein E (ApoE) enhances the degradation of ~-amylo i d, but some ApoE isofo rms are more efficient
13 than others. In particu lar, the ApoE-e4 isofo rm is inefficient and is associated with an increased risk of
14 Alzheime r disease (AD). ApoE is not the protein most associated with the neurofibrillary tang les of AD.
Apolipoprotein E Protein isoform Neurofibrillary tangle Protein Apolipoprotein Alzheimer' s disease
15
16 D is not correct. 5°/o chose this.
17 Glial fibrillary acid ic protein is a no rmal intermediate filament protein found in glia l cel ls. It contributes to the
18
mechanica l integ rity of the cel l and is a useful marker for gl ial cells. It is not associated with the neu rofibrilla ry
tangles of Alzheime r disease.
19 Intermediate filament Glial fibrillary acidic protein Neuroglia Neurofibrillary tangle Protein Alzheimer' s disease Acid
20
21
Bottom Line:
22
23 I nd ividuals with Down synd rome have an increased risk of developing Alzheime r disease, which has
character istic intracellular neu ral inclusions formed primarily of the protein tau.
24 Down syndrome Protein Alzheimer' s disease Tau protein Autism
25 ~

6
lock
s
Suspend
0
End Block
Item: 23 of 25 ~ 1 • M k -<:J 1>- Jil ~· !:';-~
QIO: 3346 ..L ar Pre v ious Next Lab fli!ltues Not es Calcula t o r

A FIRST AID FAC T S A


5
6
FA17 p59.1
7 Autosomal trisomies
8
Down syndrome Findings: intellectual disabilit~, Aat facies, incidence 1:700.
9 (trisomy 21 ) prominent epicanthal folds, single palmar Orin king age (21).
10 crease, gap between 1st 2 toes, duodenal \ l o~t common ,.i<lble chromosomal disorder and
11 atresia, Hirschsprung disease, congenital heart most common cause of genetic intellectual
disease (eg, atrioventricular septal defect), disability.
12
Brushfield spots. ssociated with early-onset First-trimester ultrasound commonly shows
13
Alzheimer disease (chromosome 21 codes for t nuchal translucency and hypoplastic nasal
14 amyloid precursor protein) and t risk of ALL bone; 1 serum PAPP-A, t free P-hCC.
15 and A~ I L . Second-trimester quad screen shows
16 95% of cases due to meiotic nondisjunction 1 a-fctoprotein, t P-hCC, 1 estriol,
(t with advanced materna I age; from I:1500 in t inhibin A.
17
women< 20 to l :25 in women> 45 yea rs old).
18
4% of cases clue to unbalanced Robertson ian
19 translocation, most typically between
20 chromosomes 14 and 21. 1% of cases due
21 to mosaicism (no association with materna l
22
nondisjunction; postfertilization mitotic error).
23 Edwards syndrome Findings: PRTKCF: Edward-Prominent Incidence 1:8000.
(trisomy 18) occiput, Rocker-bottom feet, Intellectual Election age (18).
24
disability, "\ondisjunction, Clenched fists 2nd most common autosomal trisomy resulting
25 fn •it-1.. 1"\'l"~:t~orl"lnt"'\;nn ht'lo.nrc\ ln"'-~"'t l t",..rc in 1i, ·~ hirth / n-.nc+- IV\0"\t"'nt"H"l ic llnu n "'""r1rnt••·u~\

a
Lock Suspend
s 8
End Bl ock
Item: 23 of 25 ~ 1 • M k -<:J 1>- Jil ~· !:';-~
QIO: 3346 ..L ar Pre v ious Next Lab fli!ltues Not es Calcula t o r

5
Edwards syndrome Find ings: PRJ!\CF: Edward- Prominent Incidence 1:8000.
6
(trisomy 18) occiput, Rocker-bottom feel, I ntelleclua I Election age (1 8}.
7
disability, " ondisjunction, C lenched fists 2nd most common autosomal trisomy resulting
8 (with O\·erlapping fingers}, low-set Ears, in li\·e birth (most common is Down syndrome}.
9 micrognathia (small ja\\'}, congenital heart PAPP- and free ~-hCG are l in first trimester.
10 disease. D eath usually occurs by age I. Quad screen shows l a-fetoprotein, l ~-hCC,
11
l estriol, l or nonnal inhibin A.

12 Patau syndrome Findings: se\·ere intellectual disabilit), rocker- Incidence 1:15,000.


(trisomy 13) bottom feet, microphthalmia, microcephaly, P uberty ( 13).
13

l
cleft liP/Palate, holoP rosencephalr, First-trimester pregnancy screen sho\\'S l free
14
Pol)dact) ly, cutis aPlasia, congenital heart ~-hCG, l P. PP-, \.
15 disease. Death usuallr occurs br age I.
16 Nondi.sjunction in meiosis I Nondisjunction in meiosis II
17
18
tJ Meiosis I
19
20 t-
. ., , , "". A
... <;
21
22
23 c) c) c) c)
c) Meiosis II
24

'
c) c)
25 • IU--~: ...: .•- -'-!--

a
Lock
s
Suspend
8
End Bl ock
Item: 23 of 25 ~ 1 • M k -<:J 1>- Jil ~· !:';-~
QIO: 3346 ..L ar Pre v ious Next Lab fli!ltues Not es Calcula t o r

5
6 Patau syndrome Findings: severe intellectual clisabilil), rocker- Incidence 1:15,000.
(trisomy 13) bottom feet, microphthalmia, microcephaly, Puberty (13).
7
cleft liP/Palate, holoProsencephaly, First-trimester pregnancy screen shows 1 free
8 Pol) dact)ly, cutis a Plasia, congenital heart P-hCG, 1 PAPP-A.
9 disease. Death usually occurs by age I.
10 Nondisjunction in meiosis I Nondisjunction in meiosis II
11
12
Meiosis I
13
14
15 Nondisjunction

16
17 c' c' c' c'
18 c' Meiosis II

19
c' c' '
) ~''"'"i"""''"
20
21 A A A
22
23
Ill Ill I ~ Gametes
II II I Ill
n+1 n+1 n-1 n- 1 n n n- 1 n+l
24
Trisomy Monosomy Normal Monosomy Trisomy lil
25 •

a
Lock
s
Suspend
8
End Bl ock
Item: 23 of 25 ~. I • M k <:] t> al ~· ~
QIO: 3346 .l. ar Previous Next lab 'lifllues Notes Calculator

• •
5
6 FA17 p490.1

7 Neurodegenerative l in cognitive ability, memory, or function with intact consciousness.


8 disorders
DISEASE DESCRIPTION HISTOLOGIC/GROSS FINDINGS
9
10
Parkinson disease Parkinson TRAPS your body: Loss of dopam inergic neurons (ie,
Tremor (pill-rolling tremor at rest) depigmentation) of substantia nigra pars
11
Rigidity (cogwheel) compacta.
12 Akinesia (or bradykinesia) Lewy bodies: composed of a-synuclein
13 Postural instability (intracellular eosi noph ilic inclusions t:i.l).
14 ShufAing gait
15
MPTP, a contaminant in illegal drugs, is
metabolized to lPP+, which can cause

~
16
parkinsonian symptoms.
17
Huntington disease Autosomal dominant trinucleotide (CAG)n Atrophy of caudate and putamen with ex vacuo
18 repeat disorder on chromosome 4. Symptoms ventriculomega ly.
19 manifest between ages 20 and 50: chorea, t dopamine, l GABA, l ACh in brain. 1 euronal
20 athetosis, aggression, depression, dementia death via MDA-R binding and glutamate
21 (sometimes initially mistaken for substance excitotoxicity.
abuse).
22
Anticipation results from expansion of CAG
23 repeats. Caudate loses ACh and GABA.
24
Alzheimer disease Most common cause of dementia in elderly. Widespread cortical atrophy (normal cortex IJ);
25 • Down svndrome natients hm·e t risk of cortex in Alzheimer dise:1se ri). esneciallv •

6
lock
s
Suspend
0
End Block
Item: 23 of 25 ~ 1 • M k -<:J 1>- Jil ~· !:';-~
QIO: 3346 ..L ar Pre v ious Next Lab fli!ltues Not es Calcula t o r

5
p '.
Alzheimer disease !\ lost common cause of dementia in elderh·. Widespread cortical atrophy (norma l cortex (l);
6 '
00\m syndrome patients ha,·e t risk of cortex in Alzheimer disease ), especially
7
de,·eloping Alzheimer disease, as APP is rn
hippocampus (arrows in and ). j arro\\ ing
8 located on chromosome 21. of g) ri and widening of sulci.
9 Associated with the folio" ing altered proteins: enile plaques m in gray matter: extracellular
10 ApoE2: ' risk of sporadic form 13-amyloid core; may cause am) loid angiopathy
poE4: t risk of sporadic form - intracranial hemorrhage; Al3 (amyloid-13)
11
• PP, presenilin-1, presenilin-2: familial srnthesized by clea,·ing amyloid precursor
12
forms (10%) with earlier onset protein (APP).
13 ' ACh 'eurofibrillary tangles 1]: intracellular,
14 hrperphosphorylated tau protein = insoluble
15 crtoskeletal elements; number of tangles
correlates with degree with dementia.
16
17
Frontotemporal Early changes in personality and behavior F'rontolemporallobe degeneration 0 .
dementia (Pick (behavioral variant), or aphasia (primary Inclusions of hyperphosphorylated tau (round
18
19
20
disease) progressive aphasia).
lay have associated movement disorders
(eg, parkinsonism, ALS-I ike UMN/L I
Pick bodies (!!) or ubiquitinated TDP-43.
I
21 degeneration).
22 Lewy body dementia Dementia and visual hallucinations Intracellular Lewy bodies rJ primarily in corte>..
23 ("ha Lewycinations") - parkinsonian features
24 Vascular dementia Result of multiple arterial infarcts and/or IRI or CT shows multiple cortical and/or
25 ~. . ..
chronic ischemia.
I • I • . • , I I ,
subcortical infarcts.

a
Lock Suspend
s 8
End Bl ock
Item: 23 of 25 ~ 1 • M k -<:J 1>- Jil ~· !:';-~
QIO: 3346 ..L ar Pre v ious Next Lab fli!ltues Not es Calcula t o r
A A
5 Step-wise decline in cognitive ability with late-
6 onset memory impairment. 2nd most common
7 cause of dementia in elderly.
8 Creutzfeldt-Jakob Rapidly progressi,·e (weeks to months) dementia Spongiform cortex.
disease with myoclonus ('"startle myoclonus"). Prions (Prpc - Prpsc sheet [~-pleated sheet
9
Commonly see periodic sharp waves on EEG resistant to proteases]) Cl
10
and t H-3-3 protein in CSF.
11
12
13
14
15
16
17
18
19
20
21
22
I
23
24
25

a
Lock
s
Suspend
8
End Bl ock
Item: 23 of 25 ~. I • M k <:] t> al ~· ~
QIO: 3346 .l. ar Previous Next lab 'lifllues Notes Calculator

• •
5
FA17 p213.1
6
Amyloidosis Abnormal aggregation of proteins (or their fragments) into ~ -plea ted linear sheets .... insoluble fibrils
7 -+ cellular damage and apoptosis. Amyloid deposits visualized by Congo red stain f.i.l, polarized

8 light (apple green birefringence) E), and H&E stain (~ shows deposits in glomerular mesangial
9 areas [white arrows], tubular basement membranes [black arrows]).
10 COMMON TYPES DESCRIPTION

11
AL(primary) Due to deposition of proteins from lg Light chai ns. Can occur as a plasma cell disorder or
associated with multiple myeloma. Often affects multiple organ systems, including renal
12
(nephrotic syndrome), cardiac (restrictive cardiomyopathy, arrhythmia), hematologic (easy
13 bruisi ng, splenomegaly), Gl (hepatomegaly), and neurologic (neuropathy).
14
AA (secondary) Seen with chronic inflammatory conditions such as rheumatoid arthritis, IBD,
15 spondyloarthropathy, familial Mediterranean fever, protracted infection . Fibrils composed of
16 serum Amyloid A. Often multisystem like AL amyloidosis.
17 Dialysis-related Fibri ls composed of ~Tm icroglobulin in patients with ESRD and/or on long-term dialysis. May
18 present as carpal tunnel syndrome.
19 Heritable Heterogeneous group of disorders, including familial amyloid polyneuropathies due to transthyretin
20 gene mutation.
21 Age-related (senile) Due to deposition of normal (wild-type) transthyretin (TTR) predominantly in cardiac ventricles.
22
systemic SIO\\·er progression of cardiac dysfunction relative to AL amyloidosis.

23 Organ-specific Amyloid deposition localized to a single organ. Most important form is amyloidosis in Alzheimer
disease due to deposition of ~-amyloid protein cleaved from amyloid precursor protein (APP).
24
Islet amyloid polypeptide (lAPP) is commonly seen in diabetes mellitus type 2 and is caused by
25 • ,-l~t"\t"\~il"inn nf 'ln'l\/1;,., in r·v.lnf"rP'3t-ir- idPt-c:o •

lock
6 s
Suspend
0
End Block
Item: 23 of 25 ~ 1 • M k -<:J 1>- Jil ~· !:';-~
QIO: 3346 ..L ar Pre v ious Next Lab fli!ltues Not es Calcula t o r
A
Heterogeneous group of disorders, includ ing familial amyloid polyneuropathies due to transthyretin
A
5 Heritable
6 gene mutation.
7 Age-related (senile) Due to deposition of normal (wild-type} transthyretin (TT'R) predominantly in cardiac ventricles.
systemic SJo,,·er progression of cardiac dysfunction rclati,·e to AL amyloidosis.
8
9
Organ-specific Amyloid deposition localized to a single organ. \!lost important form is amyloidosis in lzheimer
disease due to deposition of ~-am} loid protein clea,·ed from amyloid precursor protein ( PP).
10
Islet amyloid polypeptide (I PP) is common I} seen in diabetes mellitus type 2 and is caused by
11 deposition of amyl in in pancreatic islets.
12 Isolated atrial amyloidosis due to atrial natrimetic peptide is common in normal aging and can
13 predispose to increased risk of atrial fibrillation.
14 Amyloid deposition to ventricular endomyocardium in restrictive cardiomyopatl1y.

. -.
Calcitonin deposition in tumor cells in medullary carcinoma of the thyroid.
15
. ...,r ·
16 .,, ....
17
18 • j

• f
19
20
21
22
23
24
25

a
Lock
s
Suspend
8
End Bl ock

You might also like